You are on page 1of 194

HO CHI MINH CITY UNIVERSITY OF SCIENCE

FACULTY OF MATHEMATICS AND COMPUTER SCIENCE


________________________________________________________



________________________________________________________
LECTURER: PROF. DANG DUC TRONG



VO A. KHOANGUYEN T. HOAIDANG P. NHAT
TRAN T. HUNGLE T. NHAN

FUNCTIONS
OF
ONE COMPLEX VARIABLE
FUNCTIONS OF ONE COMPLEX VARIABLE
VO ANH KHOA - NGUYEN THANH HOAI - DANG PHUOC NHAT - TRAN
THE HUNG - LE THANH NHAN
February 15, 2012
Vo Anh Khoa - Nguyen Thanh Hoai - Dang Phuoc Nhat
Tran The Hung - Le Thanh Nhan
Vietnam National University
Ho Chi Minh City (HCMC) University of Science
Faculty of Mathematics and Computer Science
227 Nguyen Van Cu Street, District 5, Ho Chi Minh City
Vietnam
2
PREFACE
This book is intended as our English essay for students who study in university. In specic,
the essay consists of some exercises which are chosen and solved by us; and these exercises is
taken from A. David Wunsch, Complex Variables with Applications, 3rd ed. To read this essay,
the readers need some knowledges of Functions of One Complex Variable (such as second - year
students or higher), but if the reader is a student in high school, still he or she can reads some
exercises which is shown in chapter I. Then, we submit an essay to our tutor who is Prof. Dang
Duc Trong, that is also our lecturer.
Remark. We will present the following chapters :
1. Chapter 1 : Complex Number.
2. Chapter 2 : The Complex Function and Its Derivative.
3. Chapter 3 : The Basic Transcendental Functions.
4. Chapter 4 : Integration in the Complex Plane.
5. Chapter 5 : Innite Series Involving a Complex Variable.
6. Chapter 6 : Residues and Their Use in Integration.
Acknowledgement. We thank the collaborators for all their helps. These include :
Name University
Mai Thanh Nhat Truong HCMC University of Science
Tran Hong Tai HCMC University of Science
Vu Tran Minh Khuong HCMC University of Science
Ngo Thanh Ha HCMC University of Science
The reader can nd and download our Functions of One Complex Variable at :
http://anhkhoavo1210.wordpress.com/
And if you have any questions of feedback, please feel free to contact us at :
hambienphuck10@gmail.com
Note. Although this essay is free on sharing, we do not accept an arbitrary case
that relates to the copyright of it.
3
Bibliography
[1] A. David Wunsch (University of Massachusetts Lowell), Complex Variables with Applica-
tions, 3rd ed, 2005.
[2] Dau The Cap (Ho Chi Minh City University of Pedagogy), Ham Bien Phuc & Phep Tinh
Toan Tu, 2006.
[3] Terence Tao (University of California Los Angeles), Lecture Note of Complex Analysis.
[4] Dang Duc Trong - Dinh Ngoc Thanh - Pham Hoang Quan (Ho Chi Minh City University
of Science), Giao trinh Giai Tich 2, 2008.
4
Contents
PREFACE 3
1 COMPLEX NUMBERS 7
1.1 INTRODUCTION . . . . . . . . . . . . . . . . . . . . . . . . . . . . . . . . . . . 7
1.2 MORE PROPERTIES OF COMPLEX NUMBERS . . . . . . . . . . . . . . . . . 13
1.3 COMPLEX NUMBERS AND THE ARGAND PLANE . . . . . . . . . . . . . . 18
1.4 INTEGER AND FRACTIONAL POWERS OF A COMPLEX NUMBER . . . . 27
1.5 POINTS, SETS, LOCI, AND REGIONS IN THE COMPLEX PLANE . . . . . . 39
2 THE COMPLEX FUNCTION AND ITS DERIVATIVE 41
2.1 INTRODUCTION . . . . . . . . . . . . . . . . . . . . . . . . . . . . . . . . . . . 41
2.2 LIMITS AND CONTINUITY . . . . . . . . . . . . . . . . . . . . . . . . . . . . . 51
2.3 THE COMPLEX DERIVATIVE . . . . . . . . . . . . . . . . . . . . . . . . . . . 57
2.4 THE DERIVATIVE AND ANALYTICITY . . . . . . . . . . . . . . . . . . . . . 60
2.5 HARMONIC FUNCTIONS . . . . . . . . . . . . . . . . . . . . . . . . . . . . . . 67
3 THE BASIC TRANSCENDENTAL FUNCTIONS 74
3.1 THE EXPONENTIAL FUNCTION . . . . . . . . . . . . . . . . . . . . . . . . . 74
3.2 TRIGONOMETRIC FUNCTIONS . . . . . . . . . . . . . . . . . . . . . . . . . . 81
3.3 HYPERBOLIC FUNCTIONS . . . . . . . . . . . . . . . . . . . . . . . . . . . . . 88
3.4 THE LOGARITHMIC FUNCTION . . . . . . . . . . . . . . . . . . . . . . . . . 91
3.5 ANALYTICITY OF THE LOGARITHMIC FUNCTION . . . . . . . . . . . . . 96
3.6 COMPLEX EXPONENTIALS . . . . . . . . . . . . . . . . . . . . . . . . . . . . 100
3.7 INVERSE TRIGONOMETRIC AND HYPERBOLIC FUNCTIONS . . . . . . . 105
4 INTEGRATION IN THE COMPLEX PLANE 109
4.1 INTRODUCTION TO LINE INTEGRATION . . . . . . . . . . . . . . . . . . . 109
4.2 COMPLEX LINE INTEGRATION . . . . . . . . . . . . . . . . . . . . . . . . . . 111
4.3 CONTOUR INTEGRATION AND GREENS THEOREM . . . . . . . . . . . . 116
4.4 PATH INDEPENDENCE, INDEFINITE INTEGRALS, FUNDAMENTAL THE-
OREM OF CALCULUS IN THE COMPLEX PLANE . . . . . . . . . . . . . . . 120
4.5 THE CAUCHY INTEGRAL FORMULA AND ITS EXTENSION . . . . . . . . 124
4.6 SOME APPLICATIONS OF THE CAUCHY INTEGRAL FORMULA . . . . . . 127
5 INFINITE SERIES INVOLVING A COMPLEX VARIABLE 131
5.1 INTRODUCTION AND REVIEW OF REAL SERIES . . . . . . . . . . . . . . . 131
5.2 COMPLEX SEQUENCES AND CONVERGENCE OF COMPLEX SERIES . . 135
5.3 UNIFORM CONVERGENCE OF SERIES . . . . . . . . . . . . . . . . . . . . . 140
5
Contents
5.4 POWER SERIES AND TAYLOR SERIES . . . . . . . . . . . . . . . . . . . . . 145
5.5 TECHNIQUES FOR OBTAINING TAYLOR SERIES EXPANSIONS . . . . . . 155
5.6 LAURENT SERIES . . . . . . . . . . . . . . . . . . . . . . . . . . . . . . . . . . 163
6 RESIDUES AND THEIR USE IN INTEGRATION 169
6.1 INTRODUCTION AND DEFINITION OF THE RESIDUES . . . . . . . . . . . 169
6.2 ISOLATED SINGULARITIES . . . . . . . . . . . . . . . . . . . . . . . . . . . . 173
6.3 FINDING THE RESIDUE . . . . . . . . . . . . . . . . . . . . . . . . . . . . . . 177
6.4 EVALUATION OF REAL INTEGRALS WITH RESIDUE CALCULUS I . . . . 180
6.5 EVALUATION OF INTEGRALS II . . . . . . . . . . . . . . . . . . . . . . . . . 183
6.6 EVALUATION OF INTEGRALS III . . . . . . . . . . . . . . . . . . . . . . . . . 186
6
1 COMPLEX NUMBERS
1.1 INTRODUCTION
In this section, we show the exercise 10, 11, 20, 21, 22, 23, 29, 30, 32; and we note that E,
P mean the exercise, the page which are numbered respectively. We aslo call that 1, are
respectively the symbols of real part and imaginary part of the complex number.
Exercise 1.1. (E. 10 P. 7)
An infinite decimal such as e = 2.718281... is an irrational number since there is no repetitive
pattern in the successive digits. However, an infinite decimal such a 23.232323... is a rational
number. Because the digits do repeat in a cyclical manner, we can write this number as the
ration of two integers, as the following steps will show.
First we rewrite the number as 23 (1.010101...) = 23
_
1 + 10
2
+ 10
4
+ 10
6
+ ...
_
.
1. Recall from your knowledge of infinite geometric series that
1
1 r
= 1 + r + r
2
+ ..., where
r is a real number such that 1 < r < 1.
Sum the series
_
1 + 10
2
+ 10
4
+ 10
6
+ ...

.
2. Use the result of part (a) to show that 23.232323... equals
2300
99
. Verify this wit a divison
on a pocket calculator.
3. Using the same technique, express 376.376376... as a ration of integers.
Solution.
1. For the innite geometric series, we can choose r = 10
2
. Hence, we evaluate sum of the
following series.
1 + 10
2
+ 10
4
+ 10
6
+ ... =
1
1 10
2
=
100
99
2. In a similar fashion, we have
23.232323... = 23 (1.010101...)
= 23
_
1 + 10
2
+ 10
4
+ 10
6
+ ...
_
=
2300
99
7
1 COMPLEX NUMBERS
3. By choosing r = 10
3
, we have
_
1 + 10
3
+ 10
6
+ ...

=
1
1 10
3
=
1000
999
Thus, we apply the above expression for expressing 376.376376...
376.376376... = 376 (1.001001...)
= 376
_
1 + 10
3
+ 10
6
+ ...
_
= 376.
1000
999
=
376000
999
Exercise 1.2. (E. 11 P. 7)
Recall from your knowledge of innite geometric series that
1
1 r
= 1 +r +r
2
+..., where r is
a real number such that 1 < r < 1.
1. Using the method of this, express 3.04040404 . . . as the ratio of integers.
2. Using the above method show that .9999... is identical to 1. If you have any doubt about
this, try to nd a number between .9999... and 1.
Solution.
1. We observe that
3.04040404... = 3 (1.01010101...) + 0.01010101...
= 3
_
1 + 10
2
+ 10
4
+ 10
6
+ ...
_
+ 10
2
+ 10
4
+ 10
6
+ ...
We choose r = 10
2
for innite geometric series, so the above equality is equivalent to
3.
1
1 10
2
+
1
1 10
2
1 =
301
99
2. Using a similar fashion, we consider
8
1 COMPLEX NUMBERS
0.9999... = 9 (0.1111...)
= 9
_
10
1
+ 10
2
+ 10
3
+ 10
4
+ ...
_
= 9
_
1
1 10
1
1
_
= 1
Completing the proof by using r = 10
1
.
Exercise 1.3. (E. 20 P. 8)
In the following exercise, perform the operations and express the result in the form a+ib, where
a and b are real.
(x + iy) (u iv) (x iy) (u + iv)
where x, y, u, v are real.
Solution.
We use the commutative rule to simplify this expression.
(x + iy)(u iv)(x iy)(u + iv) = (x + iy)(x iy)(u iv)(u + iv)
=
_
x
2
ixy + iyx i
2
y
2
_ _
u
2
iuv + ivu i
2
v
2
_
= (x
2
+ y
2
)(u
2
+ v
2
)
Exercise 1.4. (E. 21 P. 8)
Review the following binomial theorem.
(a + b)
n
=
n

k=0
a
nk
b
k
n!
(n k)!k!
where n 0, a and b are complex numbers.
1. Use this theorem to nd a sum to represent (1 + iy)
n
, where n is a positive integer.
2. Use your above results to nd the real and imaginary parts of (1 + i2)
5
.
Solution.
1. For n is a positive integer, we can extend the expression (1 + iy)
n
.
(1 + iy)
n
=
n

k=0
(iy)
k
n!
k! (n k)!
2. By applying the above results, we have
9
1 COMPLEX NUMBERS
(1 + 2i)
5
=
n

k=0
(2i)
k
5!
k!(5 k)!
= 5!
_
1 +
2i
4!
+
4i
2
2!3!
+
8i
3
3!2!
+
16i
4
4!
+
32i
5
5!
_
= 1 + 10i 40 80i + 80 + 32i
= 41 38i
So that, real part is 41 and imaginary part is 38.
Exercise 1.5. (E. 22, E. 23 P. 9)
Let z
1
= x
1
+ iy
1
and z
2
= x
2
+ iy
2
be complex numbers, where the subscripted x and y are
real. Show that
1. 1(z
1
z
2
) = 1(z
1
) 1(z
2
) (z
1
) (z
2
)
2. (z
1
z
2
) = 1(z
1
) (z
2
) +(z
1
) 1(z
2
)
Solution.
1. Let z
1
= x
1
+ iy
1
and z
2
= x
2
+ iy
2
be complex numbers with 1(z
1
) = x
1
, 1(z
2
) =
x
2
, (z
1
) = y
1
and (z
2
) = y
2
respectively. We have
z
1
z
2
= x
1
x
2
y
1
y
2
+ i(x
1
y
2
+ x
2
y
1
)
Hence,
1(z
1
z
2
) = x
1
x
2
y
1
y
2
= 1(z
1
)1(z
2
) (z
1
)(z
2
)
The exercise is completely shown.
2. Let z
1
= x
1
+iy
1
and z
2
= x
2
+iy
2
be complex number again. We have 1(z
1
) = x
1
, (z
1
) =
y
1
, 1(z
2
) = x
2
, (z
2
) = y
2
. Then we review z
1
z
2
= x
1
x
2
y
1
y
2
+ i(x
1
y
2
+ x
2
y
1
). We will
obtain
(z
1
z
2
) = x
1
y
2
+ x
2
y
1
= 1(z
1
) (z
2
) +(z
1
) 1(z
2
)
, as desired.
10
1 COMPLEX NUMBERS
Exercise 1.6. (E. 29, E. 30, E. 32 P. 9)
For the following equations, x and y are real numbers. Solve for x and y. Begin by equating
the real parts on each side of the equation, and then the imaginary parts, thus obtaining two
real equations. Obtain all possible solutions.
1. e
x
2
+y
2
+ i2y = e
2xy
+ i
2. Log(x + y) + iy = 1 + ixy
3. cos x + i sin x = cosh(y 1) + ixy
Solution.
1. By equating the real part of the left side to the real part on the right, and similarly for
imaginary parts, we get a set of equations
_
_
_
e
x
2
+y
2
= e
2xy
2y = 1
This is equivalent to
_

_
_
x +
1
2
_
2
= 0
y =
1
2
So we have (x, y) =
_

1
2
;
1
2
_
as a unique root.
2. According to (1), we obtain a set of equations
_
_
_
Log (x + y) = 1
y = xy
In the second equation, we can derive x = 1 or y = 0.
By using rst equation, we have y = e1 for the case x = 1, and x = e for the case y = 0.
Hence, we investigate (x, y) = (1; e 1) ; (e; 0) as the needed roots.
3. As a similar method, we have
_
_
_
cos x = cosh (y 1)
sin x = xy
We remember a formula
cosh u =
e
u
+ e
u
2
By using AM-GM inequality, we obtain cosh u 1 for all u. And we also know that
cos v 1 for all v. Hence
cos x 1 cosh (y 1)
Thus that set of equations is equivalent to
11
1 COMPLEX NUMBERS
_

_
cos x = 1
cosh (y 1) = 1
sin x = 1
Then, we nd a root which is (0; 1).
12
1 COMPLEX NUMBERS
1.2 MORE PROPERTIES OF COMPLEX NUMBERS
We have the exercise 3, 6, 12, 14, 18, 19, 21, 22, 24 in this section.
Exercise 1.7. (E. 3, E. 6 P. 12)
We showed in this section material that z
1
+ z
2
= z
1
+z
2
, where z
1
= x
1
+iy
1
and z
2
= x
2
+iy
2
.
Follow a similar argument, to show the following.
1.
_
1
z
1
_
=
1
z
1
2. 1(z
1
z
2
) = 1(z
1
z
2
)
Solution.
1. Where z
1
= x
1
+ iy
1
is a complex number, we consider the following expression.
1
z
1
=
1
x
1
+ iy
1
=
x
1
iy
1
x
2
1
+ y
2
1
We derive the left side which is equivalent to
_
1
z
1
_
=
_
x
1
iy
1
x
2
1
+ y
2
1
_
=
x
1
iy
1
x
2
1
+ y
2
1
=
x
1
+ iy
1
x
2
1
+ y
2
1
=
1
z
1
Hence, we prove (1) completely.
2. In Exercise 1.5 (E. 22 P. 9), we already have 1(z
1
z
2
) = x
1
x
2
y
1
y
2
. Then, we also derive
z
1
z
2
= x
1
x
2
y
1
y
2
+ i (x
1
y
2
+ x
2
y
1
) by using z
1
= x
1
iy
1
and z
2
= x
2
iy
2
where
x
1
, x
2
, y
1
, y
2
are real numbers.
At this point, we can understand why the proof is completed as desired.
Exercise 1.8. (E. 12, E. 14 P. 12)
Compute the numerical values of the following expressions. Give the answers in the form a +ib
where a and b are real.
1. 2i +
3 4i
1 + 2i
2.
_
4 4i
2 + 2i
_
7
+
_
4 + 4i
2 2i
_
7
Solution.
13
1 COMPLEX NUMBERS
1. For evaluating this expression, we can simplify the fraction
3 4i
1 + 2i
. We have
3 4i
1 + 2i
=
(3 4i) (1 2i)
(1 + 2i) (1 2i)
=
3 6i 4i + 8i
2
1 4i
2
=
5 10i
5
At this point, it is easy to compute this expression. For concretness let us write
2i +
3 4i
1 + 2i
= 1
2. For calculating this expression, we can break it into smaller parts. In specic, we get

_
4 4i
2 + 2i
_
7
=
_
0 16i
8
_
7
= 2
7
i

_
4 + 4i
2 2i
_
7
=
_
0 + 16i
8
_
7
= 2
7
i
So the sum of expression equals 0.
Exercise 1.9. (E. 18, E. 19, E. 21 P. 13)
Let z
1
, z
2
and z
3
be three arbitrary complex numbers. Which of the following equations are
true in general?
1. i (z
1
+ z
2
+ z
3
) = i (z
1
+ z
2
+ z
3
)
2. 1(z
1
z
2
z
3
) = 1(z
1
z
2
z
3
)
3. 1(z
1
z
2
z
3
) = (iz
1
z
2
z
3
)
Solution.
1. The equation i(z
1
+ z
2
+ z
3
) = i(z
1
+ z
2
+ z
3
) is not true in general.
For concreteness let us choose z
1
= z
2
= 0;z
3
= i, we have
_
_
_
i(z
1
+ z
2
+ z
3
) = 1
i(z
1
+ z
2
+ z
3
) = 1
We can see i(z
1
+ z
2
+ z
3
) = i(z
1
+z
2
+z
3
) is true for any z
1
, z
2
, z
3
. If the readers have
any doubt about this, try to nd a distinct equation.
2. This equation is true in general. According to Exercise 1.7 (E. 6 P. 12), we have 1
_
k
1
k
2
_
=
1
_
k
1
k
2
_
, with k
1
, k
2
are two arbitrary complex numbers.
14
1 COMPLEX NUMBERS
Hence,
1(z
1
z
2
z
3
) = 1[z
1
(z
2
z
3
)]
= 1[z
1
(z
2
z
3
)]
= 1
_
z
1
(z
2
z
3
)
_
= 1[z
1
(z
2
z
3
)]
= 1(z
1
z
2
z
3
)
3. In general, we assert this which is true. In details, we know that 1(z
1
z
2
z
3
) = 1(z
1
z
2
z
3
)
by applying the above exercise. Then, we put z
1
z
2
z
3
= z
4
because they are also a complex
number. So this equation is equivalent to
1z
4
= (iz
4
)
At this point, we can use properties of complex numbers. For concretness let we write
_

_
1z
4
=
z
4
+ z
4
2
(iz
4
) =
iz
4
iz
4
2i
We note that iz
4
= z
4
, hence the event 1z
4
= (iz
4
) that is really true.
Exercise 1.10. (E. 22 P. 13)
Consider this problem : (3
2
+5
2
)(2
2
+7
2
) = (p
2
+q
2
). Our unknowns p and q are assumed to be
nonnegative integers. This problem has two sets of solution : p = 29, q = 31 and p = 41, q = 11,
as reader can verify. We derive here a general solution to problems of the following type : we
are given k, l, m, n which are nonnegative integers. We seek two sets of nonnegative integers p
and q such that (p
2
+q
2
) = (k
2
+l
2
)(m
2
+n
2
). It is not obvious that there are integers solutions,
but complex will prove that there are.
1. Note that by factoring we have (p+iq)(piq) = (k +il)(k il)(m+in)(min). Explain
why if (p + iq) = (k + il)(m + in) is satised, then (p iq) = (k il)(m in) is also.
With this hint, show that we can take p = [kmnl[ and q = lm+kn as solutions to our
problem.
2. Rearrange the equation for (p + iq)(p iq) given in (1) to show that we can also take
p = km + nl and q = [lmkn[ as a solution to our problem.
3. Using the results of parts (1) and (2), verify the values for p and q given at the start of
the problem. In addition, solve the following problem for two sets of values for p and q :
(p
2
+ q
2
) = (122)(53).
Solution.
1. If (p+iq) = (k+il)(m+in) is satised, then we have (p+iq)(piq) = (k+il)(m+in)(piq).
15
1 COMPLEX NUMBERS
For the case (k + il)(m + in) = 0, this means p + iq = 0 or p = q = 0. But we already
have p, q which are nonnegative numberss. So this case does not happen.
Hence, if we want (p + iq)(p iq) = (k + il)(k il)(m + in)(m in) to occur, then
(p iq) = (k il)(min).
Take p = [kmnl[ and q = lm + kn, we can see that p
2
+ q
2
=
_
k
2
+ l
2
_ _
m
2
+ n
2
_
. We
thus complete this proof.
2. At this time, we look at (p + iq) (p iq) = (k + il) (min) (m + in) (k il). So if
(p + iq) = (k + il) (min) is satised, then (p iq) = (m + in) (k il).
We can prove that if (p + iq) = (k + il) (min) is occur, we need take p = km + nl
and q = [lmkn[. Hence, we verify this values of p, q, we also see that p
2
+ q
2
=
_
k
2
+ l
2
_ _
m
2
+ n
2
_
.
We prove the exercise successfully.
3. Now we need to check with the solution : p
2
+ q
2
= (122) (53) to nd out whether these
above results are true. We rst divide 122 into sum of 11
2
and 1
2
, and break 53 into sum
of 7
2
and 2
2
. So we have p
2
+ q
2
=
_
11
2
+ 1
2
_ _
7
2
+ 2
2
_
.
Then we use the formula in part (1) with k = 11, l = 1, m = 7 and n = 2, they are already
nonnegative integers so let take p = [kmln[ = 75 and q = kn + lm = 29. For formula
in part (2), we have p = km + nl = 79 and q = [lmkn[ = [15[ = 15.
The readers can verify two p and q of each others.
Exercise 1.11. (E. 24 P. 13)
In Hamintons formulation two complex numbers (a, b) and (c, d) are said to be equal if and only
if a = c and b = d. These are necessary and sufficient conditions. In dealing with fractions we
are to making a comparable statement in asserting their equality. Consider
p
q
and
r
s
, where the
numerators and denominators are complex numbers. Find the necessary and sufficient conditons
for these fractions to be equal.
Solution.
With a
1
, a
2
, b
1
, b
2
are real numbers, we can build four complex numbers which is formed
_

_
p = a
1
+ ib
1
q = a
2
+ ib
2
r = c
1
+ id
1
s = c
2
+ id
2
But we must note that if we want to have the fractions
p
q
and
r
s
, then we need both denom-
inators are defined as the initial conditions. It means a
2
2
+ b
2
2
,= 0 and c
2
2
+ d
2
2
,= 0.
We consider two fractions and try to simplify these if we can.
16
1 COMPLEX NUMBERS
p
q
=
a
1
+ ib
1
a
2
+ ib
2
=
(a
1
+ ib
1
) (a
2
ib
2
)
(a
2
+ ib
2
) (a
2
ib
2
)
=
(a
1
a
2
+ b
1
b
2
) + i (a
2
b
1
a
1
b
2
)
a
2
2
+ b
2
2
In a similar fashion, we also reduce the following fraction.
r
s
=
c
1
+ id
1
c
2
+ id
2
=
(c
1
+ id
1
) (c
2
id
2
)
(c
2
+ id
2
) (c
2
id
2
)
=
(c
1
c
2
+ d
1
d
2
) + i (c
2
d
1
c
1
d
2
)
c
2
2
+ d
2
2
At this point, we can apply Hamiltons formulation for these fractions. In details,
p
q
and
r
s
are said to be equal if and only if
_

_
a
1
a
2
+ b
1
b
2
a
2
2
+ b
2
2
=
c
1
c
2
+ d
1
d
2
c
2
2
+ d
2
2
a
2
b
1
a
1
b
2
a
2
2
+ b
2
2
=
c
2
d
1
c
1
d
2
c
2
2
+ d
2
2
with the above initial conditions.
17
1 COMPLEX NUMBERS
1.3 COMPLEX NUMBERS AND THE ARGAND PLANE
We present the exercise 7, 8, 10, 11, 12, 13, 17, 35, 38, 39, 41, 43, 44, 45. And in this section,
we have a convention cis which is equivalent to cos + i sin .
Exercise 1.12. (E. 7, E. 8 P. 26)
Find the modulus of each of the following complex expressions :
1.
(1 + i)
5
(2 + 3i)
5
2.
(1 i)
n
(2 + 2i)
n
, n > 0 is an integer.
Solution.
1. We have

(1 + i)
5
(2 + 3i)
5

=
[1 + i[
5
[2 + 3i[
5
We evaluate the modulus of numerator and denominator.
[1 + i[
5
=
_
2
_
5
[2 + 3i[
5
=
_
13
_
5
So the consistent result is
4
169
_
2
13
.
2. We have to compute the denominator and the numerator respectively after having the
dierent form of expression.

(1 i)
n
(2 + 2i)
n

=
[1 i[
n
[2 + 2i[
n
Analyzing [2 + 2i[
n
= 8
n
2
and [1 i[
n
= 2
n
2
, we have the following fraction
[1 i[
n
[2 + 2i[
n
=
2
n
2
8
n
2
=
1
2
n
, n > 0 is an integer.
So this is the result of the exercise.
Exercise 1.13. (E. 10 P. 26)
Find two complex numbers that are conjugates of each other. The magnitude of their sum is 1
and the sum of their magnitudes is 2.
Solution.
We derive that z
1
and z
2
are two complex numbers that are conjugates of each other, with
z
1
= a + ib, z
2
= a bi, where a, b are real numbers.
18
1 COMPLEX NUMBERS
Therefore, we get a set of equations.
_
_
_
[z
1
+ z
2
[ = 1
[z
1
[ +[z
2
[ = 2
Because of [z
1
+ z
2
[ =
_
(a + a)
2
+ (b b)
2
and [z
1
[ + [z
2
[ =

a
2
+ b
2
+

a
2
+ b
2
, a set of
equations is equivalent to
_
_
_

2a
2
= 1

a
2
+ b
2
= 1
Thus, we have 4 pairs complex numbers.
(a, b) =
__
1

2
;
1

2
_
,
_

2
;
1

2
_
,
_

2
;
1

2
_
,
_
1

2
;
1

2
__
So we can choose
_

_
z
1
=
1

2
+ i
1

2
z
2
=
1

2
i
1

2
as the answer.
Exercise 1.14. (E. 11 P. 26)
Find two complex numbers that are conjugates to each other. The magnitude of their sum is a
and the sum of their magnitude is
1
2
, where 0 < a < 1. Answer in terms of a.
Solution.
We set z
1
= x
1
+iy
1
, z
2
= x
1
iy
1
where x
1
, y
1
are real numbers, because z
1
, z
2
are conjugates
to each other as the exercise says.
And we have these complex numbers satised by two initial conditions, namely
_
_
_
[z
1
+ z
2
[ = a
[z
1
[ +[z
2
[ =
1
a
, where 0 < a < 1.
We also have [z
1
+ z
2
[ = 2 [x
1
[ and [z
1
[ +[z
2
[ = 2
_
x
2
1
+ y
2
1
. Hence, the above set of equation
is equivalent to
_
_
_
2 [x
1
[ = a
2
_
x
2
1
+ y
2
1
=
1
a
At this point, we can solve this set of equations to derive the roots by using a < 1. And we
see that the exercise only suggests nding two complex numbers, so we can choose
_

_
x
1
=
a
2
y
1
=
1
2
_
1
a
2
a
2
19
1 COMPLEX NUMBERS
as a result.
Hence,
_

_
z
1
=
a
2
+
1
2
i
_
1
a
2
a
2
z
2
=
a
2

1
2
i
_
1
a
2
a
2
Exercise 1.15. (E. 12 P. 26)
Find two complex numbers whose product is 2 and whose difference is i.
Solution.
In a similar way of Exercise 1.14 (E. 11 P. 26), we build z
1
= x
1
+ iy
1
, z
2
= x
1
iy
1
where
x
1
, y
1
are real numbers.
The initial conditions is known as two informations of this exercise : z
1
z
2
= 2 and z
1
z
2
= i
because we can take z
1
z
2
> 0. In details, we also write these to become a set of equations
with x
1
, y
1
variables.
_
_
_
x
2
1
+ y
2
1
= 2
2iy
1
= i
At this point, we can solve this set of equations easily. Thus we have the following result.
_

_
z
1
=

7
2
+
1
2
i
z
2
=

7
2

1
2
i
Exercise 1.16. (E. 13, E. 17 P. 26)
The following vectors represent complex numbers. State these numbers in the form a + ib.
1. The vector beginning at (1, 3) and terminating at (1, 4).
2. The vector of length
3
2
beginning at the origin and ending, in the rst quadrant, on the
circle (x 1)
2
+ y
2
= 1.
Solution.
1. We show the vector in Figure 1 which is called

a .
20
1 COMPLEX NUMBERS
Figure 1.
Hence, we have

a = (1; 4) (1; 3) = (2; 7). Then the consistent form is 2 + 7i.
2. We see that the vector of length is known as its magnitude, so we have
_
x
2
+ y
2
=
3
2
.
Then, we can create a set of equations.
_
_
_
_
x
2
+ y
2
=
3
2
(x 1)
2
+ y
2
= 1
By solving this set and noticing the rst quadrant of circle, we obtain the consistent form
is z =
9
8
+ i
3

7
8
.
We can see it in Figure 2.
Figure 2.
21
1 COMPLEX NUMBERS
Exercise 1.17. (E. 35 P. 27)
Reduce the following expression to the form rcis, giving only the principal value of the angle.
A =
(1 i) cis
_

4
_
_
3 + i
_
2
Solution.
We consider the numerator and denominator of expression A, and we can present these into
trigonometric forms.
_
_
_
1 i =

2cis
_

4
_
_
3 + i
_
2
= 2cis
_

6
_
Thus, expression A is equivalent to
A =

2cis
_

4
_
cis
_

4
_
2cis
_

6
_
=

2
2
cis
_

2
_
cis
_

6
_
=

2
2
cis
_

3
_
Exercise 1.18. (E. 38 P. 27)
1. Let z
1
and z
2
be complex numbers. By replacing z
2
with z
2
in the inequality [z
1
+ z
2
[
[z
1
[ +[z
2
[, show that
[z
1
z
2
[ [z
1
[ +[z
2
[
Interpret this result with the aid of triangle.
2. What must be the relationship between z
1
and z
2
in order to have the equality hold in
part (1) ?
Solution.
1. Using inequality [z
1
+ z
2
[ [z
1
[ + [z
2
[ and replacing z
2
with z
2
in this, we have z
2
which is also a complex number. Hence,
[z
1
z
2
[ = [z
1
+ (z
2
)[ [z
1
[ +[z
2
[
The readers can verify [z
2
[ = [z
2
[ easily, so that we obtain the completed proof.
This inequality can be explain explicitly in Figure 3.
22
1 COMPLEX NUMBERS
Figure 3.
2. The equality hold in part (1) if only if z
1
= z
2
, namely
_
_
_
cos (arg z
1
) = cos (arg z
2
)
sin (arg z
1
) = sin (arg z
2
)
For the rst equation, we can deduce that arg z
1
= arg z
2
+ k2 or arg z
1
= arg z
2

+ k2, for all k is an integer. In a similar way, for the second equation, we derive that
arg z
1
= arg z
2
+ k2 or arg z
1
= + arg z
2
+ k2, for all k is an integer.
Finally, we have arg z
1
= + arg z
2
+ k2 for all k is an integer.
Exercise 1.19. (E. 39 P. 27)
Let z
1
and z
2
be complex numbers. Show that
[z
1
+ z
2
[ [z
1
[ [z
2
[ 0 if [z
1
[ [z
2
[
[z
1
+ z
2
[ [z
2
[ [z
1
[ 0 if [z
2
[ [z
1
[
Explain why both formulas can be reduced to the single expression
[z
1
+ z
2
[ [[z
1
[ [z
2
[[
Solution.
In a similar way of Exercise 1.18 (E. 38 P. 27), we still use inequality [z
1
+ z
2
[ [z
1
[ + [z
2
[
and try to get some distinct patterns to prove the inequalities of this exercise. We look at the
following technique.
[z
1
[ = [z
1
+ z
2
+ (z
2
)[
At this point, we have [z
1
+ z
2
+ (z
2
)[ [z
1
+ z
2
[ +[z
2
[. It is easy to see that [z
2
[ = [z
2
[.
Thus we prove [z
1
+ z
2
[ [z
1
[ [z
2
[ for all z
1
, z
2
are complex numbers, so if [z
1
[ [z
2
[, we can
deduce an equality [z
1
+ z
2
[ [z
1
[ [z
2
[ 0.
23
1 COMPLEX NUMBERS
Hence, we also show the inequality [z
1
+ z
2
[ [z
2
[ [z
1
[ for all z
1
, z
2
are complex numbers
by using the same technique.
[z
2
[ = [z
2
+ z
1
+ (z
1
)[ [z
1
+ z
2
[ +[z
1
[
So we can derive an equality [z
1
+ z
2
[ [z
2
[ [z
1
[ 0 if [z
2
[ [z
1
[.
Because an inequality [z
1
+ z
2
[ [[z
1
[ [z
2
[[ can divide into two cases. These cases are really
two inequalities which is shown in this exercise. Hence, the proof is completed.
Exercise 1.20. (E. 41 P. 28)
1. By considering the expression (p q)
2
, where p and q are nonnegative real numbers, show
that p + q

2
_
p
2
+ q
2
.
2. Use the preceding result to show that for any complex number z we have [1z[ + [z[

2 [z[.
Solution.
1. We consider the expression (p q)
2
, where p and q are nonnegative real numbers. We
have this expression which interpolates an inequality
(p + q)
2
2
_
p
2
+ q
2
_
Thus we can complete this proof because of square roots.
2. For any complex number z, we put z = x+iy, where x and y are real numbers. The proof
is equivalent to
[x[ +[y[

2 [x + iy[
We see that [x + iy[ =
_
x
2
+ y
2
, so this proof becomes part (1) which is shown.
Exercise 1.21. (E. 43 P. 28)
1. By considering the product of 1 + ia and 1 + ib, and the argument of each factor, where
a and b are real numbers. Show that
arctan(a) + arctan(b) = arctan
_
a + b
1 ab
_
2. Use the preceding formula to prove that
= 4
_
arctan
_
1
2
_
+ arctan
_
1
3
__
3. Extend the technique used in (1) to nd a formula for arctan(a) + arctan(b) + arctan(c).
Solution.
1. Let z
1
be 1 + ia and z
2
be 1 + ib, where a, b are real numbers. We get
z
1
z
2
= 1 ab + i(a + b)
24
1 COMPLEX NUMBERS
By using these knowledges : arg z
1
+ arg z
2
= arg (z
1
z
2
), arg z
1
= arctan a, arg z
2
=
arctan b and arg (z
1
z
2
) = arctan
_
a + b
1 ab
_
, we can derive
arctan a + arctan b = arctan
_
a + b
1 ab
_
, as desired.
2. Applying the above result, we take a =
1
2
and b =
1
3
. Hence,
arctan
1
2
+ arctan
1
3
= arctan (1)
=

4
So we prove (2) completely.
3. By the same the technique used in (1), we can nd that
arctan a + arctan b + arctan c = arctan
a + b + c abc
1 ab bc ca
where a, b, c are real numbers.
If the readers have any doubt about the above result, try to prove it in a similar interpo-
lation.
Exercise 1.22. (E. 44 P. 28)
1. Consider the inequality [z
1
+ z
2
[
2
[z
1
[
2
+ [z
2
[
2
+ 2 [z
1
[ [z
2
[. Prove this expression by
algebraic means (no triangles).
2. Observe that [z
1
[
2
+[z
2
[
2
+ 2 [z
1
[ [z
2
[ = ([z
1
[ +[z
2
[)
2
. Show that the inequality proved in
part (1) leads to the triangle inequality [z
1
+ z
2
[ [z
1
[ +[z
2
[.
Solution.
1. By using the formula of modulus of one complex number, we have
[z
1
+ z
2
[
2
= (z
1
+ z
2
) (z
1
+ z
2
)
= (z
1
+ z
2
) (z
1
+ z
2
)
= [z
1
[
2
+[z
2
[
2
+ z
1
z
2
+ z
1
z
2
Then we notice that z
1
z
2
+ z
1
z
2
= 21(z
1
z
2
) 2 [z
1
z
2
[. Hence, the above expression is
less than or equal to [z
1
[
2
+[z
2
[
2
+ 2 [z
1
z
2
[. Next, we also notice that [z
1
z
2
[ is equivalent
to [z
1
[ [z
2
[.
At this point, we have the successful proof.
25
1 COMPLEX NUMBERS
2. According to part (1), we get
[z
1
+ z
2
[
2
[z
1
[
2
+[z
2
[
2
+ 2 [z
1
[ [z
2
[
, for all z
1
, z
2
are complex numbers.
By using the initial condition that is
[z
1
[
2
+[z
2
[
2
+ 2 [z
1
[ [z
2
[ = ([z
1
[ +[z
2
[)
2
, we derive that
[z
1
+ z
2
[
2
([z
1
[ +[z
2
[)
2
And this inequality is equivalent to
[z
1
+ z
2
[ [z
1
[ +[z
2
[
because the modulus of one complex number is already nonnegative real number. Hence,
the proof is completed.
Exercise 1.23. (E. 45 P. 28)
Beginning with the product (z
1
z
2
) (z
1
z
2
), show that
[z
1
z
2
[
2
= [z
1
[
2
+[z
2
[
2
21(z
1
z
2
)
Solution.
Beginning with the product (z
1
z
2
) (z
1
z
2
), for all z
1
= x
1
+iy
1
and z
2
= x
2
+iy
2
, where
x
1
, x
2
, y
1
, y
2
are real numbers. We have
[z
1
z
2
[
2
= (z
1
z
2
) (z
1
z
2
)
= [x
1
x
2
+ i (y
1
y
2
)] [x
1
x
2
+ i (y
1
+ y
2
)]
= x
2
1
+ x
2
2
2x
1
x
2
+ y
2
1
+ y
2
2
2y
1
y
2
= [z
1
[
2
+[z
2
[
2
2 (x
1
x
2
+ y
1
y
2
)
At this point, we get z
1
z
2
= x
1
x
2
+y
1
y
2
+i (x
1
y
2
+ y
1
x
2
), so 1(z
1
z
2
) is exactly x
1
x
2
+y
1
y
2
.
Therefore, we obtain what we need to prove.
[z
1
z
2
[
2
= [z
1
[
2
+[z
2
[
2
21(z
1
z
2
)
26
1 COMPLEX NUMBERS
1.4 INTEGER AND FRACTIONAL POWERS OF A COMPLEX
NUMBER
We choose the exercise 7, 8, 9, 20, 27, 28, 29, 34, 35 for illustrational section.
Exercise 1.24. (E. 7 P. 35)
1. Using DeMoivres theorem, the binomial formula, and an obvious trigonometric identity,
show that for integer n,
cos n = 1
n

k=0
_
cos
nk

_
_

1 cos
2

_
k
i
k
n!
(n k)!k!
2. Show that the preceding expression can be written as
cos n =
n/2

m=0
(cos )
n2m
_
1 cos
2

_
m
(1)
m
n!
(n 2m)! (2m)!
, if n is even,
cos n =
(n1)/2

m=0
(cos )
n2m
_
1 cos
2

_
m
(1)
m
n!
(n 2m)! (2m)!
, if n is odd.
Solution.
1. We can change the right side for using the binomial formula.
1
n

k=0
_
cos
nk

__
_
1 cos
2

_
k
i
k
n!
(n k)!k!
= 1
n

k=0
(cos )
nk
(i [sin [)
k
n!
(n k)!k!
= 1(cos + i [sin [)
n
For the case sin 0, the above expression is equivalent to
1(cos + i sin )
n
= 1(cos n + i sin n) = cos n
For the case sin < 0, we obtain
1(cos i sin )
n
= 1[cos () + i sin ()]
n
= cos (n)
= cos n
The proof is completed by using DeMoivres theorem.
2. If n is even, we put n = 2p, where p is integer. Hence that rst proof is equivalent to
27
1 COMPLEX NUMBERS
cos 2p =
p

m=0
(cos )
2p2m
_
1 cos
2

_
m
(1)
m
(2p)!
(2p 2m)! (2m)!
=
p

m=0
(cos )
2p2m
_
_
1 cos
2

_
2m
(i)
2m
(2p)!
(2p 2m)! (2m)!
= 1
2p

k=0
_
cos
2pk

__
_
1 cos
2

_
k
i
k
(2p)!
(2p k)!k!
This is proved in part (1), so the rst proof is completely true.
In a similar fashion, if n is odd, we put n = 2q + 1, where q is integer. Thus the second
proof is equivalent to
cos (2q + 1) =
q

m=0
(cos )
2q+12m
_
1 cos
2

_
m
(1)
m
(2q + 1)!
(2q + 1 2m)! (2m)!
=
q

m=0
(cos )
2q+12m
_
_
1 cos
2

_
2m
(i)
2m
(2q + 1)!
(2q + 1 2m)! (2m)!
= 1
2q+1

k=0
_
cos
2q+1k

__
_
1 cos
2

_
k
i
k
(2q + 1)!
(2q + 1 k)!k!
This is shown in part (1), hence the second proof is exact.
Exercise 1.25. (E. 8 P. 36)
Prove that
_
1 + i tan
1 i tan
_
n
=
1 + i tan n
1 i tan n
where n is any integer.
Solution.
For n is an arbitrary integer and an angle of that tan is dened. We have
_
1 + i tan
1 i tan
_
n
=
(cos + i sin )
n
(cos i sin )
n
=
cos n + i sin n
cos n i sin n
=
1 + i tan n
1 i tan n
Completing the proof by using DeMoivres theorem.
28
1 COMPLEX NUMBERS
Exercise 1.26. (E. 9, E. 20 P.36)
Express the following in the form a +ib. Give all values and make a polar plot of the points or
the vectors that represent your results.
1. (9i)
1
2
2. (1 + i)
5
4
Solution.
1. We put z = i, then we can derive the trigonometric form of z. This means z = cis
_

2
_
easily. Applying the formula z
1
m
=
m

rcis
_

m
+
2k
m
_
, k = 0, m1, with z = rcis (),
we have
(9i)
1
2
= 3
_
cos
_

2
_
+ i sin
_

2
__1
2
= 3
_
cos
_

4
+ k
_
+ i sin
_

4
+ k
__
, k = 0, 1.
Expressed as decimals, these answers become approximately
2.121 +i2.121 , k = 0
2.121 i2.121 , k = 1
Vectors representing the roots are plotted in Figure 4. We can see that they are spaced
or 180

apart.
Figure 4.
29
1 COMPLEX NUMBERS
2. We put z = 1 + i and rst compute r =

2 and argument of z which we call , equals



4
by evaluating tan = 1.
Therefore, we have a following trigonometric form of z.
z =

2
_
cos

4
+ i sin

4
_
And then, our result is
z
5
4
=
_

2
_5
4
_
cos
_

5
16

5k
2
_
+ i sin
_

5
16

5k
2
__
=
_

2
_5
4
_
cos
_

5
16

k
2
_
+ i sin
_

5
16

k
2
__
, k = 0, 1, 2, 3.
Expressed as decimals, these answers become approximately
0.360 i0.539 , k = 0
0.539 i0.360 , k = 1
0.360 + i0.539 , k = 2
0.539 + i0.360 , k = 3
Vectors representing the roots are plotted in Figure 5. We can see that they are spaced

2
or 90

apart.
Figure 5.
30
1 COMPLEX NUMBERS
Exercise 1.27. (E. 27 P. 36)
1. Show that z
n+1
1 = (z 1)
_
z
n
+ z
n1
+ ... + z + 1
_
, where n 0 is an integer annd z
is any complex number.
The preceding implies that
z
n+1
1
z 1
= z
n
+ z
n1
+ ... + z+1 for z ,= 1
which the reader should recognize as the sum of a geometric series.
2. Use the preceding result to find and plot all solutions of z
4
+ z
3
+ z
2
+ z + 1 = 0.
Solution.
1. We prove it by mathematical induction :
For the case n = 0, the expression says
z 1 = (z 1) 1
This assertion is really true. Given the articiality of these assumptions, we may be
happier if the base case for n = 1 is also given.
For the case n = 1, the expression says
z
2
1 = (z 1) (z + 1)
It is easy to see that the above assertion is true. Thus we have the base cases for our
induction.
For the induction step we assume that
z
k+1
1 = (z 1)
_
z
k
+ z
k1
+ ... + z + 1
_
and show that
z
k+2
1 = (z 1)
_
z
k+1
+ z
k
+ ... + z + 1
_
We consider the left side z
k+2
1 which can be analyse the sum of (z 1) z
k+1
and
z
k+1
1. Hence,
(z 1) z
k+1
+
_
z
k+1
1
_
= (z 1) z
k+1
+ (z 1)
_
z
k
+ z
k1
+ ... + z + 1
_
= (z 1)
_
z
k+1
+ z
k
+ z
k1
+ ... + z + 1
_
At this point, we can have completely the proof.
2. By using the result of part (1), we can solve this equation. Moreover, we might present
these roots by plotting of trigonometric forms. This equation is equivalent to
z
5
1
z 1
= 0
31
1 COMPLEX NUMBERS
Hence, we derive the roots because of z ,= 1.
z
5
= cos

2
+ i sin

2
Applying the DeMoivres theorem, we thus have
z = cos
_

10
+
k2
5
_
+ i sin
_

10
+
k2
5
_
, k = 0, 1, 2, 3, 4
Expressed as decimals, these answers become approximately
z
0
= cos

10
+ i sin

10
= 0.951 + i0.309
z
1
= cos
_

10
+
2
5
_
+ i sin
_

10
+
2
5
_
= i
z
2
= cos
_

10
+
4
5
_
+ i sin
_

10
+
4
5
_
= 0.951 + i0.309
z
3
= cos
_

10
+
6
5
_
+ i sin
_

10
+
6
5
_
= 0.587 i0.809
z
4
= cos
_

10
+
8
5
_
+ i sin
_

10
+
8
5
_
= 0.587 i0.809
Then, according to the suggestion, we will plot all the above roots. The readers can see
it in Figure 6.
Figure 6.
32
1 COMPLEX NUMBERS
Exercise 1.28. (E. 28 P. 37)
1. If z = rcis, show that the sum of the values of z
1
n
is given by
n1

k=0
n

r
_
cis

n
__
cis
_
2
n
__
k
where n 2 is an integer.
2. Show that the sum of the values of z
1
n
is zero. Do this by rewriting the preceding problem
in Exercise 1.26 (E. 27 P. 36), but with n1 used in place of n, and employing the formula
of part (1).
Solution.
1. If z = rcis, we use the formula of z
1
n
, where n 2 is an integer.
z
1
n
=
n

r
_
cos
_

n
+
2k
n
_
+ i sin
_

n
+
2k
n
__
, k N, k = 0, n 1.
For the case k = 0, this formula says
z
1
n
=
n

r
_
cos

n
+ i sin

n
_
=
n

rcis
_

n
_
For the case k = 1, this formula says
z
1
n
=
n

r
_
cos
_

n
+
2
n
_
+ i sin
_

n
+
2
n
__
=
n

rcis
_

n
_
cis
_
2
n
_
For the case k = 2, this formula also says
z
1
n
=
n

r
_
cos
_

n
+
4
n
_
+ i sin
_

n
+
4
n
__
=
n

rcis
_

n
_
cis
_
4
n
_
In a similar fashion, for the case k = n 1, we obtain
z
1
n
=
n

rcis
_

n
_
cis
_
2 (n 1)
n
_
33
1 COMPLEX NUMBERS
Hence, the sum of the values of z
1
n
, which is called S, is created by the following expression.
S =
n

rcis
_

n
__
1 + cis
_
2
n
_
+ cis
_
4
n
_
+ ... + cis
_
2 (n 1)
n
__
=
n1

k=0
n

r
_
cis

n
__
cis
_
2
n
__
k
, as desired.
2. We get the following formula
z
n
1
z 1
=
n1

k=0
z
k
, where n 0 is an integer and z is any complex number.
Thus we can choose z = cis
_
2
n
_
, where n 2 is an integer. The above expression
becomes
_
cis
_
2
n
__
n
1
cis
_
2
n
_
1
=
n1

k=0
_
cis
_
2
n
__
k
Because of
_
cis
_
2
n
__
n
1 = cis2 1 = 0, we can completely prove that the sum of
the values of z
1
n
is zero.
Exercise 1.29. (E. 29 P. 37)
1. Suppose a complex number is given in the form z = rcis. Recalling the indentities
sin
_

2
_
=

1
2

_
1
2
_
cos and cos
_

2
_
=

1
2
+
_
1
2
_
cos , show that
z
1
2
=

r
_
_
1 + cos
2
+ i
_
1 cos
2
_
for 0 .
2. Explain why the preceding formula is invalid for < < 0, and nd the corresponding
correct formula for this interval.
3. Use the formulas derived in (1) and (2) to nd the square roots of 2cis
_

6
_
and 2cis
_

6
_
,
respectively.
Solution.
1. Using the form z = rcis, we have
34
1 COMPLEX NUMBERS
z
1
2
=

r
_
cos
_

2
+ k
_
+ i sin
_

2
+ k
__
, k = 0, 1. For 0 , we see that if we take k = 0, the above expression is equivalent
to
z
1
2
=

r
_
cos
_

2
_
+ i sin
_

2
__
=

r
_
_
1 + cos
2
+ i
_
1 cos
2
_
And for the case k = 1, we get
z
1
2
=

r
_
cos
_

2
+
_
+ i sin
_

2
+
__
=

r
_
_
1 + cos
2
+ i
_
1 cos
2
_
Then we prove part (1) completely.
2. For < < 0 because of principal argument. We observe that cos
_

2
_
and sin
_

2
_
are negative numbers, we obtain
z
1
2
=

r
_
_
1 + cos
2
i
_
1 cos
2
_
So this formula is not the above formula of part (1). Hence we nd out the corresponding
correct formula for this interval.
3. Applying the formula of part (1), the square roots of 2cis
_

6
_
equals

2
_
_
_
_

_
1 + cos

6
2
+ i

_
1 cos

6
2
_
_
_
_
=
__

3 + 1
_
+ i
_

3 1
__
And using the formula of part (2), the square roots of 2cis
_

6
_
is
__
3 1
_
i
_
3 + 1
_
.
35
1 COMPLEX NUMBERS
Exercise 1.30. (E. 34 P. 38)
Use the formula for the sum of a geometric series in Exercise 27 and DeMoivres theorem to
derive the following formulas for 0 < < 2 :
1 + cos + cos 2 + + cos n =
cos
_
n
2
_
sin
_
(n + 1)
2
_
sin
_

2
_
sin + sin 2 + sin 3 + + sin n =
sin
_
n
2
_
sin
_
(n + 1)
2
_
sin
_

2
_
Solution.
By using DeMoivres theorem, we can have
n

k=0
cos k + i
_
n

k=0
sin k
_
= 1 + (cis) + (cis2)
2
+ + (cisn)
n
According to the formula z
n+1
1 = (z 1)
_
z
n
+ z
n1
+ + z + 1
_
, where n is an nonneg-
ative integer and z is any complex number, we have
36
1 COMPLEX NUMBERS
1 + (cis) + (cis2)
2
+ + (cisn)
n
=
(cis)
n+1
1
cis 1
=
cos [(n + 1) ] + i sin [(n + 1) ] 1
cos + i sin 1
=
(cos 1 i sin ) cos [(n + 1) ] 1 + i sin [(n + 1) ]
(cos 1 + i sin ) (cos 1 i sin )
=
(cos 1) cos [(n + 1) ] 1 + sin sin [(n + 1) ]
2 (1 cos )
+i
(cos 1) sin [(n + 1) ] sin cos [(n + 1) ] 1
2 (1 cos )
=
cos [n] cos cos [(n + 1) ] + 1
4 sin
2

2
+i
sin (n) sin [(n + 1) ] + sin
4 sin
2

2
=
2 sin
__
2n + 1
2
_

_
sin

2
+ 2 sin
2

2
4 sin
2

2
+i
2 sin

2
cos

2
2 cos
__
2n + 1
2
_

_
sin

2
4 sin
2

2
=
cos
_
n
2

_
sin
__
n + 1
2
_

_
sin

2
+ i
sin
n
2
sin
__
n + 1
2
_

_
sin

2
Using the homogeneous method, we can deduce that
1 + cos + cos 2 + + cos n =
cos
_
n
2

_
sin
__
n + 1
2
_

_
sin

2
sin + sin 2 + sin 3 + + sin n =
sin
n
2
sin
__
n + 1
2
_

_
sin

2
, as desired.
37
1 COMPLEX NUMBERS
Exercise 1.31. (E. 35 P. 38)
If n is an integer greater than or equal to 2, prove that
cos
_
2
n
_
+ cos
_
4
n
_
+ ... + cos
_
2 (n 1)
n
_
= 1
and that
sin
_
2
n
_
+ sin
_
4
n
_
+ ... + sin
_
2 (n 1)
n
_
= 0
Solution.
For n is an integer greater than or equal to 2, we have this sum in Exercise 1.28 (E. 28 P. 37)
n1

k=0
n

r
_
cis

n
__
cis
_
2
n
__
k
=
n1

k=0
z
1
n
_
cis
_
2
n
__
k
By choosing z = 1, the above formula becomes
n1

k=0
_
cis
_
2
n
__
k
.
And then, using general DeMoivres theorem, we get
n1

k=0
_
cis
_
2
n
__
k
=
n1

k=0
cis
_
2k
n
_
= cos 0 + cos
_
2
n
_
+ cos
_
4
n
_
+ ... + cos
_
2 (n 1)
n
_
+i
_
sin 0 + sin
_
2
n
_
+ sin
_
4
n
_
+ ... + sin
_
2 (n 1)
n
__
Using again the result of Exercise 1.28 (E. 28 P. 37), this says that the above sum is zero,
namely
_

_
cos 0 + cos
_
2
n
_
+ cos
_
4
n
_
+ ... + cos
_
2 (n 1)
n
_
= 0
sin 0 + sin
_
2
n
_
+ sin
_
4
n
_
+ ... + sin
_
2 (n 1)
n
_
= 0
We note that cos 0 = 1 and sin 0 = 0, we thus have the proof successfully.
38
1 COMPLEX NUMBERS
1.5 POINTS, SETS, LOCI, AND REGIONS IN THE COMPLEX
PLANE
We call the exercise 12, 31 as the presentation.
Exercise 1.32. (E. 12 P. 46)
Find the points on the circle [z 1 i[ = 1 that have the nearest and furthest linear distance
to the point z = 1 + i0. In addition, state what these two distances are.
Solution.
We put z = x + iy, where x, y are real numbers. Since [z 1 i[ = 1, we have
1 = [x + iy 1 i[
1 = [(x 1) + i (y 1)[
1 = (x 1)
2
+ (y 1)
2
We then consider the above equation, we have center of there circle which is called I and has
the coordinate I (1, 1). And the modulus of radius equals 1.
Put A(1, 0), we obtain that the equation of line AB is y =
1
2
x +
1
2
. Thus, by replacing this
into expression (x 1)
2
+ (y 1)
2
= 1, we get that the left side is equivalent to
(x 1)
2
+ (y 1)
2
= (x 1)
2
+
_
1
2
x +
1
2
1
_
2
= (x 1)
2
+
1
4
(x 1)
2
=
5
4
(x 1)
2
Then, we can deduce x = 1 +
2

5
or x = 1
2

5
. And we call K
1
_
1 +
2

5
, 1 +
1

5
_
;
K
2
_
1
2

5
, 1
1

5
_
are the points which is recently found.
It is easy too see that
AK
1
=

_
2 + 2

5
_
2
+
_
1 +
1

5
_
2
> AK
2
=

_
2 2

5
_
2
+
_
1 +
1

5
_
2
So we have the conclusions :
K
2
is the point on the circle [z 1 i[ = 1 that have the nearest linear distance to the point
z = 1 + 0i,
K
1
is the point on the circle [z 1 i[ = 1 that have the furthest linear distance to the point
z = 1 + 0i.
Exercise 1.33. (E. 31 P. 47)
Is a boundary point of a set necessarily an accumulation point of that set?
39
1 COMPLEX NUMBERS
Solution.
We display 2 denitions of a boundary point and an accumulation point.
u is a boundary point of set D if it is a limit point of D and ED.
u is an accumulation of D if for all r > 0 : (B (u; r) u) D ,= .
In Z space, given A = [0; 1], we can see that 0 is a boundary point of this set but is not an
accumulation point.
40
2 THE COMPLEX FUNCTION AND ITS
DERIVATIVE
2.1 INTRODUCTION
For the introduction, we have some exercises which are 2, 4, 6, 8, 10, 13, 15, 17, 19, 21, 24, 27.
Exercise 2.1. (E. 2, E. 4 P. 53)
Suppose z = x + iy. Let f (z) =
(z i) (z 2)
(z
2
+ 1) cos x
. State where in the following domains this
functions fails to be dened.
1. [z[ < 1.1
2.

z (1 + i)

2

<

2
Solution.
1. It is easy to observe that f (z) is dened when its denominator is not equal to 0. It means
z ,= i and z ,=

2
+ k + iy for k is an integer. So that z = i, z =

2
+ k + iy make
this function fail to be dened basically. Then we see that z = i belong in inequality
[z[ < 1.1 but z =

2
+ k + iy are not here for all k is an integer.
Hence, this function is not dened at two points z = i.
2. We simplify the modulus of z (1 + i)

2
which equals
_
_
x

2
_
2
+
_
y

2
_
2
. Thus we
have the coordinate of origin of this circle that is
_

2
;

2
_
and the radius of domain is aslo

2
. We have both points z = i which are not in inequality

z (1 + i)

2

<

2
. For the
only case k = 0, we have a point
_

2
; y
_
that belongs in this inequality. Then we can
deduce y satisfying 0 < y < .
Hence, f (z) is not dened on line x =

2
, 0 < y < .
Exercise 2.2. (E. 6, E. 8 P. 53)
For each of the following functions, nd f (1 + 2i) in the form a+ib. If the function is undened
at 1 + 2i, state this fact.
1.
1
zz 5
2.
z
cos x + i sin y
Solution.
41
2 THE COMPLEX FUNCTION AND ITS DERIVATIVE
1. With z = 1 + 2i, we have zz = [z[
2
= 5. So that the fraction
1
zz 5
is not dened. That
means the function is undened at 1 + 2i.
Thus we cannot nd f (1 + 2i).
2. In this function, we can get a condition of denominator that is cos x ,= 0 and sin y ,= 0.
So the function is dened at 1 + 2i. Then f (1 + 2i) becomes
f (1 + 2i) =
1 + 2i
cos 1 + i sin 2
=
cos 1 + 2 sin 2 + i (2 cos 1 sin 2)
cos
2
1 + sin
2
2
Exercise 2.3. (E. 10, E. 13 P. 54)
Write the following functions of z in the form u(x, y) + iv (x, y), where u(x, y) and v (x, y) are
explicit real functions of x and y.
1.
1
z
+ i
2. z
3
+ z
Solution.
1. We put z = x + iy, where x, y are real numbers. We can call f (z) for this function and
nd u(x, y) and v (x, y) by simplifying it. For concreteness let us write
f (z) =
1
z
+ i
=
1
x + iy
+ i
=
x iy
x
2
+ y
2
+ i
=
x
x
2
+ y
2
+ i
_
1 +
y
x
2
+ y
2
_
We deduce the following result.
_
_
_
u(x, y) =
x
x
2
+y
2
v (x, y) = 1 +
y
x
2
+y
2
2. In a similar pattern, we also reduce this function.
42
2 THE COMPLEX FUNCTION AND ITS DERIVATIVE
f (z) = z
3
+ z
= (x iy)
3
+ x iy
= x
3
3x
2
iy 3xy
2
+ iy
3
+ x iy
= x
3
3xy
2
+ x + i
_
y
3
3x
2
y y
_
Hence, we get
_
_
_
u(x, y) = x
3
3xy
2
+ x
v (x, y) = y
3
3x
2
y y
Exercise 2.4. (E. 15, E. 17 P. 55)
Rewrite the following functions in terms of z and if necessary z as well as constants. Thus x
and y must not appear in your answer. Simplify your answer as much as possible.
1.
1
x
+
1
iy
2. x +
x
x
2
+ y
2
+ iy +
iy
x
2
+ y
2
Solution.
1. Using the following formula
_

_
x =
z + z
2
y =
1
i
z z
2
we can reduce a function of part (1). Moreover, we have no need of existent condition.
1
x
+
1
iy
=
2
z + z
+
2
z z
=
2z 2z + 2z + 2z
z
2
z
2
=
4z
z
2
z
2
2. In a similar way, we can apply the above method to simplify a function. But we note that
a denominator is exactly [z[. Hence, we have a distinct way as a solution.
x +
x
x
2
+ y
2
+ iy +
iy
x
2
+ y
2
= (x + iy) +
x + iy
x
2
+ y
2
= z +
z
[z[
2
43
2 THE COMPLEX FUNCTION AND ITS DERIVATIVE
Furthermore, we remember a formula of magnitude of one complex number that is [z[ =

zz. So we obtain that expression is equivalent to


z +
1
z
This is a result which we nd.
Exercise 2.5. (E. 19, E. 21 P. 55)
For each of the following functions, tabulate the value of the function for these values of z :
1, 1 + i, i, , 1 + i, 1. Indicate graphically the correspondence between values of w and values
of z.
1. w =
i
z
2. w = z
3
Solution.
1. In the following table and in Figure 1, we have investigated a few points in the case of
w = f (z) =
i
z
.
z w =
i
z
A = 1 A

= i
B = 1 + i B

=
i + 1
2
C = i C

= 1
D = 1 + i D

=
1 i
2
E = 1 E

= i
Then, we show the Figure 1 to clarify the exercise.
44
2 THE COMPLEX FUNCTION AND ITS DERIVATIVE
z-plane
w-plane
Figure 1.
2. In the following table and in Figure 1, we have interpolated a few points in the case of
w = f (z) = z
3
.
z w = z
3
A = 1 A

= 1
B = 1 + i B

= 2 + 2i
C = i C

= i
D = 1 + i D

= 2 + 2i
E = 1 E

= 1
Next, we present the Figure 1 to clarify the exercise.
45
2 THE COMPLEX FUNCTION AND ITS DERIVATIVE
z-plane
w-plane
Figure 1.
Exercise 2.6. (E. 24 P. 55)
Let f (z) =
1
z + i
. Find the following.
f
_
1
f (z)
_
Solution.
46
2 THE COMPLEX FUNCTION AND ITS DERIVATIVE
By considering f (z), we must have the initial condition which makes f (z)s existence. But
in this exercise, this can be omitted. So we have
f
_
1
f (z)
_
=
1
1
f (z)
+ i
=
f (z)
if (z) + 1
=
1
z + i
i
z + i
+ 1
=
1
z + 2i
, as a result.
Exercise 2.7. (E. 27 P. 55)
Using MATLAB obtain three-dimensional plots of the function f (z) =
1
z
3
2
i
and allow z to
assume values over a grid in the region of the complex plane dened by 1 x 1, 1 y 1.
Solution.
In this exercise, we present three kinds of three-dimensional plots. These are plots of modulus,
real part and imaginary part of f (z). Hence, we will respectively show these plots with the
corresponding scripts which are typed in MATLAB.
1. Plot of Modulus of f (z) :
>> x = -1 : 0.2 : 1;
>> y = -1 : 0.2 : 1;
>> [x , y] = meshgrid(x , y);
>> z = x + i*y;
>> w = 1./(z - 1.5*i);
>> surfc(x , y , abs(w))
47
2 THE COMPLEX FUNCTION AND ITS DERIVATIVE
Figure 1.
2. Plot of Real Part of f (z) :
>> x = -1 : 0.2 : 1;
>> y = -1 : 0.2 : 1;
>> [x , y] = meshgrid(x , y);
>> z = x + i*y;
>> w = 1./(z - 1.5*i);
>> surfc(x , y , real(w))
48
2 THE COMPLEX FUNCTION AND ITS DERIVATIVE
Figure 1.
3. Plot of Imaginary Part of f (z) :
>> x = -1 : 0.2 : 1;
>> y = -1 : 0.2 : 1;
>> [x , y] = meshgrid(x , y);
>> z = x + i*y;
>> w = 1./(z - 1.5*i);
>> surfc(x , y , imag(w))
49
2 THE COMPLEX FUNCTION AND ITS DERIVATIVE
Figure 1.
50
2 THE COMPLEX FUNCTION AND ITS DERIVATIVE
2.2 LIMITS AND CONTINUITY
This is an important section, so we show the exercise 5, 10, 11, 12, 14, 15, 16, 17, 18 because
we think that these are so necessary for solutions.
Exercise 2.8. (E. 5 P. 62)
Assuming the continuity of the functions f (z) = z and f (z) = c, where c is any constant. Prove
the continuity of the following function in the domain indicated. Take z = x + iy.
z
4
+
1 + i
z
2
+ 3z + 2
, all z ,= 1, 2.
Solution.
In this exercise, we can break the function into smaller parts. We then determine two functions
which is dened for all z ,= 1, 2.
_
_
_
h(z) = z
4
g (z) =
1 + i
z
2
+ 3z + 2
We must notice that for any z ,= 1, 2, we can write down a denominator of g (z) which is
(z + 1) (z + 2). Of course, this denominator is continuous function. By applying respectively
theorem of products and quotient of continuous functions, we gain h(z) that is continuous
function. Thus applying theorem of sums of continuous functions, we can derive that h(z)+g (z)
is continuous function.
Completing the proof by using theorem of continuous functions.
Exercise 2.9. (E. 10 P. 62)
Prove that the following function is continuous at z = i.
f (z) =
_
_
_
z i
z
2
3iz 2
, z ,= i
i , z = i
Solution.
Because f (i) is dened, we must determine lim
zi
z i
z
2
3iz 2
. For all z ,= i, we have a
denominator of f (z) which is equivalent to (z i) (z 2i). Moreover, we can state that
lim
zi
z i
z
2
3iz 2
= lim
zi
z i
(z i) (z 2i)
= lim
zi
1
z 2i
From this we might conclude that f (z) is a continuous function because of lim
zi
f (z) = f (i) =
i.
Hence, we have the completed proof.
51
2 THE COMPLEX FUNCTION AND ITS DERIVATIVE
Exercise 2.10. (E. 11 P. 62)
1. Consider the function f (z) =
z
2
5z + 6
z
2
4
dened for z ,= 2. How should this function
be dened at z = 2 so that f (z) is continuous at z = 2 ?
2. Consider the function f (z) =
z
4
+ 10z
2
+ 9
z
2
4iz 3
dened for z ,= 3i and z ,= i. How should
this function be dened at z = 3i and z = i so that f (z) is continuous everywhere ?
Solution.
1. Consider this function f (z), we note that we can simplify it because of factoring numerator
and denominator.
f (z) =
z
2
5z + 6
z
2
4
=
(z 2) (z 3)
(z 2) (z + 2)
=
z 3
z + 2
This function does not belong to one of rst conditions. This is z ,= 2 because the
expression z 2 is cancelled. Hence this function is really dened at z = 2. Moreover,
the following conditions are both satised easily : f (2) is dened; lim
z2
f (z) exists and
lim
z2
f (z) = f (2). According to denition of continuity of the complex function, we assert
f (z) is continuous at z = 2.
2. In a similar way, it is easy to reduce this function, which is not dened at z ,= 3i and
z ,= i, to dene for these points.
f (z) =
z
4
+ 10z
2
+ 9
z
2
4iz 3
=
_
z
2
+ 9
_ _
z
2
+ 1
_
(z 3i) (z i)
=
(z 3i) (z + 3i) (z i) (z + i)
(z 3i) (z i)
= (z + 3i) (z + i)
At this point, we can observe that f (z) is continuous everywhere. In details, if we take
an arbitrary complex number z
0
, we obtain explicitly that both conditions of continuity
of the comlex function are satised.
52
2 THE COMPLEX FUNCTION AND ITS DERIVATIVE
Exercise 2.11. (E. 12 P. 62)
In this problem we prove rigorously, using the denition of the limit at innity, that
lim
z
z
1 + z
= 1
1. Explain why, given > 0, we must nd a function r () such that

1
z + 1

< for all


[z[ > r.
2. Using one of the triangle inequalities, show that the preceding inequality is satised if we
take r > 1 +
1

.
Solution.
1. We present the following denition of the limit at innity :
If for every real number > 0 there exists a real number r () > 0 such that

1
z + 1
1

<
for all [z[ > r ().
If we have lim
z
z
1 + z
= 1 rstly, we can say the above denition. But if we need prove
lim
z
z
1 + z
= 1 as the mention of this exercise, we must alter the phrase for every into
given and the phrase there exists into nd, to see some works need to be make the
proof. For concretness let us write down
Given real number > 0, nd a real number r () > 0 such that

1
z + 1

<
for all [z[ > r ().
2. If we take r () > 1+
1

, we obtain [z[ > 1+


1

. We need verify that this r () is used in this


proof, which is either true or false. It means that the inequality

1
z + 1

< is satised.
In explicit, we have the triangle inequality [z + 1[ [z[ 1; so if we want to have the
inequality

1
z + 1

< , then we must have the inequality


1
[z[ 1
< or [z[ > 1 +
1

. This
fact is true because this inequality obtained.
Hence, we prove successfully that the preceding inequality is satised if we take r () >
1 +
1

.
Exercise 2.12. (E. 14 P. 63)
1. Knowing that f (z) = z
2
is everywhere continuous. Explain why the real function xy is
everywhere continuous.
2. Explain why the function g (x, y) = xy + i (x + y) is everywhere continuous.
Solution.
53
2 THE COMPLEX FUNCTION AND ITS DERIVATIVE
1. The form of z must be x+iy, where x, y are real numbers. Thus we have z
2
= x
2
y
2
+i2xy.
Because f (z) = z
2
is everywhere continuous and 2xy is an imaginary of this function, the
real function v (x, y) = 2xy is obviously everywhere continuous. Hence it is explicit to end
the explaination.
2. The continuity of complex function belongs to the continuity of both parts of that complex
function. We have that xy is really a continuous function. We observe that function x+y
is also a continuous function because it does not belong to an arbitrary condition. Thus
the function g (x, y) must be continuous everywhere.
Exercise 2.13. (E. 15 P. 63)
Show by nding an example, that the sum of two functions, neither of which possesses a limit
at a point z
0
, can have a limit at this point.
Solution.
Firstly, we take z
0
= 1 for the following example. Then, we will choose two functions which
have no a limit at z
0
= 1, witth assuming existent condition of denominator. These are
_

_
g (z) =
z
z
2
+ z 2
h(z) =
1
z
2
+ z 2
The reader can see easily these limits at z
0
= 1 that do not exist by factoring these denomi-
nators. But the function f (z) = g (z)+h(z) can have a limit exactly at z
0
= 1. For concretness
let us write
lim
z1
z 1
(z 1) (z + 2)
= lim
z1
1
z + 2
=
1
3
Finally, we show the exercise by nding an example.
Exercise 2.14. (E. 16 P. 63)
Show by nding an example, that the product of two functions, neither of which possesses a
limit at a point z
0
, can have a limit at this point.
Solution.
In a similar fashion, for the exercise 2.13 (E. 15 P. 63), we show that the product of two
functions, neither of which possesses a limit at a point z
0
, can have a limit at that point by
nding an example. This fact is not only true for the sum of two functions, but also the product
of two functions.
At this time, we take z
0
= 0, and choose g (z) =
[z[
z
; h(z) =
[z[
z
with z ,= 0 as the initial
condition. Although [z[ is really a real number and is not relate to limit of the considering
functions, its eect is important for our proof. In details, we consider
lim
z0
g (z) h(z) = lim
z0
[z[
2
zz
= 1
54
2 THE COMPLEX FUNCTION AND ITS DERIVATIVE
We see that the limit of this product exists and is always equal to 1. But it is easy to show
that g (z) and h(z) have no limit at z
0
= 0.
Then, we prove this exercise completely.
Exercise 2.15. (E. 17 P. 63)
Show that, in general, if g (z) has a limit as z tends to z
0
but h(z) does not have such a limit,
then f (z) = g (z) + h(z) does not have a limit as z tends to z
0
either.
Solution.
By showing mathematically, we must present the denition of limit and unlimitedness at a
point z
0
. We need analyse the assumption and conclusion in this needful proof. And we consider
three denitions of three clauses respectively.
g (z) has a limit as z tends to z
0

Let g
0
be a complex constant.
If for every real number > 0 there exists a real number () > 0 such that [g (z) g
0
[ < ,
for all z satisfying 0 < [z z
0
[ < ().
For the clause h(z) does not have such a limit, we must negate the denition of limit.
Let h
0
be a complex constant.
There exists a real number
1
> 0, for every real number
1
(z,
1
) > 0, have one z which
is satised 0 < [z z
0
[ <
1
(z,
1
), such that [h(z) h
0
[
1
.
We also negate the denition of limit for the conclusion f (z) = g (z) + h(z) does not
have a limit as z tends to z
0
.
Let f
0
= g
0
+ h
0
be a complex constant.
We need nd a real number
2
> 0, for every given real number
2
(z,
2
) > 0, have one z
which is satised 0 < [z z
0
[ <
2
(z,
2
), such that [f (z) f
0
[
2
.
We look at the inequality [f (z) f
0
[
2
and do not forget that f (z) = g (z) + h(z) and
f
0
= g
0
+h
0
. We deduce that [h(z) h
0
[g
0
g (z)][
2
. If we want to have this inequality,
we must have the inequality [h(z) h
0
[[g (z) g
0
[
2
because of [h(z) h
0
[g
0
g (z)][
[h(z) h
0
[ [g (z) g
0
[. According to the rst and the second clauses, we have [h(z) h
0
[
[g (z) g
0
[ >
1
. Hence, we can take =

2
2
,
1
= 2
2
and () =
1
(z,
1
) =
2
(z,
2
) to
clarify the proof.
55
2 THE COMPLEX FUNCTION AND ITS DERIVATIVE
Exercise 2.16. (E. 18 P. 63)
This problem deals with functions of a complex variable having a limit of innity (or ). We
say that lim
zz
0
f (z) = if, given p > 0, there exists a > 0 such that [f (z)[ > for all
0 < [z z
0
[ < . In other words, one can make the magnitude of f (z) exceed any preassigned
positive real number if one remains anywhere within a deleted neighborhood of z
0
. The radius
of this neighborhood, , typically depends on and shrinks as increases.
1. Using this denition, show that lim
z0
1
z
= . What should we takes as ?
2. Repeat the previous problem, but use the function
1
(z i)
2
as z i.
3. Consult a textbook on real calculus concerning the subject of innite limits and explain
why one does not say that lim
x0
1
x
= . What is the correct statement? Contrast this to
the result in (1).
4. The denition used above and in parts (1) and (2) cannot be used for functions whose
limits at innity are innite. Here we modify the denition as follows : We say that
lim
z
f (z) = if, given > 0, there exists r > 0 such that [f (z)[ > for all r < [z[. In
other words, one can make the magnitude of f (z) exceed any preassigned positive real
number if one is at any point at least a distance r from the origin. Using this denition,
show that lim
z
z
2
= . How should we choose r?
Solution.
1. To show lim
z0
1
z
= , according to the denition, it says that :
Given p > 0, we must nd a
p
> 0 such that
1
[z[
> p, for all 0 < [z[ <
p
.
At this point, we observe that inequality 0 < [z[ <
p
can be conclude a distinct form
1
[z[
>
1

p
. This fact is the same as
1
[z[
> p if we compare these. Hence, the
p
, which we
need nd, is obviously p.
Thus the proof is completed by choosing
p
= p.
2. We still use the above denition for proving lim
zi
1
(z i)
2
= . We have the following
clause
Given p > 0, we must nd a
p
> 0 such that
1
[z i[
2
> p, for all 0 < [z i[ <
p
.
If we unchange this clause to see the similarities, we cannot nd out
p
. We note that
0 < [z i[ <
p
can modify into 0 < [z i[
2
<
2
p
. Then if we put
p
=
2
p
> 0 and
[z i[
2
=

(z i)
2

= [w(z)[, we will see clearly that the clause is really the denition.
For concretness let us write again
Given p > 0, we must nd a
p
> 0 such that
1
[w(z)[
> p, for all 0 < [w(z)[ <
p
.
At this point, we only prove this clause in a same way of part (1).
56
2 THE COMPLEX FUNCTION AND ITS DERIVATIVE
2.3 THE COMPLEX DERIVATIVE
In this section, we present exercise 3, 12, 14, 18, 19 for examples.
Exercise 2.17. (E. 3 P. 70)
Assume that f (z) = z is not dierentiable. Obtain this conclusion by using the denition of
the derivative of a function of a complex variable and show that this results in your having to
evaluate lim
z0
2 arg (z). Why does this limit not exist?
Solution.
By using the denition of the derivative of a function of a complex variable, we take an
arbitrary point z
0
= x
0
+ iy
0
, where x
0
, y
0
are real numbers. We have f (z
0
) = x
0
iy
0
and
with z = x + iy, then f (z
0
+ z) = (x
0
+ x) i (y
0
+ y). Thus
f

(z
0
) = lim
z0
f (z
0
+ z) f (z
0
)
z
= lim
x,y0
(x
0
+ x) i (y
0
+ y) x + iy
x + iy
= lim
x,y0
x iy
x + iy
We put g (x, y) =
x iy
x + iy
, we must prove this limit which does not exist. In explicit,
we can nd two sequences which are also approach to 0, to prove that. For all n is an integer,
we take two sequences a
n
=
_
1
n
; 0
_
and b
n
=
_
0;
1
n
_
. At innity, they approach to (0; 0) but
g (a
n
) = 1 diers from g (b
n
) = 1. Thus these mean that lim
x,y0
g (x, y) is does exist.
Hence, f (z) is not dierentiable.
In a similar fashion, with z = x + iy, we can see that arg (z) =
y
x
. Hence,
lim
x,y0
2 arg (z) = lim
x,y0
2
y
x
Then we put h(x, y) = 2
y
x
and choose two sequences c
n
=
_
1
n
;
1
n
_
and d
n
=
_
1
n
;
1
n
_
for all n is an integer. They also approach to (0; 0) at innity but h(c
n
) = 2 diers from
h(d
n
) = 2. Thus this limit does not exist.
Exercise 2.18. (E. 12, E. 14 P. 70)
For what values of the complex variable z do the following functions have derivatives?
1. e
x
+ ie
2y
2. (x 1)
2
+ iy
2
+ z
2
Solution.
1. It is easy to know that u(x, y) = e
x
and v (x, y) = e
2y
. So we consider
57
2 THE COMPLEX FUNCTION AND ITS DERIVATIVE
_

_
u
x
= e
x
u
y
= 0
v
x
= 0
v
y
= 2e
2y
as a test. And then we see that the Cauchy-Riemann equations is satised if only if
e
x
= 2e
2y
since u(x, y) , v (x, y) are continuous in R.
Hence, if this function want to have derivate, the form of z must be log 2 + 2 log y + iy.
2. By using z = x + iy, we can simplify this function as follows
(x 1)
2
+ iy
2
+ z
2
= (x 1)
2
+ iy
2
+ (x + iy)
2
= (x 1)
2
+ x
2
y
2
+ i
_
y
2
+ 2xy
_
Then, we have u(x, y) = (x 1)
2
+ x
2
y
2
and v (x, y) = y
2
+ 2xy. At this point, we
derive
_

_
u
x
= 4x 2
u
y
= 2y
v
x
= 2y
v
y
= 2y + 2x
In a similar fashion, if this function want to have the derivate, we need 4x 2 = 2y +2x.
From this we can say that the form of z must be y + 1 + iy.
Exercise 2.19. (E. 18 P. 70)
Let f (z) = u(x, y) + iv (x, y). Assume that the second derivative f

(z) exists. Show that


f

(z) =

2
u
x
2
+ i

2
v
x
2
and
f

(z) =

2
u
y
2
i

2
v
y
2
Solution.
We showed that
f

(z) =
u
x
+ i
v
x
and
58
2 THE COMPLEX FUNCTION AND ITS DERIVATIVE
f

(z) =
v
y
i
u
y
as the exercise. So in the rst expression, we take the second derivate for x variable and for
y variable in second expression. We thus assert that
f

(z) =

2
u
x
2
+ i

2
v
x
2
and
f

(z) =

2
u
y
2
i

2
v
y
2
since the second derivative f

(z) exists.
Hence, the proof is completed.
Exercise 2.20. (E. 19 P. 70)
Show that if f

(z
0
) exists, then f (z) must be continuous at z
0
.
Solution.
The existence of f

(z
0
) means that f (z) has a derivate at point z
0
. The denition says that
this is dened by the following expression, provided the limit exists
f

(z
0
) = lim
z0
f (z
0
+ z) f (z
0
)
z
At this point, if we desire that f (z) is contiunous at z
0
, we need prove
lim
zz
0
f (z) = f (z
0
)
If we put z = z
0
+z in the above expression and turn the right side to the left side because
of denement of lim
zz
0
f (z
0
), it becomes
lim
z0
[f (z
0
+ z) f (z
0
)] = 0
, as the consistent proof.
Next, we consider this consistent proof which can be break into products of two limit. For
concretness let us write
lim
z0
[f (z
0
+ z) f (z
0
)] = lim
z0
f (z
0
+ z) f (z
0
)
z
z
= lim
z0
f (z
0
+ z) f (z
0
)
z
lim
z0
z
,z approachs to 0 but equals 0, that means z ,= 0. And it is easy to see that lim
z0
z = 0,
we can derive the consistent proof because of the existence of lim
z0
f (z
0
+ z) f (z
0
)
z
.
Therefore, we can end this proof successfully.
59
2 THE COMPLEX FUNCTION AND ITS DERIVATIVE
2.4 THE DERIVATIVE AND ANALYTICITY
We have exercise 2, 3, 9, 10, 11, 23 as the practice.
Exercise 2.21. (E. 2 P. 77)
1. Find the derivative of f (z) =
1
z
+(x 1)
2
+ixy at any points where the derivative exists.
Give the numerial value.
2. Where is this function analytic ?
Solution.
1. We have f(z) =
x iy
x
2
+ y
2
+(x 1)
2
+ixy (z ,= 0, it means that x, y dont equal to zero at
the same time)
f(x, y) = u(x, y) + iv(x, y) with u(x, y) =
x
x
2
+ y
2
+ (x 1)
2
and v(x, y) = xy
y
x
2
+ y
2
We use Cauchy-Riemann equations to nd this f(z)s derivative
u
x
= 2(x 1) +
x
2
+ y
2
2x
2
(x
2
+ y
2
)
2
= 2(x 1) +
y
2
x
2
(x
2
+ y
2
)
2
v
y
= x
x
2
+ y
2
2y
2
(x
2
+ y
2
)
2
= x
x
2
y
2
(x
2
+ y
2
)
2
Let
u
x
(x
0
, y
0
) =
v
y
(x
0
, y
0
) and we have 2(x
0
1) +
y
2
0
x
2
0
(x
2
0
+ y
2
0
)
2
= x
0

x
2
0
y
2
0
(x
2
0
+ y
2
0
)
2
Thus x
0
= 1
The second equations are
_
_
_
u
y
=
2xy
(x
2
+y
2
)
2

v
x
= y
2xy
(x
2
+y
2
)
2
By equality of these equations, we get y
0
= 0. So f (z) is derivative at z = 1 and
f

(1) = 1.
2. Because f (z) is only derivative at one point, f (z) is nowhere analytic.
Exercise 2.22. (E. 3 P. 77)
1. Where is the function f (z) = z
3
+ z
2
+ 1 analytic ?
2. Find an expression for f

(z) and give the derivative at 1 + i.


Solution.
1. Let z be x + iy for all x, y in R, we have
f (z) = u(x, y) + iv (x, y)
60
2 THE COMPLEX FUNCTION AND ITS DERIVATIVE
with u(x, y) = x
3
3xy
2
+ x
2
y
2
and v (x, y) = y
3
+ 3x
2
y + 2xy
Then we examine that f is statises Cauchy-Riemann equations. Indeed, we see that
_

_
u
x
= 3x
2
3y
2
+ 2x =
v
y
u
y
= 6xy 2y =
v
x
So this function is analytic everywhere.
2. We can get f

(z) = 3z
2
+ 2z easily because of similarity of real number. And then
f

(1 + i) = 3 (1 + i)
2
+ 2 (1 + i)
= 8i + 2
Exercise 2.23. (E. 9, E. 10 P. 77)
Use LHopitals Rule to establish these limits :
1.
(z i) +
_
z
2
+ 1
_
z
2
3iz 2
as z i.
2.
_
z
3
+ i
_
(z
2
+ 1) z
as z i.
Solution.
1. We set g(z) = z i +
_
z
2
+ 1
_
and h(z) = z
2
3iz 2
We can easily see that g(i) = h(i) = 0 as z = i and g(z), h(z) are dierentiable at z = i
(the reader can verify that). Moreover h

(i) = i ,= 0 so we can use LHopitals rule to


solve this limit
We have g

(z) = 1 + 2z and h

(z) = 2z 3i
Thus lim
zi
(z i) + (z
2
+ 1)
z
2
3iz 2
= lim
zi
1 + 2z
2z 3i
=
1 + 2i
i
= i 2
2. Like that exercise above, we set g(z) = z
3
+ i and h(z) =
_
z
2
+ 1
_
z. Of course g(i) =
h(i) = 0
g(z) and h(z) are dierentiable and h

(i) = 2 ,= 0, therefore we can use LHopitals rule


lim
zi
z
3
+ 1
(z
2
+ 1)z
= lim
zi
3z
2
3z
2
+ 1
=
3
2
=
3
2
Exercise 2.24. (E. 11 P. 77)
If g (z) has a derivative at z
0
and h(z) does not have a derivative at z
0
, explain why g (z)+h(z)
cannot have a derivative at z
0
.
Solution.
61
2 THE COMPLEX FUNCTION AND ITS DERIVATIVE
We can put g (z) = u(x, y) +iv (x, y) and h(z) = m(x, y) +in(x, y) to show the assumption
explicitly. With z
0
= (x
0
, y
0
), because of derivation of g (z), we have Cauchy-Riemann equations
at point z
0
.
_

_
u
x
(x
0
, y
0
) =
v
y
(x
0
, y
0
)
v
x
(x
0
, y
0
) =
u
y
(x
0
, y
0
)
And for the function h(z), it does not have a derivative at z
0
; this means one of Cauchy-
Riemann equations is not satised at least. We thus choose the following expression for illus-
tration.
m
x
(x
0
, y
0
) ,=
n
y
(x
0
, y
0
)
Then, we see that
m
x
(x
0
, y
0
) +
u
x
(x
0
, y
0
) ,=
n
y
(x
0
, y
0
) +
v
y
(x
0
, y
0
)
We note that if we take g (z) +h(z) = u(x, y) +m(x, y) +i [v (x, y) + n(x, y)], it is easily to
deduce the sum of these functions cannot have a derivative at z
0
by the above fact.
62
2 THE COMPLEX FUNCTION AND ITS DERIVATIVE
Exercise 2.25. (E. 23 P. 79)
Polar form of the Cauchy-Riemann equations.
1. Suppose, for the analytic function f (z) = u(x, y) + iv (x, y), that we express x
and y in terms of the polar variables r and , where x = r cos and y = r sin
_
r =
_
x
2
+ y
2
, = arctan
y
x
_
. Then f (z) = u(r, ) + iv (r, ). We want to rewrite the
Cauchy-Riemann equations entirely in the polar variables. From the chain rule for partial
dierentiation, we have
u
x
=
_
u
r
_

_
r
x
_
y
+
_
u

_
r
_

x
_
y
Give the corresponding expressions for
u
y
,
v
x
,
v
y
.
2. Show that
_
r
x
_
y
= cos and
_

x
_
y
=
sin
r
and nd corresponding expressions for
_
r
y
_
x
and
_

y
_
x
. Use these four expressions
in the equations for
u
x
,
u
y
,
v
x
and
v
y
found in part (1). Show that u and v satisfy the
equations
h
x
=
h
r
cos
1
r
h

sin and
h
y
=
h
r
sin +
1
r
h

cos
where h can equal u or v.
3. Rewrite the Cauchy-Riemann equations using the two equations from part (2) of this
exercise. Multiply the rst Cauchy-Riemann equation by cos , multiply the second by
sin , and add to show that
u
r
=
1
r
v

.
Now multiply the rst Cauchy-Riemann equation by sin , the second by cos , and add
to show that
v
r
=
1
r
u

.
These are the polar form of the Cauchy-Riemann equations. If the rst partial derivatives
of u and v are continuous at some point whose polar coordinates are r, (r ,= 0), then the
polar form of the Cauchy-Riemann equations provide a necessary and sucient condition
for the existence of the derivative at this point.
4. Use equation f

(z
0
) =
_
u
x
+ i
v
x
_
x
0
,y
0
and the Cauchy-Riemann equations in polar
form to show that if the derivative of f (r, ) exists, it can be found from
f

(z) =
_
u
r
+ i
v
r
_
[cos i sin ] or from f

(z) =
_
u

+ i
v

_ _
i
r
_
[cos i sin ]
Solution.
63
2 THE COMPLEX FUNCTION AND ITS DERIVATIVE
1. The exercise has already given us
u
x
=
_
u
r
_

_
r
x
_
y
+
_
u

_
r
_

x
_
y
so
u
y
,
v
x
,
v
y
are just the same expression as above, using Cauchy-Riemann equations
we have
u
y
=
_
u
r
_

_
r
y
_
x
+
_
u

_
r
_

y
_
x
v
x
=
_
v
r
_

_
r
x
_
y
+
_
v

_
r
_

x
_
y
v
y
=
_
v
r
_

_
r
y
_
x
+
_
u

_
r
_

y
_
x
2. We get r =
_
x
2
+ y
2
and tan =
y
x
so that
_
r
x
_
y
=
x
_
x
2
+ y
2
= cos
To solve
_

x
_
y
, rst we have to compute (tan )

cos
2

. Then we obtain
_

x
_
y
=
y
x
2
cos
2

=
y
x
2
1
1 +
y
2
x
2
=
sin
r
Let h be u, we replace
r
x
,

x
with these above and see that
h
x
=
h
r
cos
1
r
h

sin
Hence, we can do the same for the others.
3. Using the above part, we have
u
x
=
u
r
cos
1
r
u

sin
v
y
=
v
r
sin +
1
r
v

cos
We multiply them to cos and let them equal as Cauchy-Riemann rst equation. It means
64
2 THE COMPLEX FUNCTION AND ITS DERIVATIVE
u
x
cos =
v
y
cos
Express this and then
u
r
cos
2

1
r
u

sin cos =
v
r
sin cos +
1
r
v

cos
2

We do the same with Cauchy-Riemann second equation but we multiply them to sin
instead of cos and express them too. We obtain
v
r
cos sin
1
r
v

sin
2
=
v
r
sin
2

1
r
u

cos sin
The next is adddition of these two expression to each other and simplication of them.
Thus
u
r
cos
2

1
r
v

sin
2
=
1
r
v

cos
2

v
r
sin
2

Because of sin
2
+ cos
2
= 1, we deduce that
u
r
=
1
r
v

Hence we prove this part successfully, the reader can do the others.
4. We need to prove
f

(z) =
_
u
r
+ i
v
r
_
(cos i sin )
And we have
f

(z) =
u
x
+ i
v
x
We use part (b) to express
u
x
,
v
x
. Hence,
_

_
u
x
=
u
r
cos
1
r
u

sin (1)
v
x
=
v
r
cos
1
r
v

sin (2)
Then, we replace (1) and (2) into f(z).
u
r
cos
1
r
u

sin +i
_
v
r
cos
1
r
v

sin
_
= cos
_
u
r
+ i
v
r
_

1
r
sin
_
u

+ i
v

_
We are now using part (c) to simplify above equation
65
2 THE COMPLEX FUNCTION AND ITS DERIVATIVE
cos
_
u
r
+ i
v
r
_

1
r
sin
_
r
v
r
+ ri
u
r
_
Note that i
2
= 1, so we can deduce that
cos
_
u
r
+ i
v
r
_

1
r
sin
_
r
v
r
+ ri
u
r
_
= cos
_
u
r
+ i
v
r
_
i sin
_
i
v
r
+
u
r
_
=
_
u
r
+ i
v
r
_
(cos i sin )
We prove the exercise for the rst feedback, and in the second, the reader can show
similarly.
66
2 THE COMPLEX FUNCTION AND ITS DERIVATIVE
2.5 HARMONIC FUNCTIONS
The reader can see some exercises we will show such as 1, 2, 3, 6, 7, 8, 9, 11, 12, 13, 18, 20.
Exercise 2.26. (E. 1 P. 85)
Where in the complex plane will the function (x, y) = x
2
y
4
satisfy Laplaces equation? Why
isnt this a harmonic function?
Solution.
Take (x
0
, y
0
) is a point of function, we will nd this point such that (x
0
, y
0
) satises Laplaces
equation.

x
2
(x
0
, y
0
) +

2

y
2
(x
0
, y
0
) = 0
This is equivalent to
_

_
x
0
R
y
0
=
1

6
Because the domain of function which satises Laplaces equation is a subset of R
2
. Thus it
is not a harmonic function.
Exercise 2.27. (E. 2 P. 86)
Where in the complex plane will the function (x, y) = sin(xy) satisfy Laplaces equation? Why
isnt this a harmonic function?
Solution.
We have (x, y) = sin (xy), let take the first derivative and second derivative of (x, y)


x
= y cos (xy) and then

2

x
2
= y
2
sin (xy)


y
= xcos (xy)so that

2

y
2
= x
2
sin (xy)
Hence we obtain

x
2
+

2

y
2
=
_
x
2
+ y
2
_
sin (xy)
Dealing with the equation
_
x
2
+ y
2
_
sin (xy) = 0. we have the result is sin (xy) = 0 or x
2
+y
2
=
0. Then we can derive the roots below
_

_
y
0
=
k
x
0
, x
0
,= 0
y
0
= 0, x
0
= 0
Therefore, is not harmonic on its domain but harmonic on (0, 0) and
_
x
0
,
k
x
0
_
when x
0
,= 0.
67
2 THE COMPLEX FUNCTION AND ITS DERIVATIVE
Exercise 2.28. (E. 3 P. 86)
Consider the function (x, y) = e
ky
sin(mx). Assuming this function is harmonic throughout
the complex plane, what must be he relationship between the real constants k and m? Assume
that m ,= 0.
Solution.
This is a harmonic function then we can display Laplaces equation there.

x
2
+

2

y
2
= 0
It is equivalent to
m
2
e
ky
sin(mx) + k
2
e
ky
sin(mx) = 0
So we can derive that k
2
= m
2
.
Exercise 2.29. (E. 6 P. 86)
Putting z = x + iy. Show that 1
_
z
3
_
is harmonic in any domain.
Solution.
Factoring out z
3
, we obtain
z
3
= x
3
3xy
2
+ i
_
3x
2
y y
3
_
Thus 1
_
z
3
_
= x
3
3xy
2
. We will examine this function whether or not satisfying Laplaces
equation.

2
x
2
_
x
3
3xy
2
_
+

2
y
2
_
x
3
3xy
2
_
= 6x 6x = 0
Hence, it is harmonic in any domain.
Exercise 2.30. (E. 7 P. 86)
Find two values of k such that cos x
_
e
y
+ e
ky

is harmonic.
Solution.
We put u(x, y) = cos x
_
e
y
+ e
ky

is harmonic function. Then we get

2
u
x
2
+

2
u
y
2
= 0
It means that

_
e
y
+ e
ky
_
cos x +
_
e
y
+ k
2
e
ky
_
cos x = 0
After solving the above equation, we have k = 1.
Exercise 2.31. (E. 8 P. 86)
If g(x)
_
e
2y
e
2y

is harmonic, g(0) = 0, g

(0) = 1, nd g(x).
68
2 THE COMPLEX FUNCTION AND ITS DERIVATIVE
Solution.
Like the above exercise, we put u(x, y) = g(x)
_
e
2y
e
2y

is harmonic function and get


Laplaces equation.

2
u
x
2
+

2
u
y
2
= 0
This is equivalent to
_
e
2y
e
2y

(x) + 4g(x)
_
e
2y
e
2y

= 0
Then we need to solve the following equation
g

(x) + 4g(x) = 0
We can have general root is g(x) = C
1
cos 2x + C
2
sin 2x.
Exercise 2.32. (E. 9 P. 86)
1. Consider (x, y) = x
3
yy
3
x+y
2
x
2
+x. Show that this can be the part or the imaginary
part of an analytic function.
2. Assuming the preceding is the real part of an analytic function, nd the imaginary part.
3. Assuming that (x, y) is the imaginary part of an analytic function, nd the real part.
4. If (x, y) + iv(x, y) is an analytic function and if u(x, y) + i(x, y) is also analytic where
(x, y) is an arbitrary harmonic function, prove that, neglecting constants, u(x, y) and
v(x, y) must be negatives of each other.
Solution.
1. With (x, y) = x
3
y y
3
x +y
2
x
2
+x, we prove this can be real part or imaginary part
of an analytic function by proving this satisfy Laplaces equation.


x
= 3x
2
y y
3
2x + 1 so

2

x
2
= 6xy 2


y
= x
3
3y
2
x + 2y so

2

y
2
= 6xy + 2
Let

2

x
2
+

2

y
2
= 6xy 2 6xy + 2 = 0
Thus (x, y)satisfy Laplaces equation. Hence it is harmonic, therefore (x, y)can be real
part or imaginary part of an analytic function.
2. Assuming (x, y) is the real part of an analytic function, we need to find the imaginary
part v (x, y) of this function.
By applying Cauchy-Riemann equations


x
=
v
y
so
v
y
= 3x
2
y y
3
2x + 1
v =
3
2
x
2
y
2

y
4
4
2xy + y + C

(x)
69
2 THE COMPLEX FUNCTION AND ITS DERIVATIVE
The second equation is x
3
+ 3xy
2
2y .
Hence C

(x) = x
3
, it means that C (x) =
x
4
4
+ C.
Thus v (x, y) =
3
2
x
2
y
2

y
4
4
2xy + y +
x
4
4
+ C
3. As the same of part (b), let (x, y) be the imaginary part of an anlytic function and we
find u(x, y) is the real part of this function
We found u(x, y) = v (x, y) which we found in part (b)
4. We have f = u + i analytic so we deduce that
_

_
u
x
=

y
u
y
=

x
As the same g = + iv analytic so we derive that
_

x
=
v
y

y
=
v
x
By transforming these, we have
_

_
u
x
=
v
x
u
y
=
v
y
By neglecting constants, we see that u(x, y) = v (x, y) so we have what we need to prove.
Exercise 2.33. (E. 11 P. 86)
Find the harmonic conjugate of e
x
cos y + e
y
cos x + xy.
Solution.
Putting u the harmonic conjugate of v(x, y) = e
x
cos y + e
y
cos x + xy, we need to apply
Cauchy-Riemann equations :
_
_
_
u
x
=
v
y
u
y
=
v
x
We have
u
x
= e
x
cos ye
y
sin x+y. Then we can deduce that v = e
x
sin ye
y
sin x+
y
2
2
+C(x).
Hence, we obtain that
v
x
= e
x
sin y e
y
cos x + C

(x)
But we can see that
u
y
= e
x
sin y + e
x
cos x + x
Thus C

(x) = x. It means that C(x) =


x
2
2
+ C. Then the harmonic conjugate is e
x
sin y
e
y
sin x +
y
2
2
+
x
2
2
+ C.
70
2 THE COMPLEX FUNCTION AND ITS DERIVATIVE
Exercise 2.34. (E. 13 P. 86)
Show, if u(x, y) and v(x, y) are harmonic functions, that u+v must be a harmonic function but
that uv need not be a harmonic function.
Solution.
We get u, v are harmonic functions. That means
_

2
u
x
2
+

2
u
y
2
= 0

2
v
x
2
+

2
v
y
2
= 0
If u + v is harmonic, then need to show

2
(u + v)
x
2
+

2
(u + v)
y
2
= 0
This is equivalent to

2
u
x
2
+

2
v
x
2
+

2
u
y
2
+

2
v
y
2
= 0
This can be derived from Cauchy-Riemann equations above.
But if (uv) is harmonic, then we need to prove

2
(uv)
x
2
+

2
(uv)
y
2
= 0
We can see that

2
(uv)
x
2
= v

2
u
x
2
+ u

2
v
x
2
+ 2
v
x
u
x

2
(uv)
y
2
= v

2
u
y
2
+ u

2
v
y
2
+ 2
v
y
u
y
Then by using Cauchy-Riemann equations above, we get only that

2
(uv)
x
2
+

2
(uv)
y
2
= 2
_
v
x
u
x
+
v
y
u
y
_
It is not really equal to 0. Hence, we complete the proof.
Exercise 2.35. (E. 18 P. 86)
Show that f(z) = e
x
cos y + ie
x
sin y = u + iv is entire.
Solution.
Examining Cauchy-Riemann equations, we obtain
u
x

v
y
= e
x
cos y e
x
cos y = 0
u
y
+
v
x
= e
x
sin y + e
x
sin y = 0
Because it is analytic throughout nite z-plane, it is entire function.
71
2 THE COMPLEX FUNCTION AND ITS DERIVATIVE
Exercise 2.36. (E. 20 P. 87)
1. Let x = r cos and y = r sin , where r and are the usual polar coordinate variables.
Let f(z) = u(r, ) + iv(r, ) be a function that is analytic in some domain that does not
include z = 0. An assumed continuity of second partial derivatives to show that in this
domain u and v satisfy the dierential equation

r
2
+
1
r
2

2
+
1
r

r
= 0
This is Laplaces equation in the polar variables r and .
2. Show that u(r, ) = r
2
cos 2 is a harmonic function.
3. Find v(r, ), the harmonic conjugate of u(r, ), and show that it too satises Laplaces
equation everywhere.
Solution.
1. By using the polar coordinate variables, we have the form of the Cauchy-Riemann equa-
tions is now
_

_
u
r
=
1
r
v

v
r
=
1
r
u

In the system of equations, we deduce that

2
u
r
2
=
1
r
2
v

+
1
r

2
v
r

2
u

2
= r

2
v
r
Hence,

2
u
r
2
+
1
r
2

2
u

2
+
1
r
u
r
=
1
r
2
v

+
1
r

2
v
r

1
r

2
v
r
+
1
r
2
v

= 0
It is the same for case of v.
2. We see that u has continuity of second partial derivatives in the given domain. Thus we
can easily show that it is harmonic function by applying equation above.
3. Using the Cauchy-Riemann equations above, we have
v

= r
u
r
= 2r
2
cos 2
Then we deduce that v = r
2
sin 2 + C(r). Hence
72
2 THE COMPLEX FUNCTION AND ITS DERIVATIVE
v
r
= 2r sin 2 + C

(r)
But we note that
1
r
u

= 2r sin 2, then C

(r) = 0. It means that C(r) = C. So that


v = r
2
sin 2 + C.
Thus it too satises Laplaces equation everywhere.
73
3 THE BASIC TRANSCENDENTAL
FUNCTIONS
3.1 THE EXPONENTIAL FUNCTION
The reader can see some exercises below such as 2, 3, 4, 5, 6, 13, 14, 15, 16, 19, 20, 22, 23, 25,
26.
Exercise 3.1. (E. 2, E. 3, E. 4, E. 5, E. 6 P. 106)
Express each of the following in the form a + ib where a and b are real numbers. If the result
is multivalued, be sure to state all the values.
1. e
1/2+2i
2. e
1/22i
3. e
i
4. e
1/2+2i
e
1/22i
5. e
(i)
7
Solution.
It is easy for the reader or even the beginners. Indeed, we only need to get the brief solutions.
1. e
1
2
+2i
= e
1
2
(cos 2 + i sin 2) = e
1
2
cos 2 + ie
1
2
sin 2
2. e
1
2
2i
= e
1
2
(cos 2 i sin 2)
3. e
i
= cos (1) + i sin (1)
4. e
1
2
+2i
e

1
2
2i
= e
1
2
+2i
1
2
2i
= e
0
= 1
5. e
(i)
7
= e
(i)
6
(i)
= e
i
= e
0
(cos 1 i sin 1)
Exercise 3.2. (E. 13 P. 106)
Find all solution of e
z
= e by equating corresponding parts (reals and imaginaries) on bith sides
of the equation.
Solution.
We have
74
3 THE BASIC TRANSCENDENTAL FUNCTIONS
e
x+iy
= e
x
cos y + ie
x
sin y
e
1+i0
= e + i0
So e
x
cos y + ie
x
sin y = e + i0 and we can deduce that
_
_
_
e
x
cos y = e
e
x
sin y = 0
Then we take x = 1 and notice that cos x = 1 and sin x = 0 which mean tan y = 0. Thus
y = kwhere k Z
Exercise 3.3. (E. 14, E. 15, E. 16 P. 106)
Recalling that an analytic functionn of an analytic function is analytic, state the domain of
analyticity of each the following functions. Find the real and imaginary parts u(x; y) and v (x; y)
of the function, show that these satisfy the Cauchy-Riemann equations and find f

(x) in term
of z.
1. f (z) = e
iz
2. e
1/z
3. e
e
z
Solution.
1. Let z = x + iy. We have
f (z) = e
iz
= e
i(x+iy)
= e
ixy
= e
y
(cos x + i sin x) = e
y
cos x + ie
y
sin x
Thus
u(x; y) = e
y
cos x
v (x; y) = e
y
sin y
Check the Cauchy-Riemann equations
u
x
= e
y
sin x =
v
y
u
y
= e
y
cos y =
v
x
Thus f (z) analytic everywhere
75
3 THE BASIC TRANSCENDENTAL FUNCTIONS
f

(z) = ie
iz
2. We have
e
1
z
= e
xiy
x
2
+y
2
= e
x
x
2
+y
2
_
cos
y
x
2
+ y
2
+ i sin
y
x
2
+ y
2
_
_
_
_
u(x; y) = e
x
x
2
+y
2
cos
y
x
2
+y
2
v (x; y) = e
x
x
2
+y
2
sin
y
x
2
+y
2
We compute some following derivative and consider the Cauchy-Riemann equations
_

_
u
x
=
x
2
+ y
2
(x
2
+ y
2
)
2
.e
x
x
2
+y
2
. cos
2xy
(x
2
+ y
2
)
+
2xy
(x
2
+ y
2
)
2
.e
x
x
2
+y
2
. sin
y
x
2
+ y
2
=
v
y
u
y
=
2xy
(x
2
+ y
2
)
2
.e
x
x
2
+y
2
. cos
2xy
(x
2
+ y
2
)
+
x
2
+ y
2
(x
2
+ y
2
)
2
.e
x
x
2
+y
2
. sin
y
x
2
+ y
2
=
v
x
Thus f (z) = e
1
z
is an analytic function, so we can compute f

(z) is term of z
f

(z) =
1
z
2
e
1
z
3. In a similar way, we obtain
e
e
z
= e
e
x
(cos y+i sin y)
= e
e
x
cos y
[cos (e
x
sin y) + i sin (e
x
sin y)]
Then we set
u(x, y) = e
e
x
cos y
cos (e
x
sin y)
v (x, y) = e
e
x
cos y
sin (e
x
sin y)
We will use Cauchy-Riemann equations to examine its analytic domain
_

_
u
x
= e
x
e
e
x
cos y
cos (y) cos (e
x
sin y) e
e
x
cos y
sin (e
x
sin y) e
x
sin y
u
y
= e
x
e
e
x
cos y
sin (y) cos (e
x
sin y) e
e
x
cos y
sin (e
x
sin y) e
x
cos y
_

_
v
x
= e
x
e
e
x
cos y
cos (y) sin (e
x
sin y) + e
e
x
cos y
cos (e
x
sin y) e
x
sin y
v
y
= e
x
e
e
x
cos y
sin (y) sin (e
x
sin y) + e
e
x
cos y
cos (e
x
sin y) e
x
cos y
From the following equations we can see that f (z) satisfy Cauchy-Riemann equations on
entire Z plane. So f (z) is an entire function.
Thus we have f

(z) = e
z
e
e
z
76
3 THE BASIC TRANSCENDENTAL FUNCTIONS
Exercise 3.4. (E. 19 P. 106)
Consider the indentity e
(z
1
+z
2
)
= e
z
1
e
z
2
, which we proved somewhat tediously in this section.
Here is an elegant the proof which relies on our knowing that
de
z
dz
= e
z
and e
0
= 1.
1. Taking a as a constant, show that
d (e
z
e
az
)
dz
= 0 by using the usual formula for the
derivative of a product, as well as the derivative of e
z
, and the chain rule. Note that you
cannot combine the exponents, as this has not been justified.
2. Since e
z
e
az
has just been shown to bae a constant, which we will call k, evaluate k in
terms of a by using the fact that e
0
= 1.
3. Using e
z
e
az
= k as well as k found above, and z = z
1
, a = z
1
+ z
2
, show that e
z
1
+z
2
=
e
z
1
.e
z
2
.
Solution.
1. We have
d (e
z
e
az
)
dz
= e
z
.e
az
+
_
e
az
_

.e
z
= e
z
.e
az
+ e
z
_
e
az
_
= 0
2. Because of e
z
.e
az
= k where k is a constant, we obtain
e
z
e
a
e
z
= k then k = e
a
.
3. We see the equations below
_

_
e
a
= k
z = z
1
a = z
1
+ z
2
And notice that k = e
z
e
az
is equivalent to e
z
1
.e
z
1
+z
2
z
1
= e
z
1
+z
2
, hence e
z
1
e
z
2
= e
z
1
+z
2
.
Exercise 3.5. (E. 22, E. 23 P. 106)
For the following closed bounded regions, R, where does the given [f (z)[ achieve its maximum
and minimum values, and what are these values?
1. R is [z 1 i[ 2 and f (z) = e
z
2. R is [z[ 1 and f (z) = e
(z
2
)
Solution.
1. We have
77
3 THE BASIC TRANSCENDENTAL FUNCTIONS
[f (z)[ = [e
z
[ = e
x
And,
[z 1 i[ 2
is the circle (inside and on the bounded as well) centered 1 + i with radius 2.
So [f (z)[has maximum and minimum when [e
z
[ = e
x
has too that mean x is maximum
and minimum.
From the region R we can see that x achieve maximum when x = 3, y = 1 and minimum
when x = 1, y = 1. So that
max [f (z)[ = e
3
min [f (z)[ = e
1
2. We have
e
(z
2
)
= e
(x+iy)
2
= e
x
2
y
2
e
2xyi
= e
x
2
y
2
(cos 2xy + i sin 2xy)

e
(z
2
)

e
x
2
y
2
(cos 2xy + i sin 2xy) = e
x
2
y
2

And f (z) has maximum value of e


x
2
y
2
when x
2
has maximum and y
2
has minimum which
mean x = 1, y = 0. Thus max [f (z)[ = e.
Similarly, f (z) has minimum value of e
x
2
y
2
when x
2
has minimum and y
2
has maximum
which mean x = 0, y = 1. Hence, min [f (z)[ = e
1
.
78
3 THE BASIC TRANSCENDENTAL FUNCTIONS
Exercise 3.6. (E. 25 P. 107)
The absolute magnitude of the expression:
P () = 1 + e
i
+ e
2i
+ . . . + e
i(N1)
=
N1

n=0
e
in
is of interest in many problems involving radiation from N identical physical elements (e.g.,
antennas, loudspeakers). Here is real quantity that depends on the separation of the element
and the position of an observer of the radiation. [P[ can tell us the strength of the radiation
observed.
1. Using the formula for the sum of finite geometric series (see Exercise 27, section 1.4), show
that
[P ()[ =

sin
_
N
2
_
sin
_

2
_

2. Find lim
0
[P ()[
Solution.
1. Use the formular P () = 1 + e
i
+ e
2i
+ . . . + e
i(N1)
=
_
e
i
_
N
1
e
i
1
We obtain
[P ()[ =

_
e
i
_
N
1
e
i
1

_
e
i
_
N
1

[e
i
1[
=
[cos (N) 1 + i sin (N)[
[cos () 1 + i sin ()[
=
_
2 2 cos (N)

2 2 cos
=
_
4 sin
2 N
2
_
4 sin
2
2
=

sin
N
2
sin

2

2. We get e
iM0
= e
0
= 1 for all M N. Hence,
79
3 THE BASIC TRANSCENDENTAL FUNCTIONS
lim

[P ()[ = lim
0

1 + e
i
+ e
i2
+ . . . + e
i(N1)

1 + e
i.0
+ e
i.2.0
+ . . . + e
i.(N1).0

= [1 + 1 + 1 + . . . + 1[ = N
Exercise 3.7. (E. 26 P. 107)
Let z = re
i
, where r and are the usual polar variables.
Show that 1
_
1 + z
1 z
_
=
1 r
2
1 + r
2
2r cos
. Why must this function satisfy Eq. (2.5-14) through-
out any domain not containing z = 1.
Solution.
We have
1 + z
1 z
=
1 + re
i
1 re
i
=
1 + r (cos + i sin )
1 r (cos + i sin )
=
1 + r cos + ir sin
1 r cos ir sin
=
(1 + r cos + ir cos ) (1 r cos + ir sin )
(1 r cos )
2
+ r
2
sin
2

=
1 r
2
+ i2r sin
1 + r
2
2r cos
Hence, we easily see that
1
_
1 + z
1 z
_
=
1 r
2
1 + r
2
2r cos
Because we can easily prove that
1 + z
1 z
is analytic on the Z plane except z = 1 so when it is
analytic, its real and imaginary part must be harmonic so 1
_
1 + z
1 z
_
must satisfy equation.
80
3 THE BASIC TRANSCENDENTAL FUNCTIONS
3.2 TRIGONOMETRIC FUNCTIONS
The next are exercises 5, 9, 10, 12, 13, 14, 15, 16, 17, 25, 26, 27, 28, 29, 30.
Exercise 3.8. (E. 5, E. 9, E. 10 P. 112)
Using Eqs. (3.2-9) and (3.2-10), nd the numerical values of the following in the form a + ib,
where a and b are real numbers. If there is more than one numerical value, state all of them.
1. sin
_
i
1/2
_
2. tan
_
i arg
_
1 +

3i
__
3. arg (tan i)
Solution.
This exercise is basic for the beginner. We have 3 brief solutions there.
1. sin
_
i
1
2
_
= sin
_

2
2
+ i

2
2
_
= sin
_

2
2
_
cosh
_

2
2
_
+ i cos
_

2
2
_
sinh
_

2
2
_
2. tan
_
i arg
_
1 + i

3
__
= tan
_
i
_

3
+ k2
__
=
sin
_
i
_

3
+ k2
__
cos
_
i
_

3
+ k2
__ =
i sinh
_

3
+ k2
_
cosh
_

3
+ k2
_
3. arg (tan i) = arg
_
sin i
cos i
_
= arg
_
sin 0 cosh 1 + i cos 0 sinh 1
cos 0 cosh 1 i sin 0 sinh 1
_
= arg
_

_
i
(e
2
1)
2
_
e
2
+ 1
_
2
_

_
= arg
_
i
_
e
2
1
_
e
2
+ 1
_
=

2
+ k2
Exercise 3.9. (E. 12 P. 112)
Prove the identity sin
2
z + cos
2
z = 1 by the following two methods
1. Use the definitions of sine and cosine contained in Eq.(3.2-5) and (3.2-6).
2. Use cos
2
z + sin
2
z=(cos z + i sin z) (cos z i sin z) as well as Eulers identity generalized
to complex z.
Solution.
1. From the equation 3.2-5, we can get
sin
2
z =
_
e
iz
e
iz
2i
_
2
=
e
2iz
+ e
2iz
2
4
And the equation 3.2-6 is that
cos
2
z =
_
e
iz
+ e
iz
2
_
2
=
e
2iz
+ e
2iz
+ 2
4
We take a sum of sin
2
z and cos
2
z is
81
3 THE BASIC TRANSCENDENTAL FUNCTIONS
cos
2
z + sin
2
z =
2 + 2
4
= 1
2. It is easy to see that
cos
2
z + sin
2
z = (cos z + i sin z) (cos z i sin z)
= e
iz
.e
iz
= e
0
= 1.
Exercise 3.10. (E. 13, E. 14, E. 15 P. 112)
Using the denitions of the sine and cosine, Eqs. (3.2-5) and (3.2-6), prove the following.
1.
d
dz
sin z = cos z and
d
dz
cos z = sin z
2. cos
2
z =
1
2
+
1
2
cos 2z
3. sin (z + 2) = sin z and cos (z + 2) = cos z
Solution.
1. We will solve them directly.
d
dz
(sin z) =
d
dz
_
e
iz
e
iz
2i
_
1
2i
_
d
dz
_
e
iz
_

d
dz
_
e
iz
_
_
1
2i
_
ie
iz
(i) e
iz
_
1
2
_
e
iz
+ e
iz
_
= cos z
d
dz
(cos z) =
d
dz
_
e
iz
+ e
iz
2
_
=
1
2
_
d
dz
_
e
iz
_
+
d
dz
_
e
iz
_
_
=
1
2i
_
e
iz
(1) e
iz
_
=
1
2i
_
e
iz
e
iz
_
= sin z
2. The next is similar.
82
3 THE BASIC TRANSCENDENTAL FUNCTIONS
cos
2
z =
_
e
iz
+ e
iz
2
_
2
=
e
i2z
+ e
i2z
+ 2
4
=
1
2
+
1
2
cos 2z
3. Furthermore, we obtain
sin (z + 2) =
e
i(z+2)
e
i(z+2)
2i
=
e
iz
e
i2
e
iz
e
i2
2i
=
e
iz
e
iz
2i
= sin z
cos (z + 2) =
e
i(z+2)
+ e
i(z+2)
2
=
e
iz
e
i2
+ e
iz
e
i2
2
=
e
iz
+ e
iz
2
= cos z
Exercise 3.11. (E. 16, E. 17 P. 112)
1. Show that the equation sin z = 0 has solutions in the complex zplane only where z = n
and n = 0, 1, 2, . . . Thus like cos z; sin z has zeros only on the real axis.
2. Show that sin z cos z = 0 has solution only for real values of z. What are the solution?
Solution.
1. From the equation 3.2-5, we have
sin z =
e
iz
e
iz
2i
Hence,
e
iz
e
iz
2i
= 0
Simplify this and we get e
2iz
= 1. Thus the solution of sin z = 0 is 2z = k2 where k Z,
so that z = k.
83
3 THE BASIC TRANSCENDENTAL FUNCTIONS
We can see that this solution is real so sin z only has zeros on the real axis.
In a similar fashion, we can get the solution of cos z = 0 which are z =

2
+ m where
m Z. Then cos z also has zeros only on the real axis.
2. From the equations 3.2-5 and 3.2-6, we have cos z =
e
iz
+ e
iz
2
and sin z =
e
iz
e
iz
2i
.
Take a subtraction, we see that
sin z cos z =
e
iz
e
iz
2i

e
iz
+ e
iz
2
=
e
iz
(1 i) e
iz
(1 + i)
2i
Let sin z cos z = 0 which means
e
iz
(1 i) e
iz
(1 + i)
2i
= 0
Transform this and we get

2
_
e
i

e
i

4
z

_
= 0
Hence the solutions of this equation are 2z

2
= k2 where k Z. Then We get z =

4
+k
where k Z.
The reader can see that these solutions are real values so sin z cos z = 0 has solution
only for real values of z and its solutions are z =

4
+ k where k Z
Exercise 3.12. (E. 25, E. 26, E. 27, E. 28 P. 112)
1. Show that [cos z[ =
_
sinh
2
y + cos
2
x
2. Show that [sin z[ =
_
sinh
2
y + sin
2
x
3. Show that [sin z[
2
+[cos z[
2
= sinh
2
y + cosh
2
y
4. Show that
tan z =
sin 2x + i sinh 2y
cos 2x + cosh 2y
Solution.
84
3 THE BASIC TRANSCENDENTAL FUNCTIONS
[cos z[ = [cos xcosh y i sin xsinh y[
=
_
[cos xcosh y]
2
+ [sin xsinh y]
2
=
_
cos
2
x
_
1 + sinh
2
y

+ [1 cos
2
x] sinh
2
y
=
_
cos
2
x + cos
2
xsinh
2
y + sinh
2
y cos
2
xsinh
2
y
=
_
cos
2
x + sinh
2
y
[sin z[ =
_
sin
2
x. cosh
2
y + cos
2
x. sinh
2
y
=
_
sin
2
x.
_
1 + sinh
2
y
_
+ cos
2
x. sinh
2
y
=
_
sin
2
x + sinh
2
y
[sin z[
2
+[cos z[
2
= sinh
2
y + cos
2
x + sinh
2
y + sin
2
x
= 2 sinh
2
y + 1
= 2 sinh
2
y + cosh
2
y sinh
2
y
= sinh
2
y + cosh
2
y
tan z =
sin z
cos z
=
sin (x + iy)
cos (x + iy)
=
sin xcosh y + i cos xsinh y
cos xcosh y i sin xsinh y
=
(sin xcosh y + i cos xsinh y) (cos xcosh y + i sin xsinh y)
cos
2
xcosh
2
y + sin
2
xsinh
2
y
We have
cos
2
xcosh
2
y + sin
2
xsinh
2
y = cos
2
xcos
2
(iy) sin
2
xsin
2
(iy)
=
(1 + cos 2x) (1 + cos (2iy)) (1 cos 2x) (1 cos (2iy))
4
=
cos 2x + cos (2iy)
2
=
cos 2x + cosh (2iy)
2
Then
(sin xcosh y + i cos xsinh y) (cos xcosh y + i sin xsinh y) =
1
2
sin 2x
_
cosh
2
y sinh
2
y
_
+ i sinh y. cosh y
_
sin
2
x + cos
2
x
_
=
1
2
sin 2x + i sinh y. cosh y
=
1
2
sin 2x + sin (iy) . cos (iy)
=
1
2
(sin 2x + i sinh 2y)
85
3 THE BASIC TRANSCENDENTAL FUNCTIONS
So that
tan z =
sin 2x + i sinh 2y
cos 2x + cosh 2y
Exercise 3.13. (E. 29, E. 30 P. 113)
1. Show that
cot z =
sin 2x i sinh 2y
cosh 2y cos 2x
2. Since sin z = sin xcosh y +i cos xsinh y and [sinh y[ cosh y, show that [sinh y[ [sin z[
[cosh y[
3. Derive a comparable double inequality for [cos z[.
Solution.
1. We have
cot z =
cos z
sin z
=
cos (x + iy)
sin (x + iy)
=
cos xcosh y i sin xsinh y
sin xcos y + i cos xsinh y
=
(cos xcosh y i sin xsinh y) (sin xcos y i cos xsinh y)
sin
2
xcos
2
y + cos
2
xsinh
2
y
At this time, we do this is the same as exercise 28. Then we can complete the solution.
2. We get the inequality below.
[sin z[ =
_
sin
2
x + sinh
2
y
_
sinh
2
y [sinh y[
And
[sin z[ =
_
sin
2
xcosh
2
y + cos
2
xsinh
2
y
_
sin
2
xcosh
2
y + cos
2
cosh
2
y cosh
2
y
Thus
[sinh y[ [sin z[ [cosh y[
3. We obtain that
[cos z[ =
_
cos
2
x + sinh
2
y
_
sinh
2
y = [sinh y[
86
3 THE BASIC TRANSCENDENTAL FUNCTIONS
And
[cos z[ =
_
cos
2
x + cosh
2
y 1 =
_
cosh
2
y sin
2
x
_
cosh
2
y = [cosh y[
So a comparable double inequality is [sinh y[ [cos z[ [cosh y[
87
3 THE BASIC TRANSCENDENTAL FUNCTIONS
3.3 HYPERBOLIC FUNCTIONS
We show the solutions of exercises 1, 2, 5, 10, 11, 12, 18, 19.
Exercise 3.14. (E. 1, E. 2, E. 5 P. 114)
Use Equations. (3.3-1) and (3.3-2) to prove the following
1. sinh z = sinh x. cos y + i cosh x. sin y
2. cosh z = cosh x. cos y + i sinh x. sin y
3. sinh (i) = i sin and cosh (i) = cos . Thus the hyperbolic sine of a pure imaginary
number is a pure imaginary number while the hyperbolic cosine of a pure imaginary
number is a real number.
Solution.
1. We have
sinh z =
e
z
e
z
2
=
e
x+iy
e
xiy
2
=
e
x
(cos y + i sin y) e
x
(cos y i sin y)
2
= cos y.
_
e
x
e
x
2
_
+ i sin y.
_
e
x
+ e
x
2
_
= cos y. sinh x + i sin y. cosh x
2. We get
cosh z =
e
z
+ e
z
2
=
e
x+iy
+ e
xiy
2
=
e
x
(cos y + i sin y) + e
x
(cos y i sin y)
2
= cos y.
_
e
x
+ e
x
2
_
+ i sin y.
_
e
x
e
x
2
_
= cos y. cosh x + i sin y. sinh x
3. From exercises 1, 2, we subtitute z = i and notice that sinh 0 = 0 and cosh 0 = 1.
sinh i = cos . sinh 0 + i sin . cosh 0 = i sin
cosh i = cos . cosh 0 + i sin . sinh 0 = cos
Thus the hyperbolic sine of a pure imaginary number is a pure imaginary number while
the hyperbolic cosine of a pure imaginary number is a real number.
88
3 THE BASIC TRANSCENDENTAL FUNCTIONS
Exercise 3.15. (E. 10, E. 11 P. 114)
Find the numerical value of the following derivatives.
1.
d
dz
sinh (sin z) at z = i
2.
d
dz
sin (sinh z) at z = i
Solution.
1. We see that
d
dz
sinh (sin z) = cosh (sin z) . cos z
Then with z = i and the aid of Exercise 5,
cosh (sin i) . cos i = cosh (i sinh 1) . cos i
= cosh (sinh 1) . cosh 1 = 2.44
2. In a similar way, we consider
d
dz
sin (sinh z) = cos (sinh z) . cosh z
Then with z = i and the aid of Exercise 5 plus equation 3.2-12,
cos (sinh i) . cosh i = cos (i sin 1) .cos1
= cosh (sin 1) . cos 1 = 0.7431
Exercise 3.16. (E. 12 P. 114)
Consider the equation sinh (x + iy) = 0. Use equation 3.3-6 to equate the real and imaginary
parts of sinh z to zero. Show that this pair of equations can be satised if and only if z = in
where n Z . Thus the zeros of all sinh z all lie along the imaginary axis in the z-plane.
Solution.
In equation 3.3-6, we have sinh z = sinh x. cos y + i cosh x. sin y
Then let sinh x. cos y + i cosh x. sin y = 0, this means sinh x. cos y = 0 and cosh x. sin y = 0
The solutions are x = 0 and y = n where n Z
Hence z = in where n Z
Thus the zeros of all sinh z all lie along the imaginary axis in the z-plane.
89
3 THE BASIC TRANSCENDENTAL FUNCTIONS
Exercise 3.17. (E. 18, E. 19 P. 115)
Show that
1. [sinh z[
2
= sinh
2
x + sin
2
y
2. [cosh z[
2
= sinh
2
x + cos
2
y = cosh
2
x sin
2
y
Solution.
1. By using equation 3.3-6, we obtain
[sinh z[
2
= [sinh x. cos y + i cosh x. sin y[
2
= sinh
2
x. cos
2
y + cosh
2
x. sin
2
y
= sinh
2
x. cos
2
y +
_
1 + sinh
2
x
_
sin
2
y
= sinh
2
x.
_
cos
2
y + sin
2
y
_
+ sin
2
y
Hence we get [sinh z[
2
= sinh
2
x + sin
2
y
2. Use equation 3.3-7, we solve respectively.
[cosh z[
2
= [cosh x. cos y + i sinh x. sin y[
2
= cosh
2
x. cos
2
y + sinh
2
x. sin
2
y
=
_
1 + sinh
2
x
_
. cos
2
y + sinh
2
x. sin
2
y
= cos
2
y + sinh
2
x.
_
cos
2
y + sin
2
y
_
= sinh
2
x + cos
2
y
But notice that
cosh
2
x. cos
2
y + sinh
2
x. sin
2
y = cosh
2
x.
_
1 sin
2
y
_
+
_
cosh
2
x 1
_
. sin
2
y
= cosh
2
x cosh
2
x. sin
2
y + cosh
2
x. sin
2
y sin
2
y
= cosh
2
x sin
2
y
Thus we get
[cosh z[
2
= sinh
2
x + cos
2
y = cosh
2
x sin
2
y
90
3 THE BASIC TRANSCENDENTAL FUNCTIONS
3.4 THE LOGARITHMIC FUNCTION
The reader can see some exercises as examples such as 8, 9, 10, 11, 13, 18, 19, 23, 24, 26.
Exercise 3.18. (E. 8, E. 9, E. 10 P. 119)
Find all values of the logarithm of each of the following numbers and state in each case the
principal value. Put answers in the form a + ib.
1. e
log(i sinh 1)
2. e
e
i
3. Log (Logi)
Solution.
1. We deduce that
log z = log e
log(i sinh 1)
= log (i sinh 1)
= log [sinh 1 (0 + i)]
= log
_
sinh 1
_
cos

2
+ i sin

2
__
= Log (sinh 1) + i
_

2
+ k2
_
, k Z
Then we set =

2
+ k2 and get
Logz = Log (sinh 1) + i

2
2. We have
log
_
e
e
i
_
= log
_
e
cos 1+i sin 1
_
= log
_
e
cos 1
(cos (sin 1) + i sin (sin 1))

= Log
_
e
cos 1
_
+ i [arccos (cos (sin 1)) + k2] , k Z
The principal value is Log
_
e
cos 1
_
+ i arccos (cos (sin 1)) when k = 0.
3. We get
log Log (Logi) = log Log
_
i

2
_
= log
_
Log

2
+ i arcsin

2
_
= Log
_
Log
2

2
+ arcsin
2

2
+ i (1 + k2)
91
3 THE BASIC TRANSCENDENTAL FUNCTIONS
The principal value is Log
_
Log
2

2
+ arcsin
2

2
+ i1 when k = 0.
Exercise 3.19. (E. 11 P. 119)
For what values of z is the equation Logz = Logz true?
Solution.
We have equation 3.4-5
Logz = Log [z[ + arg z
Log z = Log [ z[ + arg z = Log [z[ arg z
Log z = Log [ z[ arg z = Log [z[ + arg z
The question is Logz = Log z, it means that
Log [z[ + arg z = Log [ z[ arg z
This is equivalent to
arg z = arg z
With z = x + iy, x ,= 0, we see that
arg
y
x
= arg
_

y
x
_
Then we obtain the fact that
y
x
=
y
x
.
Thus, the solution of equation for z R with 1z ,= 0.
Exercise 3.20. (E. 13 P. 119)
Give solutions to the following equations in Cartesian form.
(Logz)
2
+ Logz = 1
Solution.
From the equation above, we see that
(Logz)
2
+ Logz + 1 = 0
We have = 3i
2
, so that
_

_
Logz =
1 + i

3
2
Logz =
1 i

3
2
92
3 THE BASIC TRANSCENDENTAL FUNCTIONS
Exercise 3.21. (E. 18, E. 19 P. 119)
Use logarithm to nd all solutions of the following equations.
1. (e
z
1)
2
= e
z
2. (e
z
1)
3
= 1
Solution.
1. The equation given is equivalent to
(e
z
)
2
3e
z
+ 1 = 0
Then we can derive the roots below.
_

_
e
z
=
3 +

5
2
e
z
=
3

5
2
From these roots, we take logarithm both sides to get
_

_
z = log
3 +

5
2
+ il2
z = log
3

5
2
+ ik2
where k, l Z.
2. Because the case e
z
= 0 is discarded, the equation becomes
e
z
= 2
It is very easy to deal with by taking logarithm both sides, so we obtain
z = Log2 + ik2, k Z
Exercise 3.22. (E. 23, E. 24 P. 119)
Prove that if is real, then
1. 1
_
log
_
1 + e
i
_
= Log

2 cos

2

if e
i
,= 1
2. 1
_
log
_
re
i
1
_
=
1
2
Log
_
1 2r cos + r
2
_
if r 0, re
i
,= 1
Solution.
1. We have
93
3 THE BASIC TRANSCENDENTAL FUNCTIONS
1 + e
i
= cos + i sin + cos 0 + i sin 0
= cos + cos 0 + i (sin + sin 0)
= 2 cos

2
cos

2
+ i2 sin

2
cos

2
= 2 cos

2
_
cos

2
+ i sin

2
_
So that, we conclude
log
_
1 + e
i
_
= Log

2 cos

2

+ i
_

2
+ k2
_
where k Z
Hence,
1
_
log
_
1 + e
i
__
= Log

2 cos

2

2. We have
e
i
= cos + i sin
Then taking logarithm both sides, we see that
log (r cos 1 + ir sin ) = Log
_
r
2
cos
2
+ 1 2r cos + r
2
sin
2
+ i
_
arcsin
r sin
_
r
2
cos
2
+ 1 2r cos + r
2
sin
2

+ k2
_
= Log
_
r
2
+ 1 2r cos + i
_
arcsin
r sin

r
2
+ 1 2 cos
+ k2
_
=
1
2
Log
_
r
2
+ 1 2r cos
_
+ i
_
arcsin
r sin

r
2
+ 1 2 cos
+ k2
_
Hence, we get the proof completely.
1
_
log
_
re
i
1
_
=
1
2
Log
_
1 2r cos + r
2
_
, r 0, re
i
,= 1
Exercise 3.23. (E. 26 P.119)
Consider the identity log z
n
= nlog z, where n is an integer, which is valid for appropriate
choices of the logarithms on each side of the equation. Let z = 1 + i and n = 5.
1. Find values of log z
n
and log z that satisfy nlog z = log z
n
2. For the given z and n is nLogz = Logz
n
satised?
3. Suppose n = 2 and z is unchanged. Is nLogz = Logz
n
then satised?
94
3 THE BASIC TRANSCENDENTAL FUNCTIONS
Solution.
1. By letting the condition above, we have
log (1 + i)
5
= 5 log (1 + i)
We see that
log (1 + i) = log

2 + i
_

4
+ k2
_
Then
log (1 + i)
5
= 5 log

2 + 5i
_

4
+ k2
_
, k Z
2. Take k = 0, we obtain
Log (1 + i)
5
= Log

2
5
+ 5i
_

4
_
It means that
5Log (1 + i) = 5
_
log

2 + i

4
_
= 5 log

2 + i5

4
Thus, with n = 5, z = 1 + i then we get Log (1 + i)
5
= 5Log (1 + i)
3. We still analyse like the above.
Log (1 + i)
2
= log 2 + i

2
2Log (1 + i) = 2
_
log

2 + i

4
_
= log 2 + i

2
Thus, with n = 2, z = 1 + i, we obtain Log (1 + i)
2
= 2Log (1 + i)
95
3 THE BASIC TRANSCENDENTAL FUNCTIONS
3.5 ANALYTICITY OF THE LOGARITHMIC FUNCTION
We consider the exercises 2, 3, 4, 6, 7, 8, 10, 12.
Exercise 3.24. (E. 2 P. 126)
Suppose that: f (z) = log z = Logr + i, 0 < 2
1. Find the largest domain of analyticity of this function.
2. Find the numerical value of f
_
e
2
_
3. Explain why we cannot determine f
_
e
2
_
within the domain of analyticity.
Solution.
1. f (z) is discontinuous at origin z = 0, since log 0 not identified. discontinuous at the
point on the positive real axis. Because the point on the positive real axis value = 0,
while points located in quadrant-four while
lim
0
= 2
Cut branches located in the plane xy, so f (z) is a branch of logarithm analytic in the
domain D as follows
D = C z : z = 0, 1z 0
2. We consider the function f
_
e
2
_
= log
_
e
2
_
and have e
2
= e
2
e
i
. So we take logarithm
both sides as follows
log
_
e
2
_
= log
_
e
2
e
i
_
= Log
_
e
2
_
+ i ( + k2) , k Z
Then, because of 0 = + k2 < 2, we choose k = 0, it means = .
Hence,
log
_
e
2
_
= 2Loge + i
3. We have
log
_
e
2
_
= Log
_
e
2
_
+ i (0 + k2) , k Z
When we choose k = 0, at that point ,log
_
e
2
_
is (2; 0) located on the positive real axis in
the plane xy, in which the function is not analytic. Therefore, we can not define f
_
e
2
_
in
the analytic domain.
96
3 THE BASIC TRANSCENDENTAL FUNCTIONS
Exercise 3.25. (E. 3, E. 4, E. 6 P. 126)
Consider a branch of log z analytic in the domain created with the branch cut x = 0, y 0. If
for this branch, log (1) = i, find the following:
1. log (1)
2. log (ie)
3. log
_

3 + i
_
Solution.
1. Using the equation 3.4-2 along with the definition of branch cut and we achieve
log (1) = log
_
e
i0
_
= Log1 + i (0 + 2k) , k Z
We need to find , consider that

3
2
< = 0 + k2 <

2
So = 0
Thus, we obtain
log 1 = Log1 + i0 = 0
2. Using the equation 3.4-2 along with the definition of branch cut and we achieve
arg (ie) = +

2
=

2
Hence,
log (ie) = log e + i
_

2
_
= 1
i
2
3. Using the equation 3.4-2 along with the definition of branch cut and we achieve
log
_

3 + i
_
= log
_
2e
i5
6
_
= i
_
5
6
+ k2
_
, k Z
We find from the inequality below.

3
2
< =
5
6
+ k2 <

2
97
3 THE BASIC TRANSCENDENTAL FUNCTIONS
So =
7
6
.
Thus
log
_

3 + i
_
= Log2 + i
_

7
6
_
Exercise 3.26. (E. 8, E. 10 P. 127)
Consider a branch of log z analytic in the domain created with the branch cut x = y, x 0.
If, for this branch , log (1) = 2i, find the following.
1. log (i)
2. log (ie)
Solution.
1. We see that x = y and x > 0 is the branch cut so
9
4
< <

4
Using this branch cut and we get the value of
arg z = 2 +

2
=
3
2
log i = log [i[ i
3
2
= i
_

3
2
2
_
2. We consider that x = y and x > 0 is the branch cut so
9
4
< <

4
Using this branch cut and we get the value of
log (ie) log
_
e.e
i(

2
)
_
= Loge + i
_

2
+ k2
_
, k Z
We nd k from

9
4
< =

2
+ k2 <

4
, k Z
So we have =

2
, k = 0.
Thus
log (ie) = 1

2
i
98
3 THE BASIC TRANSCENDENTAL FUNCTIONS
Exercise 3.27. (E. 12 P. 127)
1. Show that Logz = Log
1
z
is valid throughout the domain of analyticity of Logz.
2. Find a nonprincipal branch of log z such that log z = log
1
z
is not satisfied somewhere
in your domain of analyticity of log z. Prove your result.
Solution.
1. We have log z = log [z[ + i arg where < arg z <
Now, if arg z lies between and , we can take arg
1
z
= arg z
And we will get < arg
1
z
<
Thus,
log
1
z
= log

1
z

+ i arg z
= log [z[ i arg z
Or we can say that
log z = log
1
z
2. We obtain that log i =
i
2
log
1
i
= log (i) = i
3
2
And note that
log i ,= log
1
i
99
3 THE BASIC TRANSCENDENTAL FUNCTIONS
3.6 COMPLEX EXPONENTIALS
We go together to solve exercises 8, 9, 11, 13, 16, 18, 19, 20, 24, 25.
Exercise 3.28. (E. 8, E. 9, E. 10 P. 132)
Find all values of the following in the form a + ib and state the principal value.
1. (Logi)
/2
2. (1 + i tan i)

2
Solution.
1. We have
Logi = Log
_
cos

2
+ i sin

2
_
= Log1 + i

2
+ k2
= i

2
+ k2
=

2
_
cos

2
+ i sin

2
_
+ k2
Thus
log (Logi) = Log

2
+ i
_

2
+ 2k
_
Moreover, we get
e

2
Log

2
_
cos
_

2
4
+ k
2
_
+ i
_

2
4
+ k
2
__
When k = 0, the principal value is
(Logi)

2
= e

2
Log

2
+

2
4

= e

2
Log

2
_
cos

2
4
+ i sin

2
4
_
2. We have
(1 + i tan 1)

2
= e

2. log(1+i tan 1)
We see the exponent here and make some changes
log (1 + i tan 1) = Log
_
1 + tan
2
1 + i arccos
1

1 + tan
2
1
= Log

1
cos 1

+ i (1 + 2k)
100
3 THE BASIC TRANSCENDENTAL FUNCTIONS
Hence,
e

2 log(1+i tan 1)
= e

2(Log[
1
cos 1
[+i(1+k2))
=
_
1
cos 1
_

2
.e
(

2+2

2k)i
And the principal value is
_
1
cos 1
_

2
.e

2i
Exercise 3.29. (E. 11 P. 132)
Show that all possible values of z
i
are real if [z[ = e
n
, where n is any integer.
Solution.
We have
z
i
= e
i log z
= e
i(log|z|+i(+k2))
= e
i log|z|(+k2)
With [z[ = e
n
, the equality above is equivalent to
e
i log e
n
.e
(+k2)
= e
in
.e
(+k2)
= cos (n) .e
(k2)
R, n N
Exercise 3.30. (E. 13 P. 132)
Using Eq. (3.1-5) of Eq. (3.1-7) and the denition in Eq. (3.6-1), prove that for any complex
values , and z, we have the following.
The values of z

are identical to the values of z


+
.
Solution.
From the equation 3.6-1, we have
z

= e
log z
z

= e
log z
Then we multiply them side by side, we get
101
3 THE BASIC TRANSCENDENTAL FUNCTIONS
z

= e
log z
e
log z
= e
log z+ log z
= e
(+) log z
= z
+
Exercise 3.31. (E. 16, E. 18 P. 132)
Using the principal branch of the function, evaluate the following.
1. f

(i) if f (z) = z
2+i
2. f

(8i) if f (z) = z
1/3+i
Solution.
1. We must get the derivative of this function, but we can change the form before.
f (z) = z
2+i
= e
(2+i)Logz
f

(z) =
_
e
(2+i)Logz
_

=
2 + i
z
e
(2+i)Logz
With z = i, we have
f

(i) =
2 + i
i
e
(2+i)Logi
=
2 + i
i
e
(2+i)i

2
=
2 + i
i
e

2
+i
2. In exercise above, we can use
f

(z) =
_
e
(2+i)Logz
_

=
2 + i
z
e
(2+i)Logz
f

(8i) =
_
1 + i
1
3
_
e
(
1
3
+i)Log(8i)
8
Then we evaluate them respectively.
Log (8i) = Log8 i

2
_
1
3
+ i
_
Log (8i) = Log2 +

2
+ i
_
Log8

6
_
102
3 THE BASIC TRANSCENDENTAL FUNCTIONS
So that, we conclude
f

(8i) =
(3 + i)
24
e
Log2+

2
cis
_
Log8

6
_
=
1
12
(3 + i) e

2
cis
_
Log8

6
_
Exercise 3.32. (E. 19, E. 20 P. 132)
Let f (z) = z
z
, where the principal branch is used. Evaluate the following.
1. f

(z)
2. f

(i)
Solution.
We need to nd the derivative now.
f

(z) =
d
dz
z
z
=
d
dz
e
z. log z
= (log z + 1) e
z. log z
= (log z + 1) .z
z
So take z = i here, we obtain
f

(i) = (log i + 1) e
i log i
=
_
i
2
+ 1
_
e
i
i
2
=
_
i
2
+ 1
_
e

2
Exercise 3.33. (E. 21, E. 24, E. 25 P. 132)
1. Let f (z) = z
sin z
, where the principal branch is used. Find f

(i).
2. Let f (z) = 10
z
3
. This function is evaluated such that f

(z) is real when z = 1. Find


f

(1 + i). Where in the complex plane is f (z) analytic?


3. Let f (z) = 10
e
z
. This function is evaluated such that [f (i/2)[ = e
2
. Find f

(z) and
f

(i/2).
Solution.
1. We still use old method above to get the derivative.
103
3 THE BASIC TRANSCENDENTAL FUNCTIONS
f (z) = z
sin z
= e
sin zLogz
f

(z) =
_
e
sin zLogz
_

=
_
cos zLogz +
sin z
z
_
e
sin zLogz
, z ,= 0
With z = i, we have
f

(i) =
_
cos iLogi +
sin i
i
_
e
sin iLogi
=
_
i

2
cos i i sin i
_
e
i

2
sin i
2. Like the exercise above, we get
f

(z) =
d
dz
_
e
z
3
. log 10
_
= 3z
2
log 10.e
z
3
. log 10
= 3z
2
log 10.f (z)
With z = 1 + i, then we deduce that
f

(1 + i) = 3 log 10. (1 + i)
2
.10
(1+i)
3
= 3 log 10.2i.10
2i2
And notice that f (z)analytic when z ,= 0.
3. In a similar fashion, we change the form of the function
f (z) = 10
(e
z
)
= e
e
z
log 10
So that the derivative is
f

(z) =
_
10
(e
z
)
_

= e
(Log10+i2)e
z
(Log10 + i2) e
z
With z =
i
2
, the result we need is
f

_
i
2
_
= 0.00464 i0.0116
104
3 THE BASIC TRANSCENDENTAL FUNCTIONS
3.7 INVERSE TRIGONOMETRIC AND HYPERBOLIC
FUNCTIONS
In this section, we have exercise 2, 4, 7, 8, 16, 17.
Exercise 3.34. (E. 2 P. 137)
1. Show that if we dierentiate a branch of arccos z, we obtain
d
dz
arccos z =
1
(1 z
2
)
1/2
2. Obtain
d
dz
arcsin z =
d
dz
_
i log
_
zi +
_
1 z
2
_
1/2
__
=
1
(1 z
2
)
1/2
by noting that
z
2
= sin
2
w = 1 cos
2
w = 1
_
dz
dw
_
2
can be solved for dw/dz.
3. Obtain
d
dz
sinh
1
z =
1
(1 + z
2
)
1/2
directly from sinh
1
z = log
_
z +
_
z
2
+ 1
_
1/2
_
; obtain it also by a procedure similar to
part (2) of this exercise.
Solution.
1. We need to nd the inverse function of cosine from the equation z = cos w. We have
z = cos w =
e
iw
+ e
iw
2
And then we put y = e
iw
, the equation above becomes y
2
2zy + 1 = 0, so the root is
y = z + i

1 z
2
. It means that
w = i log
_
z + i
_
1 z
2
_
Hence,
arccos z = i log
_
z + i
_
1 z
2
_
Then, the derivative is
105
3 THE BASIC TRANSCENDENTAL FUNCTIONS
d
dz
arccos z =
d
dz
_
i log
_
z + i
_
1 z
2
__
=
1
(1 z
2
)
1/2
2. In a similar fashion, from the given equality, we deduce that
_
dz
dw
_
2
= z
2
1
Next, we inverse both sides to see the proof needed.
3. We have
z
2
= sinh
2
w = 1 + cosh
2
w
This is equivalent to
z
2
= 1 +
_
dz
dw
_
2
Thus we see that
dw
dz
=
1
(z
2
+ 1)
1/2
Exercise 3.35. (E. 4, E. 7, E. 8 P. 137)
Find all solutions to the following equations.
1. cos w = 3
2. sinh w = i

2
3. cosh
2
w = 1
Solution.
1. Because of the exercise above, we obtain
w = arccos 3 = i log
_
3 2

2
_
+ k2
2. We have
sinh
1
z = log
_
z +
_
z
2
+ 1
_
So that from this, we obtain
106
3 THE BASIC TRANSCENDENTAL FUNCTIONS
w = sinh
1
_
i

2
_
= log
_
i

2 +
_
_
i

2
_
2
+ 1
_
= log
_
i

2 + i
_
Hence,
w = log
_

2 + 1
_
+ i
_

2
+ k2
_
3. We see the formular below.
cosh w =
1
2
_
e
w
+ e
w
_
Then we square both sides and get the equation, it means that
1
4
_
e
2w
+ e
2w
+ 2
_
= 1
We easily solve this equation and obtain the root is e
2w
= 1. Hence,
w = ik
Exercise 3.36. (E. 16 P. 137)
Show that tanh
1
_
e
i
_
=
1
2
log
_
i cot
_

2
__
.
Solution.
We start with the left side.
tanh
1
_
e
i
_
=
1
2
log
_
1 + e
i
1 e
i
_
=
1
2
log
_
e
i
2
+ e
i
2
e
i
2
e
i
2
_
Moreover, we have
cot
_

2
_
= i
e
i
2
+ e
i
2
e
i
2
e
i
2
or we can say that
e
i
2
+ e
i
2
e
i
2
e
i
2
= i cot
_

2
_
107
3 THE BASIC TRANSCENDENTAL FUNCTIONS
So that we can complete the proof here.
Exercise 3.37. (E. 17 P. 137)
Find a formula similar to the one above for tan
1
_
e
i
_
.
Solution.
We have a formula below.
tan
1
_
e
i
_
=
i
2
log
_
i + e
i
i e
i
_
Also, we can observe that
i + e
i
i e
i
=
_
i + e
i
_
2
1 e
2i
=
_
1 + e
2i
1 + e
2i
+
2ie
i
1 + e
2i
_
=
_
e
i
e
i
e
i
+ e
i
+
2i
e
i
+ e
i
_
= i
sin + 1
cos
So that we obtain the proof by replacing the equality above into a formula of tanh
1
_
e
i
_
.
108
4 INTEGRATION IN THE COMPLEX PLANE
4.1 INTRODUCTION TO LINE INTEGRATION
We can evaluate exercises 2, 3, 6, 7 to get some examples.
Exercise 4.1. (E. 2, E. 3 P. 160)
Let C be that portion of the curve y = x
2
lying between (0, 0) and (1, 1). Let F (x, y) = x+y+1.
Evaluate these integrals along C.
1.

1,1
0,0
F (x, y) dx
2.

1,1
0,0
F (x, y) dy
Solution.
1. Along the path of integration, y changes with x. The equation of the contour of integration
y = x
2
can be used to express y as a function of x in the preceding integrand. Thus

1,1
0,0
F (x, y) dx =

1,1
0,0
_
x + x
2
+ 1
_
dx
=
x
2
2
+
x
3
3
+ x

x=1
x=0
=
11
6
2. To change this to a conventional integral, we may regard x as a function of y along C.
Since y = x
2
and x 0 on the path of integration, we obtain x =

y. Hence

1,1
0,0
F (x, y) dy =

1,1
0,0
(

y + y + 1) dy =
13
6
Exercise 4.2. (E. 6 P. 160)
Let C be that portion of the curve x
2
+ y
2
= 1 lying in the rst quadrant. Let F (x, y) = x
2
y.
Evaluate the integral along C.

1,0
0,1
F (x, y) ds
Solution.
Because of portion of the curve x
2
+y
2
= 1 lying in the rst quadrant, we will use parametric
representation to solve this problem. Indeed, we put
109
4 INTEGRATION IN THE COMPLEX PLANE
_
_
_
x = r cos
y = r sin
, r 1,
_
0;

2
_
Then it is easy to get the following integral

2
0

1
0
r
3
cos
2
sin rdrd =

2
0
cos
2
sin d

1
0
r
4
dr
=
_

cos
3

2
0
__
r
5
5

1
0
_
=
1
15
Exercise 4.3. (E. 7 P. 160)
Show that

0,1
0,1
ydx =

2
.The integration is along that portion of the circle x
2
+y
2
= 1 lying
in the half plane x 0. Be sure to consider signs in taking square roots.
Solution.
Because the integration is along that portion of the circle x
2
+y
2
= 1 lying in the half plane
x 0 so we put
_
_
_
x = cos t
y = sin t
_

2
t

2
_
Therefore we have dx = sin tdt.
Thus we consider the integration is

0,1
0,1
ydx =

2

2
sin t (sin t) dt
=
1
2

2

2
(cos 2t 1) dt
=

2
110
4 INTEGRATION IN THE COMPLEX PLANE
4.2 COMPLEX LINE INTEGRATION
We will write down some exercises there such as 4, 5, 6, 7, 8, 10, 13, 14, 17.
Exercise 4.4. (E. 4, E. 5, E. 6 P. 170)
Evaluate

1
i
zdz along the contour C, where C is
1. the straight line segment lying along x + y = 1.
2. the parabola y = (1 x)
2
.
3. the portion of the circle x
2
+ y
2
= 1 in the rst quadrant.
Solution.
1. Now we put
_
_
_
x = t
y = 1 t
where 0 t 1.
And we notice that dz = (1 i) dt. Then by replacing all above, we get

1
i
zdz =

1
0
(t + (t 1) i) (1 i) dt
=

1
0
(2t 1 i) dt
= i
2. Like a method of exercise above, by setting up
_
_
_
x = t
y = (1 t)
2
with 0 t 1.
We obtain dz = [1 2 (1 t) i] dt
Hence, the result is

1
i
zdz =

1
0
_
t (t 1)
2
i
_
[1 2 (1 t) i] dt
=

1
0
_
t + 2 (t 1)
3
+
_
t
2
1
_
i
_
dt
=
2
3
i
3. Here the change of variables is give by
_
_
_
x = cos t
y = sin t
_
0 t

2
_
So we get z = cos t + i sin t. This deduce that dz = (sin t + i cos t) dt.
Thus the result is
111
4 INTEGRATION IN THE COMPLEX PLANE
I =

2
0
(cos t i sin t) (sin t + i cos t) dt
= i

2
0
dt
= i

2
Exercise 4.5. (E. 7 P. 171)
Evaluate

e
z
dz
1. from z = 0 to z = 1 along the line y = 0;
2. from z = 1 to z = 1 + i along the line x = 1;
3. from z = 1 +i to z = 0 along the line y = x. Verify that the sum of your three answers is
zero.
Solution.
1. From z = 0 to z = 1 along the line y = 0, we take x = t and y = 0, so z (t) = t and
dz = dt.
This intergral becomes

1,0
0,0
e
z
dz =

1
0
e
t
dt
= e 1
2. From z = 1 to z = 1 + i along the line x = 1, this time we set x = 1 and y = t so
z = x + iy = 1 + it. Therefore, we get dz = idt.
Now the result is

1,0
1,1
e
z
dz =

1
0
ie
1+it
dt
= ie

1
0
e
it
dt
= e
i+1
e
3. From z = 1 + i to z = 0 along the line y = x, by taking x = y = t and z = t (1 + i), we
obtain dz = (1 + i) dt.
The intergral becomes
112
4 INTEGRATION IN THE COMPLEX PLANE

0,0
1,1
e
z
dz =

0
1
e
t(1+i)
(1 + i) dt
= 1 e
1+i
Note that when we sum three answers, the result is e 1 + e
i+1
e + 1 e
1+i
= 0.
Exercise 4.6. (E. 8 P. 171)
The function z(t) = e
it
= cos t +i sin t can provide a useful parametric representation of circular
arcs. If t ranges from 0 to 2 we have a representation of the whole unit circle, while if t goes
from to we generate an arc extending from e
i
to e
i
on the unit circle. Use this parametric
technique to perform the following intergrations.
1.

1
1
1
z
dz along [z[ = 1, upper half plane.
2.

i
1
z
4
dz along [z[ = 1, rst quadrant.
Solution.
By letting z (t) = cos t + i sin t above, we deduce that dz = (sin t + i cos t) dt. So we will
apply this result to solve 2 problems.
1. Because of upper half plane, it is easy to obtain that

1
1
1
z
dz =


0
(cos t i sin t) (sin t + i cos t) dt
=


0
idt
= i
2. And for the rst quadrant, it follows that

i
1
z
4
dz =

2
0
(cos t i sin t)
4
(sin t + i cos t) dt
=

2
0
(cos 4t i sin 4t) (sin t + i cos t) dt
=

2
0
(sin 3t + i cos 3t) dt
=
1
3
+ i
1
3
113
4 INTEGRATION IN THE COMPLEX PLANE
Exercise 4.7. (E. 13 P. 171)
1. Find a parametric representation of the shorter of the two arcs lying along lying along
(x 1)
2
+ (y 1)
2
= 1 that connects z = 1 with z = i.
Hint: See discussion preceding Exercises 8-10 above, where parametization of a circle
discussed.
2. Find

i
1
zdz along the arc of (a), using the parametrization you have found.
Solution.
1. The parametization of a circle (x 1)
2
+ (y 1)
2
= 1 that connects z = 1 with z = i is
_
_
_
x = 1 + cos t
y = 1 + sin t
_
t

2
_
2. From the above, we have z = 1 + cos t + i (1 + sin t)
And then the following is
dz = (sin t + i cos t) dt
Thus we replace all to get the result.

i
1
zdz =

2

[1 + cos t i (1 + sin t)] (sin t + i cos t) dt


= i
_
2

2
_
Exercise 4.8. (E. 14 P. 171)
Consider I =

2+i
0+i0
e
z
2
dz taken along the line x = 2y. Without actually doing the integration,
show that [I[

5e
3
.
Solution.
We need to nd M that is

e
z
2

M, so M = max
_
e
x
2
y
2
_
when x : 0 2 and y : 0 1.
Thus we have M = e
3
.
Next, we must nd L. Because this path in this exercise is a line so the length of it is
L =
_
(2 0)
2
+ (1 0)
2
=

5.
Finally, we have the result [I[ ML = e
3

5
Hence, we complete the proof.
114
4 INTEGRATION IN THE COMPLEX PLANE
Exercise 4.9. (E. 17 P. 171)
1. Let g (t) be a complex function of the real variable t. Express

b
a
g (t) dt as the limit of a
sum. Show that for b > a we have

b
a
g (t) dt

b
a
[g (t)[ dt
2. Use the equation above to prove that

1
0

te
it
dt

2
3
Solution.
1. We express the integral that

b
a
g (t) dt = lim
n
n

k=1
[u(x(t
k
) , y (t
k
)) + iv (x(t
k
) , y (t
k
))] [x(t
k
) + iy (t
k
)]
Then we have the inequality

b
a
g (t) dt

lim
n
n

k=1
[u(x(t
k
) , y (t
k
)) + iv (x(t
k
) , y (t
k
))[ [x(t
k
) + iy (t
k
)]

b
a
[g (t)[ dt
2. By using the equation above, we can complete the proof easily. Indeed, we consider

1
0

te
it
dt

1
0

te
it

dt
=

1
0

tdt =
2
3
115
4 INTEGRATION IN THE COMPLEX PLANE
4.3 CONTOUR INTEGRATION AND GREENS THEOREM
In this section, the examples is exercises 1, 2, 12, 13, 16, 18.
Exercise 4.10. (E. 1 P. 180)
1. Let C be an arbitrary simple closed contour. Use Greens theorem to find a simple inter-
pretation of the line integral
1
2

C
(ydx + xdy) .
2. Consider

C
[cos ydx + sin xdy] performed around the square with corners at
(0, 0) , (1, 0) , (0, 1) , (1, 1). Evaluate this integral by doing an equivalent integral over the
area enclosed by the square.
3. Suppose you know the area enclosed by a simple closed contour C. Show with the aid of
Greens theorem that you can easily evaluate

C
zdz around C.
Solution.
1. Applying Greens theorem, we have
1
2

C
(ydx + xdy) =
1
2

R
(1 + 1) dxdy
= [R[
2.

C
[cos ydx + sin xdy] =

R
_
(sin x)
x

(cos y)
y
_
dxdy
=

1
0

1
0
(cos x + sin y) dxdy
= sin 1 cos 1 + 1
3. Put z = x + iy, so z = x iy and applying Greens theorem for

C
zdz, we get
116
4 INTEGRATION IN THE COMPLEX PLANE

C
zdz =

C
(x iy) (dx + idy)
=

C
(x iy) dx + (y + ix) dy
=

R
_
(y + ix)
x

(x iy)
y
_
dxdy
= 2i

R
dxdy
= 2i [R[
Exercise 4.11. (E. 2 P. 180)
To which of the following integral is the Cauchy-Goursat theorem directly applicable?

|z|=1
sin z
z + 2i
dz
Solution.
By setting up z = x + iy, this integral becomes

|x+iy|=1
sin (x + iy)
x + iy + 2i
(dx + idy)
Let f (x, y) =
sin (x + iy)
x + iy + 2i
, we note that the function f (x, y) do not define at the point
(0, 2), but (0, 2) / D with D =
_
(x, y) [x
2
+ y
2
1
_
Therefore, f is analytic and we can deduce that the integral is the Cauchy-Goursat theorem
directly applicable.
Exercise 4.12. (E. 12, E. 13 P. 180)
Prove the following result by means of Cauchy-Gorsat theorem. Begin with

e
z
dz per-formed
around [z[ = 1. Use the parametric representation z = e
i
, 0 2. Separate your equation
into real and imaginary parts.
1.

2
0
e
cos
[cos (sin + )] d = 0
2.

2
0
e
cos
[sin (sin + )] d = 0
Solution.
By using the parametric representation z = e
i
, we have dz = ie
i
d. Hence,
117
4 INTEGRATION IN THE COMPLEX PLANE

e
z
dz =

2
0
e
e
i
ie
i
d
= i

2
0
e
cos +i sin
(cos + i sin )d
=

2
0
e
cos
[cos (sin ) + i sin (sin )] (sin + i cos )d
=

2
0
e
cos
[sin (sin + )] d + i

2
0
e
cos
[cos (sin + )] d
According to Cauchy-Gorsat theorem

e
z
dz = 0 performed around [z[ = 1. Therefore,

2
0
e
cos
[sin (sin + )] d = 0

2
0
e
cos
[cos (sin + )] d = 0
Exercise 4.13. (E. 16 P. 181)
Show that for real a, where [a[ > 1, we have

2
0
1 a cos
1 2a cos + a
2
d = 0
Solution.
We consider the integral

1
z a
dz, where the integral is taken around the unit circle. Now
we use the parametric representation like exercise 12, 13 above. Indeed, if we put z = e
i
, we
will have dz = ie
i
d. Hence,

1
z a
dz =

2
0
ie
i
d
e
i
a
=

2
0
i (cos + i sin ) (cos a i sin )
(cos a)
2
+ sin
2

d
=

2
0
a sin
1 2a cos + a
2
d + i

2
0
1 a cos
1 2a cos + a
2
d
According to Cauchy-Gorsat theorem

1
z a
z = 0 performed around [z[ = 1. Therefore, we
get the proof completely.
118
4 INTEGRATION IN THE COMPLEX PLANE
Exercise 4.14. (E. 18 P. 181)
Evaluate the following integral. The contour is the square centered at the origin with corners
at (2 2i) (C
1
) . The result contained in the previous problem as well as the principle of
deformation of contours will be useful.

C
1
dz
z i
Solution.
We let f (z) =
1
z i
and consider (C
2
) : [z[ =
3
2
. Since a function f (z) is a analytic not only
on C
1
and C
2
but also all points lying between C
1
and C
2
so

C
1
f (z) dz =

C
2
f (z) dz
According to the following equation, with z
0
an arbitrary complex constant,

|zz
0
|=r
(z z
0
)
n
dz =
_
_
_
0, n ,= 1
2i, n = 1
where the contour of integration is a circle centered at z
0
.
We have

C
2
f (z) dz = 2i
Thus this means that

C
1
dz
z i
= 2i
119
4 INTEGRATION IN THE COMPLEX PLANE
4.4 PATH INDEPENDENCE, INDEFINITE INTEGRALS,
FUNDAMENTAL THEOREM OF CALCULUS IN THE
COMPLEX PLANE
In this section, we have some exercises to deal with, these are exercise 2, 3, 5, 7, 8.
Exercise 4.15. (E. 2, E. 3, E. 5, E. 7 P. 190)
Use theorem (Integration of Functions that are the Derivatives of Analtyic Functions) to evaluate
the following integrals along the curve y =

x.
1.
4+2i

0
e
iz
dz
2.
4+2i

0
_
1 + z
2
_
dz
3.
4+2i

0
e
z
sinh zdz
4.
4+2i

1+i
z
z
2
1
dz
Solution.
1. With
d
_
e
iz
_
dz
= ie
iz
, we can compute this integral as follows.
4+2i

0
e
iz
dz = i
4+2i

0
ie
iz
dz
= i. e
iz

4+2i
0
= i
_
e
2+4i
1
_
= i ie
2+4i
2. With
d
_
z +
z
3
3
_
dz
= 1 + z
2
, we can evaluate this integral.
120
4 INTEGRATION IN THE COMPLEX PLANE
4+2i

0
_
1 + z
2
_
dz =
_
z +
z
3
3
_

4+2i
0
= 4 + 2i +
(4 + 2i)
3
3
= 4 + 2i +
16 + 88i
3
=
28 + 94i
3
3. With
d
_
e
2z
2z
_
dz
= 2e
2z
2, like two exercises above, we have
4+2i

0
e
z
sinh zdz =
4+2i

0
e
z
e
z
e
z
2
dz
=
4+2i

0
e
2z
1
2
dz
=
1
4
4+2i

0
_
2e
2z
2
_
dz
=
1
4
_
e
2z
2z
_

4+2i
0
=
1
4
_
e
8+4i
9 4i
_
4. With
d
_
Log

z
2
1

_
dz
=
d1
z
2
1
, we obtain
4+2i

1+i
z
z
2
1
dz =
1
2
4+2i

1+i
2z
z
2
1
dz
=
1
2
4+2i

1+i
d
_
z
2
1
_
z
2
1
=
1
2
_
Log

z
2
1

4+2i
1+i
=
1
2
_
Log

(4 + 2i)
2
1

Log

(1 + i)
2
1

_
=
1
2
(Log [11 + 16i[ Log [2i 1[)
=
1
2
Log

11 + 16i
1 + 2i

=
1
2
Log

21 38i
5

121
4 INTEGRATION IN THE COMPLEX PLANE
Exercise 4.16. (E. 8 P. 190)
1. What, if anything, is incorrect about the following two integrations? The integrals are
both along the line y = x.
1+i

0+0i
zdz =
z
2
2

1+i
0+0i
=
(1 + i)
2
2
= i ()
1+i

0+0i
zdz =
z
2
2

1+i
0+0i
=
(1 i)
2
2
= i ()
2. What is the correct numerical value of each of the above integrals?
Solution.
The integrals (*),(**) are incorrect since with z = x + iy and seeing Fig.(a), Fig(b) above,
we have
122
4 INTEGRATION IN THE COMPLEX PLANE

C
zdz =

I
zdz +

II
zdz
=
1

0
xdx +
1

0
(1 + yi) dy
=
1

0
xdx +
1

0
dy + i
1

0
ydy
=
1
2
+ 1 +
i
2
=
3
2
+
i
2

C
zdz =

I
zdz +

II
zdz
=
1

0
xdx +
1

0
(1 yi) dy
=
1

0
xdx +
1

0
dy i
1

0
ydy
=
1
2
+ 1
i
2
=
3
2

i
2
123
4 INTEGRATION IN THE COMPLEX PLANE
4.5 THE CAUCHY INTEGRAL FORMULA AND ITS EXTENSION
We go to see together some exercises such as 2, 6, 17, 19.
Exercise 4.17. (E. 2, E. 6 P. 199)
Evaluate the following integrals using the Cauchy integral formula, its extension, or the Cauchy-
Goursat theorem where appropriate.
1.

sin z
z 2
dz around [z[ = 3.
2.

e
iz
z
2
+ z + 1
dz around

z +
1
2
2i

= 2.
Solution.
1. We can see that the term z 2 does become 0 at the point z = 2 inside C is the circle
[z[ = 3. We therefore apply Cauchy integral formula with f (z) = sin z. Thus

sin z
z 2
dz = 2if (2) = 2i sin 2
2. We need to solve the equation z
2
+ z + 1 = 0, then we get the roots are z =
1 i

3
2
.
And the next is that we need to denite the point makes the denominator becomes 0 such
that inside C is the circle

z +
1
2
2i

= 2.
We consider that

1 + i

3
2
+
1
2
2i

i
_

3
2
2
_

< 2
So this is the point we nd and we can verify the others is outside C.
Then we apply Cauchy integral formula with f (z) =
e
iz
z +
1+i

3
2
. Thus

e
iz
z
2
+ z + 1
dz = 2if
_
1 + i

3
2
_
=
2

3e
i
2
+

3
2
Exercise 4.18. (E. 17 P. 200)
If a is a real number and [a[ < 1 show that

2
0
1 a cos
1 2a cos + a
2
d = 2
Solution.
We consider

dz
z a
around [z[ = 1 and because of [a[ < 1, the point which makes the
denominator becomes 0 is inside C is the circle [z[ = 1. Hence, we apply Cauchy integral
124
4 INTEGRATION IN THE COMPLEX PLANE
formula with f (z) = 1 for all z inside C, we have

dz
z a
= 2i
And in the section 4.3, by putting parametric representation, we have proved that

1
z a
dz =

2
0
ie
i
d
e
i
a
=

2
0
i (cos + i sin ) (cos a i sin )
(cos a)
2
+ sin
2

d
=

2
0
a sin
1 2a cos + a
2
d + i

2
0
1 a cos
1 2a cos + a
2
d
At this time, we can deduce that

2
0
1 a cos
1 2a cos + a
2
d = 2
Exercise 4.19. (E. 19 P. 201)
Let f (z) be analytic on and inside a simple closed contour C. Let z
1
and z
2
lie inside C (see
gure below). Show that
1
2i

C
f (z)
(z z
1
) (z z
2
)
dz =
f (z
1
)
z
1
z
2
+
f (z
2
)
z
2
z
1
Solution.
We note that
f (z)
(z z
1
) (z z
2
)
=
f (z)
z
2
z
1
_
1
z z
2

1
z z
1
_
Then we have two integrals. These are

C
f (z)
(z z
1
) (z z
2
)
dz =
1
z
2
z
1
_
C
2
f (z)
z z
2
dz

C
1
f (z)
z z
1
dz
_
125
4 INTEGRATION IN THE COMPLEX PLANE
We see in the gure above and consider the rst integral, at the point z
2
which makes the
denominator becomes 0 is inside C
2
so we can apply Cauchy integral formula there. And it is
similar for the case of z
1
. Thus we obtain

C
2
f (z)
z z
2
dz

C
1
f (z)
z z
1
dz = 2i [f (z
2
) f (z
1
)]
Hence, this is really the proof we want to solve.
1
2i

C
f (z)
(z z
1
) (z z
2
)
dz =
1
z
2
z
1
[f (z
2
) f (z
1
)]
=
f (z
1
)
z
1
z
2
+
f (z
2
)
z
2
z
1
126
4 INTEGRATION IN THE COMPLEX PLANE
4.6 SOME APPLICATIONS OF THE CAUCHY INTEGRAL
FORMULA
In this section, we get the exercises 1, 3, 5, 8, 9, 10, 13, 15 to show.
Exercise 4.20. (E. 1, E. 3, E. 5 P. 211)
Use Gauss mean value theorem in its various versions and integrations around appropriate
circles to prove the following:
1.
1
2
2

0
e
e
i
d = 1
2.
1
2

cos
2
_

6
+ ae
i
_
d =
3
4
, where a > 0
3.
2

0
Log
_
a
2
+ 1 + 2a cos (n)

d = 4Log a where a > 1, n integer


Solution.
1. Let f (z) = e
z
Since function f is analytic on C, applying Gauss mean value theorem we have
1
2
2

0
e
e
i
d =
1
2
2

0
f
_
0 + e
i
_
d where r = 1
= f (0)
= 1
2. Let f (z) = cos
2
(z), like the exericise above :
1
2

cos
2
_

6
+ ae
i
_
d =
1
2

f
_

6
+ ae
i
_
d
= f
_

6
_
= cos
2
_

6
_
=
3
4
3. We have Log
_
a
2
+ 1 + 2a cos (n)

= Log

a + e
in

2
completely.
Let f (z) = 2Log [z[, since function f is analytic on D = z : [z[ , = 0, applying Gauss
mean value theorem with z =

a + e
in

D
127
4 INTEGRATION IN THE COMPLEX PLANE
2

0
Log
_
a
2
+ 1 + 2a cos (n)

d =
2

0
Log

a + e
in

2
d
=
2

0
f
_

a + e
in

_
d
= 22Log a
= 4Log a
Exercise 4.21. (E. 8 P. 212)
Let f (z)be a nonconstant function that is continuous and nonzero throughout a closed bounded
region R. Let f (z) be analytic at every interior point of R. Show that the minimum value of
[f (z)[ in R must occur on the boundary of R.
Solution.
We consider the function g (z) =
1
f (z)
. So we can get the comment is f (z) has a minimum
if and only if g (z) has a maximum.
Since f (z)is a nonconstant function that is continuous and nonzero throughout a closed
bounded region R and f (z) is analytic at every interior point of R so g(z)f (z)is a nonconstant
function that is continuous and nonzero throughout a closed bounded region R and analytic at
every interior point of R.
Therefore, apllying Maximum Modulus Theorem we have the maximum value of [g (z)[ in R
must occur on the boundary of R.
Thus, that the minimum value of [f (z)[ in R must occur on the boundary of R.
Exercise 4.22. (E. 9, E. 10 P. 212)
For the following closed regions R and functions f (z), nd the values of z in R where [f (z)[
archieves its maximum and minimum values. If your answers do not lie on the boundary of R,
give an explanation. Give the values of [f (z)[at its maximum and minimum in R.
1. f (z) = z, R is [z 1 i[ 1.
2. f (z) = z
2
, R is [z 1 i[ 2.
Solution.
1. Because f (z) = z is analytic in R, module of f (z), [f (z)[, archieves maximum on
[z 1 i[ = 1. We put z = 1 + i + e
i
, 0 2, then we obtain
f(z) = 1 + i + e
i
= 1 + i + cos + i sin
= 1 + cos + (1 + sin )i
128
4 INTEGRATION IN THE COMPLEX PLANE
Moreover, module of this function is
[f(z)[ =
_
(1 + cos )
2
+ (1 + sin )
2
=

3 + 2 cos + 2 sin
=
_
3 + 2

2 cos
_

4
+
_
Next, we consider the function g() = 3 + 2

2 cos
_

4
+
_
, 0 2, we get
cos
_

4
+
_
1 easily. Then we can deduce that g() 3 + 2

2. At this time,
we conclude [f(z)[
max
= 3 +2

2 at the point is =

4
and the function is z = 1 +i +e
i

4
.
Assuming that f(z) = 0, we have the contradiction is [ 1 i[ 1 and get f(z) ,= 0.
Thus [f (z)[ has a minimum on [z 1 i[ = 1. Like the maximum, we obtain [f(z)[
min
=
_
3 2

2 at the point =
5
4
and the function is z = 1 + i + e
i
5
4
.
2. We have f (z) = 0 if and only if z = 0 and this point is in [z i 1[ 2 by considering.
So [f (z)[
min
= 0 at z = 0. Then since f (z) is analytic in R, [f(z)[ has a minimum on
[z i 1[ = 2. We will set up
z = i + 1 + 2e
i
, [, ]
= i + 1 + 2 cos + 2i sin
Evaluating the module of f (z) follows
[z[
2
= (1 + 2 cos )
2
+ (1 + 2 sin )
2
= 6 + 4

2 cos
_

4
+
_
6 + 4

2
Then [f(z)[
max
= 6+4

2 at the point =

4
and the function is z = (1+

2) +i(1+

2).
Exercise 4.23. (E. 13 P. 212)
Let u(x, y) be real, nonconstant and continuous in a closed bounded region R . Let u(x, y) ba
harmonic in the interior of R . Prove that the maximum value of u(x, y)in the region occurs on
the boundary. This is known as the maximum priciple.
Solution.
Consider F (x, y) = u(x, y) + iv (x, y), where v is the harmonic conjugate of u.
Let f (z) = e
F(z)
, we have f (z) is an entire function and is never 0 in R ,and f (z) continuous
in closed bounded region R ( because u(x, y)continuous in a closed bounded region R) so the
minimum value of [f (z)[ in R must occur on the boundary of R.
129
4 INTEGRATION IN THE COMPLEX PLANE
With [f (z)[ =

e
F(z)

e
u(x,y)+iv(x,y)

= e
u(x,y)
, we say that [f (z)[ has a maximum if and
only if u(x, y) has a maximum too.
Thus, the maximum value of u(x, y)in the region occurs on the boundary.
Exercise 4.24. (E. 15 P. 212)
Consider the closed bounded region R given by 0 x 1, 0 y 1. Now u = x
2
y
2
is
harmonic in R. Find the maximum and minimum values of u in R and state where they are
achieved.
Solution.
In fact, the function u(x, y) = x
2
y
2
is harmonic then have maximum and minimum on
boundary of R. And we can see that
_
_
_
x
2
y
2
x
2
1
x
2
y
2
y
2
1
It means that 1 u(x, y) 1 for all x, y in R. We go on to note that on boundary of R,
the unique point (1, 0) makes u = 1 and the same is (0, 1) which makes u = 1.
Then we obtain the result above.
130
5 INFINITE SERIES INVOLVING A
COMPLEX VARIABLE
5.1 INTRODUCTION AND REVIEW OF REAL SERIES
We will solve exercises 1, 3, 4, 10, 11, 16, 17, 18, 20, 21.
Exercise 5.1. (E. 3, E. 3, E. 4 P. 231)
Use Eq (5.1-4) to obtain the rst nonzero terms in the Taylor series expansions of the following
real functions. Give a formula for the general or nth term.
1.
1
1 x
expanded about x = 0
2.
1
1 x
expanded about x = 1
3.

x expanded about x = 1
Solution.
1. We have
c
0
=
f (0)
0!
= 1
c
1
=
f

(0)
1!
= 1
c
2
=
f

(0)
2!
= 1
c
3
=
f

(0)
3!
= 1
We can deduce that c
n
= 1 by using induction.
2. Apply Taylor series expansions, we get
f (z) =

n=0
c
n
(x x
0
)
n
=

n=0
c
n
(x + 1)
n
Then, we list some coecients as follow
131
5 INFINITE SERIES INVOLVING A COMPLEX VARIABLE
c
0
= f (1) =
1
2
c
1
= f

(1) =
1
2
2
c
2
=
f

(1)
2!
=
1
2
3
c
3
=
f

(1)
3!
=
1
2
4
So that the general coecient is c
n
=
1
2
n+1
.
3. In a similar fashion, we observe that
f

(x) =
1
2

x
f

(x) =
1
4x

x
f

(x) =
3
8x
2

x
Then we have the respectively coecients are c
0
= 1, c
1
=
1
2
, c
2
=
1
8
, c
3
=
1
16
. So the
general is c
n
= (1)
n+1
1
2
n
, n ,= 0.
Exercise 5.2. (E. 10, E. 11 P. 231)
Study the ratio test and the notion of absolute convergence in a book on elementary calculus
and show that the following real series are absolutely convergent or are divergent in the intervals
specied.
1.

n=1
2
n
x
n
n
abs.conv. for [x[ <
1
2
, div. for [x[ >
1
2
2.

n=1
sinh n
e
n
(x + 1)
n
abs.conv. for 2 < x < 0, div. for x < 2 and x > 0
Solution.
1. We see the ratio test for this exercise as follows

u
n+1
u
n

=
2 [x[ n
n + 1
Hence,
lim
n

u
n+1
u
n

= lim
n
2 [x[ n
n + 1
= 2 [x[
In order to this series are absolutely convergent, we need
132
5 INFINITE SERIES INVOLVING A COMPLEX VARIABLE
lim
n

u
n+1
u
n

= 2 [x[ < 1
It means that the proof we nd. And the divergent is created simiilarly.
2. Also, we consider the ratio test below.

sinh (n + 1)
e
n+1
(x + 1)
n+1
sinh n
e
n
(x + 1)
n

=
_
1 e
22
_
(x + 1)
1 e
2
Thus if we want to solve the absolutely convergent, we must have
lim
n

_
1 e
22
_
(x + 1)
1 e
2

= [x + 1[ < 1
It means that 2 < x < 0 and vice versa for the interval of divergent.
Exercise 5.3. (E. 16, E. 17 P. 232)
Recall the comparison test for convergence, from elementary calculus : If a series of positive
constants

n=1
c
n
is known to converge, and if one is given a series

n=1
u
n
(x), where 0 u
n
(x)
c
n
, then the latter series converges. Assume that we know that the p-series

n=1
1
n
p
converges,
where p > 1, to show the convergence of the following. Use the positive value of any quantity
that appears to be multivalued.
1.

n=1
cos
2
nx
n
3/2
for < x <
2.

n=1
tanh nx
n
1.1
for < x <
3.

n=1
1
n
1+nx
for x > 0
Solution.
1. We have an inequality below.

cos
2
nx
n
3/2

1
n
3/2
And see that the series

n=1
1
n
3/2
converges, so the series given converges.
2. In a similar way, we get
133
5 INFINITE SERIES INVOLVING A COMPLEX VARIABLE

n=1
tanh nx
n
1.1

n=1
1
n
1.1
And the series on the right side converges, the series given thus converges.
3. Because of 1 + nx > 1, there exists > 0 such that 1 + nx 1 + > 1. Then
1
n
1+nx

1
n
1+
And the series on the right side converges, the series given thus converges.
Exercise 5.4. (E. 20, E. 21 P. 232)
Recall the comparison test for divergence. If a series of positive constants

n=1
c
n
is known to
diverge, and if one is given a series

n=1
u
n
(x) where 0 < c
n
u
n
(x), then the latter series
diverges. Assuming that we know that the p-series diverges if p 1, prove the following series
diverge where indicated.
1.

n=1
1 + cos
2
nx

n
for < x <
2.

n=1
coth nx
n
for [x[ > 0
Solution.
1. We consider the inequality.
1

n

1 + cos
2
nx

n
But the series on the left side diverges, hence the series given too.
2. Because of [x[ > 0, we have [coth nx[ 1. Then
1
n

coth nx
n

But the series on the left side diverges, hence the series given too.
134
5 INFINITE SERIES INVOLVING A COMPLEX VARIABLE
5.2 COMPLEX SEQUENCES AND CONVERGENCE OF
COMPLEX SERIES
We have exercises 4, 5, 6, 8, 9, 10, 11.
Exercise 5.5. (E. 4, E. 5, E. 6 P. 241)
Use the nth term test to prove that the following series are divergent in the indicated regions.
1.

n=0
(n + 1) (i + 1)
n
(z + 1)
n
for [z + 1[
1

2
2.

n=2
n(i 1)
n
(z 2i)
n
for [z 2i[

2
3.

n=1
_
2n + 2
n
_
n
(z + 1 + i)
n
for [z + 1 + i[
1
2
Solution.
1. We have
[u
n
(z) [ = [ (n + 1) (i + 1)
n
(z + 1)
n
[
= [n + 1[.[i + 1[
n
.[z + 1[
n
= (n + 1)
_

2
_
n
[z + 1[
n
If [z + 1[ =
1

2
, then
_
2
_
n
[z + 1[
n
1 when n .
Thus the series is divergent. And for the case of [z + 1[ >
1

2
, we still have too.
2. We get
[u
n
(z) [ =

n(i 1)
n
(z 2i)
n

=
[n(i 1)[
n
[(z 2i)[
n
=
n
n

2
n
[z 2i[
n
We consider a sequence

2
n
[z 2i[
n
because of n
n
. If [z2i[ =

2, you have

2
n
[z 2i[
n
=
1 when n . So that the series is divergent.
3. We obtain that
135
5 INFINITE SERIES INVOLVING A COMPLEX VARIABLE
[u
n
(z) [ =

_
2n + 2
n
_
n
(z + 1 + i)
n

=
_
2n + 2
n
_
n
[z + 1 + i[
n
= 2
n
_
1 +
1
n
_
n
[z + 1 + i[
n
Moreover, we remember that
_
1 +
1
n
_
n
e, n
And if [z + 1 + i[ =
1
2
, then 2
n
[z + 1 + i[
n
= 1. Hence,
[u
n
(z) [ =
_
1 +
1
n
_
n
e ,= 0
The proof is completed.
Exercise 5.6. (E. 8, E. 9, E. 10 P. 241)
Use the ratio test to prove the absolute convergence, in the indicated domains, of the following
series. Where does the ratio test assert that each series diverges?
1.

n=0
n!e
n
2
z
for 1z < 0
2.

n=0
(2 + i)
n
(z + i)
n
(n + i)
2
for [z + i[ >

5
3.

n=0
1
n!
_
n
z
_
n
for [z[ > e
Solution.
1. We have

u
n+1
u
n

(n + 1)!e
(n+1)
2
z
n!e
n
2
z

(n + 1) e
(2n+1)z

= (n + 1) e
(2n+1)x
For the case of x < 0, we see that

u
n+1
u
n

= (n + 1) e
(2n+1)x
0 < 1
136
5 INFINITE SERIES INVOLVING A COMPLEX VARIABLE
So the series is convergent.
For the case of x 0, we consider that

u
n+1
u
n

= (n + 1) e
(2n+1)x
> 1
Hence, the series is divergent.
2. In a similar fashion, we take the ratio test here.

u
n+1
u
n

5
_
n
2
+ 1
_
[z + i[ (n
2
+ 2n + 2)
And if we make n approachs to , the limit of this will depend on
lim
n

u
n+1
u
n

5
[z + i[
At this time, we have a notice.
For the case of [z + i[ >

5, the series is convergent.


For the case of [z + i[ <

5, the series is divergent.


For the case of [z + i[ =

5, we cannot dene the convergence of this series.


3. We get

u
n+1
u
n

=
_
n + 1
n
_
n
1
[z[
Then
lim
n

u
n+1
u
n

=
1
[z[
lim
n
_
n + 1
n
_
n
=
e
[z[
At this time, we still have 3 cases
If [z[ > e, the series is convergent.
If [z[ < e, the series is divergent,
If [z[ = e, we cannot define the convergence of this series.
Exercise 5.7. (E. 11 P. 242)
Make the substitution z = e
i
in Eq. (5.2-9), N = n 1, assume is real, and separate Eq.
(5.2-9) into its real and imaginary parts to show that
1. 1 + cos + cos 2 + ... + cos (N) = cos [N/2]
sin [(N + 1) /2]
sin [/2]
2. sin + sin 2 + ... + sin (N) = sin [N/2]
sin [(N + 1) /2]
sin [/2]
137
5 INFINITE SERIES INVOLVING A COMPLEX VARIABLE
Solution.
We have
1 + z + z
2
+ ... + z
n1
= 1 + e
i
+ e
2i
+ ... + e
iN
= 1 + cos + i sin + cos 2 + i sin 2 + ... + cos (N) + i sin (N)
= (1 + cos + cos 2 + ... + cos (N)) + i (sin + sin 2 + ... + sin (N))
So that
1 z
n
1 z
=
1 e
i(N+1)
1 e
i
=
1 cos (N + 1) i sin (N + 1)
1 cos i sin
=
2 sin
2
_
(N + 1)
2
_
i2 sin
_
(N + 1)
2
_
cos
_
(N + 1)
2
_
2 sin
_

2
__
sin
_

2
_
i cos
_

2
__
=
2 sin
_
(N + 1)
2
_ _
sin
_
(N + 1)
2
_
i cos
_
(N + 1)
2
__
2 sin
_

2
__
sin
_

2
_
i cos
_

2
__
=
sin
_
(N + 1)
2
_ _
sin
_
(N + 1)
2
_
i cos
_
(N + 1)
2
___
sin
_

2
_
i cos
_

2
__
sin
_

2
_
=
sin
_
(N + 1)
2
_ _
cos
_
N
2
_
i sin
_
N
2
__
sin
_

2
_
Because of
1 z
n
1 z
= 1 + z + z
2
+ ... + z
n1
, we deduce that
sin
_
(N + 1)
2
_ _
cos
_
N
2
_
i sin
_
N
2
__
sin
_

2
_ = (1 + cos + cos 2 + ... + cos (N))+i (sin + sin 2 + ... + sin (N))
Hence,
sin
_
(N + 1)
2
_
cos
_
N
2
_
sin
_

2
_ = 1 + cos + cos 2 + ... + cos (N)
138
5 INFINITE SERIES INVOLVING A COMPLEX VARIABLE
sin
_
(N + 1)
2
_
sin
_
N
2
_
sin
_

2
_ = (sin + sin 2 + ... + sin (N))
139
5 INFINITE SERIES INVOLVING A COMPLEX VARIABLE
5.3 UNIFORM CONVERGENCE OF SERIES
In this section, we face with some exercises such as 1, 2, 3, 4, 6, 8.
Use the Weierstrass M test to establish the uniform convergence of the following series in the
indicated regions. State the convergent series of constants that is employed.
1.

j=1
(z)
j1
for [z[ 0.999
2.

j=1
j
j + 1
(z)
j
for [z[ r, where r < 1
3.

j=1
z
j
j!
for [z[ r, where r <
4.

n=1
[n i[
n
3
z
n
for [z[ r, where r < 1
Solution.
1. Apply Leibnitzs theorem for alternating series, we have (0.999)
j1
is a sequence reduced
and approachs to 0. Hence

j=1
(1)
j1
(0.999)
j1
is converging. Then, we put M
j
= (0.999)
j1
and obtain that
[u
j
[ M
j
So that this series is uniformly converging.
2. We consider the series

j=0
(z)
j
and easily prove that it converges because of the con-
verging series

j=0
r
j
with r < 1. And then, according to Weierstrass theorem, we obtain
the series given is uniformly converging.
3. Because of [z[ r, we see that
[z[
j
j!

r
j
j!
And

j=0
r
j
j!
= e
r
is converging. So that the series given is uniformly converging.
4. We observe the inequality below.
140
5 INFINITE SERIES INVOLVING A COMPLEX VARIABLE

[n i[
n
3
z
n

n
2
+ 1
n
3
[z[
n

n
2
+ 1
n
3
r
n

n
2
+ 1
n
3

2
n
2
And the series

n=1

2
n
2
is converging. Hence, the given series is converging uniformly.
Exercise 5.8. (E. 6 P. 247)
In this problem, we prove the Weierstrass M test. We are given a series

j=1
u
j
(z) whose sum
is S (z) when z lies in a region R. We have also at our disposal a convergent series of constants

j=1
M
j
such that throughout R we have [u
j
(z)[ M
j
and wish to show this guarantees uniform
convergence of the original series as well as absolute convergence.
1. Using the comparison test from real calculus, explain why the series

j=1
u
j
(z) must be
absolutely convergent. Recall that absolute convergence guarantees ordinary convergence.
2. Using the denition of convergence explain why
[S (z) S
n
(z)[ =

lim
k
k

j=1
u
j
(z)
n

j=1
u
j
(z)

lim
k
k

j=n+1
u
j
(z)

if we take k > n.
3. Explain why

lim
k
k

j=n+1
u
j
(z)

lim
k
k

j=n+1
M
j
4. Prove that given > 0 there must exist a number N such that lim
k
k

j=n+1
M
j
< for
n > N.
Solution.
141
5 INFINITE SERIES INVOLVING A COMPLEX VARIABLE
1. With the converging series

j=1
M
j
, we have for all z in R, M
j
> 0
[u
j
(z)[ M
j
In contrast, we see that

j=1
[u
j
(z)[ = [u
1
(z)[ + ... +[u
n
(z)[ + ...
M
1
+ ... + M
n
+ ...
=

j=1
M
j
Hence, the series

j=1
[u
j
(z)[ is converging and then

j=1
u
j
(z) is absolutely converging.
2. We get
S (z) =

j=1
u
j
(z) = lim
k
k

j=1
u
j
(z)
S
n
(z) =
n

j=1
u
j
(z)
So that we take a subtraction below.
S (z) S
n
(z) =

j=n+1
u
j
(z)
= lim
k
k

j=n+1
u
j
(z) , k > n
3. We obtain the inequality below.

j=n+1
u
j
(z)

j=n+1
[u
j
(z)[
=

j=n+1
M
j
Thus we deduce that
142
5 INFINITE SERIES INVOLVING A COMPLEX VARIABLE

lim
k
k

j=n+1
u
j
(z)

lim
k
k

j=n+1
M
j
4. With M =

j=1
M
j
, we create a sequence M
n
=
n

j=1
M
j
and obtain obviously that
M M
n
=

j=1
M
j

n

j=1
M
j
=

j=n+1
M
j
At this time, M
n
approachs to M if for all > 0, there exists a positive integer N () such
that
[M M
n
[ < , n > N
This is equivalent to

j=n+1
M
j

< , n > N
Or we can say that
lim
k
k

j=n+1
M
j
<
Exercise 5.9. (E. 8 P. 248)
In this exercise, we show that the series

j=1
z
j1
converges uniformly to 1/ (1 z) in the disc
[z[ r, where r < 1. The proof requires that we obtain a value for N satisfying the denition
of uniform convergence.
1. Explain why Log (1/ [z[) Log (1/r) in the disc.
2. Prove 1/ [1 z[ 1/ (1 r) and Log [1/ ( [1 z[)] Log [1/ ( (1 r))] for [z[ r. Take
> 0.
3. Assume that 0 < < 1/2. Show that in the disc [z[ r we have
Log
1
[1 z[
Log
1
[z[

Log
1
(1 r)
Log
1
r
143
5 INFINITE SERIES INVOLVING A COMPLEX VARIABLE
Solution.
1. It is easy to see that
1
[z[

1
r
, thus we get the inequality need to explain.
2. In a similar fashion, we can observe carefully the inequality
[1 z[ 1 z 1 r
So that the next is
1
[1 z[

1
(1 r)
Hence, the proof is shown completely.
3. We have had 2 inequalities above as follows
Log
1
[1 z[
Log
1
(1 r)
1
Log
_
1
[z[
_
1
Log
_
1
r
_
Because of < 1/2, we obtain the proof.
144
5 INFINITE SERIES INVOLVING A COMPLEX VARIABLE
5.4 POWER SERIES AND TAYLOR SERIES
We get excercises 27, 28, 29, 30, 31, 32, 33.
Exercise 5.10. (E. 27 P. 261)
Use the equation below and triangle inequality to prove that in the disc [z[ 1 we have
[e
z
1[ (e 1) [z[.
e
z
=

n=0
1
n!
z
n
Solution.
By using the given equation, we have
[e
z
1[ =

n=1
1
n!
z
n

Because of [z[ 1, we can easily deduce that [z


n
[ [z[for all n.
Hence, apply that inequality, we get
[e
z
1[ =

n=1
1
n!
z
n

n=1
1
n!

[z[
= (e 1) [z[
So this is the inequality we need to nd.
Exercise 5.11. (E. 28 P. 261)
1. Let z
N
z
N
0
=
N

n=1
c
n
(z z
0
)
n
valid for all z. N is a positive integer. Show that c
n
=
N!z
Nn
0
/ [n! (N n)!].
2. Replace z in the above with z + z
0
and show that
(z + z
0
)
N
=
N

n=0
N!z
Nn
0
n! (N n)!
z
n
This is the familiar binomial expansion.
Solution.
1. We have z
N
z
N
0
=
N

n=1
c
n
(z z
0
)
n
dene for all z where N is a positive integer
From Talor series formular, we need to evaluate the rst and the second coecients.
c
1
=
f
(1)
(z
0
)
1!
= Nz
N1
0
145
5 INFINITE SERIES INVOLVING A COMPLEX VARIABLE
c
2
=
f
(2)
(z
0
)
2!
=
N (N 1) z
N2
0
2!
So we can easily see that c
n
=
N!
n! (N n)!
z
Nn
0
by using mathematical induction.
2. By replacing z in the above with z+z
0
, the function f (z) becomes f (z) = (z + z
0
)
N
z
N
0
.
We will do the same as above. First we have
(z + z
0
)
N
z
N
0
=
N

n=1
c
n
z
n
This is equivalent to
(z + z
0
)
N
=
N

n=1
c
n
z
n
+ z
N
0
We already have c
n
above. Then
(z + z
0
)
N
=
N

n=1
N!z
Nn
0
n! (N n)!
z
n
+ z
N
0
At this time, we notice that
c
0
=
N!z
N0
0
0! (N 0)!
z
0
= z
N
0
Hence we have (z + z
0
)
N
=
N

n=0
N!z
Nn
0
n! (N n)!
z
n
The proof is found completely.
146
5 INFINITE SERIES INVOLVING A COMPLEX VARIABLE
Exercise 5.12. (E. 29 P. 262)
1. Let f (z) = Log (z), where the principal branch of the logarithm is used . Thus f (z) is
dened by means of the branch cut y = 0, x 0. Show that
f (z) =

n=0
c
n
(z + 1 i)
n
where
c
0
= Log

2 + i
3
4
c
n
=
(1)
n+1
e
i(3/4)n
n
_
2
_
n
, n ,= 0
2. What is the radius of the largest circle centered at 1 + i within which the series of part
(1) converges to f (z)?
3. Use the ratio test to establish that the series of part (1) converges inside [z (1 + i)[ =

2 and diverges outside this circle. Compare this circle with the one in part (2).
Solution.
1. We have f (z) = Log (z) is the principal branch of the logarithm is used, and take the
circle [z (1 + i)[ <

2.
First c
0
= f (z
0
) = Log (z
0
) = Log
_
2
_
+ i
3
4
because of =
3
4
by deriving from
z
0
= 1 + i and [z (1 + i)[ <

2 with the principal value.


With n ,= 0, we still use Taylor series formular for general coecient, and compute the
rst, the second respectively.
c
1
=
f
(1)
(z
0
)
1!
=
e
i
3
4

2
c
2
=
f
(2)
(z
0
)
2!
=
e
2i
3
4
2
_
2
_
2
We have f

(z) =
1
z
so f
n
(z) = (1)
n+1
(n 1)!z
n
Thus
c
n
=
f
(n)
(z
0
)
n!
= (1)
n+1
(n 1)!z
n
0
n!
= (1)
n+1
e
ni
3
4
n
_
2
_
n
147
5 INFINITE SERIES INVOLVING A COMPLEX VARIABLE
Hence we have what we need to prove and that is f (z) =

n=0
c
n
(z + 1 i)
n
with c
0
=
Log
_
2
_
+ i
3
4
and c
n
=
f
(n)
(z
0
)
n!
= (1)
n+1
e
ni
3
4
n
_
2
_
n
, n ,= 0.
2. Because the branch cut of the principal Logz is y = 0 and x 0 so the largest radius of
circle center at 1 + i must be from the origin to 1 + i and that is d = [1 + i[ =

2.
3. We will examine
lim
n

c
n+1
(z + 1 i)
n+1
(z + 1 i)
n+1
c
n
(z + 1 i)
n
(z + 1 i)
n

= lim
n

(1)
n+2
e
i(n+1)
3
4
(n + 1)
_
2
_
n+1
(1)
n+1
e
in
3
4
n
_
2
_
n
(z + 1 i)

= lim
n

(1) ne
i
3
4
(n + 1)

2
(z + 1 i)

= lim
n
1

n
n + 1
(z + 1 i)

lim
n
n
(n + 1)

2
[(z + 1 i)[ = lim
n
[(z + 1 i)[

2
=
[(z + 1 i)[

2
Now with the ratio test, we set
[(z + 1 i)[

2
< 1 or [(z + 1 i)[ <

2
Our series converges absolutely if z lies inside a circle of radius

2 centered at the point
1 + i and diverges outside the circle. We need to observe on [(z + 1 i)[ =

2.
We have

(1)
n+1
e
in
3
4
n
_
2
_
n
(z + 1 i)
n

=
e
in
3
4
n
148
5 INFINITE SERIES INVOLVING A COMPLEX VARIABLE
Exercise 5.13. (E. 30 P. 262)
1. Let f (z) be analytic in a domain containing z = 0. Assume that f (z) is an even function
of z in this domain, i.e., f (z) = f (z). Show that in the Maclaurin expansion f (z) =

n=0
c
n
z
n
the coecients of odd order, c
1
, c
3
, c
5
..., must be zero.
2. Show that for an odd function, f (z) = f (z), analytic in the same kind of domain, the
coecients of even order, c
0
, c
2
, c
4
..., are zero.
3. What coecients vanish in the Maclaurin expansions of z sin z, z
2
tan z, and
cosh z/
_
1 + z
2
_
?
Solution.
1. f (z) is analytic in a domain containing z = 0 and it is an even function in this domain.
We have Maclaurin expansion f (z) =

n=0
c
n
z
n
=

n=0
c
n
(z)
n
So c
0
z
0
+ c
1
z
1
+ c
2
z
2
+ ... = c
0
(z)
0
+ c
1
(z)
1
+ c
2
(z)
2
+ ...
With even order we see (z)
n
= (z)
n
. Then we simplify the even order and get c
1
z
1
+
c
3
z
3
+ c
5
z
5
+ ... = 0 because n is an odd positive integer and z
n
,= z
n+1
for all n. In
order to get this sum equal to zero then all the odd coecients must be zero. That means
c
1
= c
3
= ... = c
2n+1
= 0
2. We do the same as part (1)

n=0
c
n
z
n
=

n=0
c
n
(z)
n
So we deduce that
c
0
z
0
+ c
1
z
1
+ c
2
z
2
+ ... = c
0
(z)
0
c
1
(z)
1
c
2
(z)
2
...
With even order we see (z)
n
= (z)
n
. Then this time it is the even order that remain in
this sum
c
0
z
0
+ c
2
z
4
+ c
4
z
4
+ ... = 0
To get the odd function, we must have even coecients equal to zero. That means
c
0
= c
2
= ... = c
2n
= 0
3. From part (1) and (2), we will now examine what coecients is zero
z sin (z) = z sin (z) because of sin (z) = sin (z) so z sin (z)is an even function, thus
its odd coecients are zero.
149
5 INFINITE SERIES INVOLVING A COMPLEX VARIABLE
z
2
tan (z) is an odd function since tan (z) =
sin (z)
cos (z)
and sin (z) = sin (z) , cos (z) =
cos (z) so its even coecients are zero.
cosh (z)
1 + z
2
=
e
z
+ e
z
2 (1 + z
2
)
=
e
z
+ e
z
2 (1 + z
2
)
so it is an even function then its odd coecients are
zero.
Exercise 5.14. (E. 31 P. 262)
Suppose a function f(z) is analytic in a domain which contains the origin, and we expand it in a
Maclaurin series f(z) =

n=0
c
n
z
n
. If the circle [z[ = r lies within the domain and if on the circle
we have the bound [f (z)[ K, then we can place a bound on [c
n
[, namely [c
n
[ K/r
n
, for
n N. This is called Cauchys inequality and it is sometimes useful in telling us if we have made
an error in computing the coecients in a Maclaurin series expansion; i.e., if the inequality is
not satised, there is a mistake. The inequality is easily generalized to Taylor series.
1. Derive the Cauchy inequality by using the following equation
c
n
=
f
(n)
(0)
n!
=
1
2i

f (z)
z
n+1
dz, n = 1, 2, 3, ...
, taking the contour C

as [z[ = r, and using the ML inequality.


2. Consider the expansion e
z
=

n=0
c
n
z
n
. We want to obtain a bound on [c
n
[ without actually
obtaining the coecients. Through suitable choices of r show that for all n 0 we have
[c
n
[ e and [c
n
[ e
2
/2
n
. Do the known coecients satisfy this inequality?
3. Generalize Cauchys inequality so that it can be applied to a Taylor series expansion f(z) =

n=0
c
n
(z z
0
)
n
. Use your result to explain why in the expansion e
z
=

n=0
c
n
(z 3)
n
we
can say, without nding any of the coecients, that [c
n
[ e
4
.
Solution.
1. We have
[c
n
[ =

1
2i

f (z)
z
n+1
dz

1
2
ML
by using ML inequality where M is the magnitude of
f (z)
z
n+1
and L is the length of the cicle
and that is L = 2r.
First

f (z)
z
n+1

K
[z
n+1
[
=
K
r
n+1
= M
Then ML inequality is that
[c
n
[ =
1
2

f (z)
z
n+1
dz

1
2
ML =
1
2
_
K
r
n+1
_
2r =
K
r
n
150
5 INFINITE SERIES INVOLVING A COMPLEX VARIABLE
Hence we obtain Cauchys inequality [c
n
[
K
r
n
, n = 1, 2, 3, ...
2. We observe that [c
n
[ e and get f (z) = e
z
=

n=0
c
n
z
n
and [c
n
[

e
z
z
n+1

r =
e
r
r
n
by
choosing r = 1. Like above but with r = 2 then we get [c
n
[
e
2
2
n
The known coecients satisfy this inequality because we using Maclaurin expansion for
z = 0 so it can satisfy.
3. We generalize the Cauchy inequality in Taylor expansion. This time is f (z) =

n=0
c
n
(z z
0
)
n
.
And
c
n
=
f
(n)
(z
0
)
n!
=
1
2i

f (z)
(z z
0
)
n+1
dz, n = 1, 2, 3, ...
where the circle is [z z
0
[ = r
If we use the same technique as part (1), then we will get the Cauchys inequality for
Taylor series expansion.
[c
n
[
K
r
n
where [f (z)[ K and [z z
0
[ = r.
151
5 INFINITE SERIES INVOLVING A COMPLEX VARIABLE
Exercise 5.15. (E. 32 P. 263)
1. Refer to the proof of theorem of Taylor series and use the equation
f (z
1
) =
1
2i

f (z)
z z
1
dz =
1
2i

f (z)
z
_
1
z
1
z
_dz
to show that
f (z
1
) =
1
2i

f (z)
_
_
_
_
1
z
+
z
1
z
2
+
z
2
1
z
3
+ ... +
z
n1
1
z
n
+
_
z
n
1
z
n
_
z z
1
_
_
_
_
dz
2. Use the expression for f (z
1
) given in part (1) to show, after integration, that
f (z
1
) = f (0) + f

(0) z
1
+
f

(0)
2
z
2
1
+ ... +
f
(n1)
(0)
(n 1)!
z
n1
1
+ R
n
where
R
n
=
z
n
1
2i

f(z)
z
n
(z z
1
)
dz
3. We can place an upper bound on the remainder in equation above. Assume [f (z)[ m
everywhere on [z[ = b (the contour C

). Use the ML inequality to show that


[R
n
[

z
1
b

n
mb
b [z
1
[
4. Suppose we wish to determine the approximate value of cosh i by the nite series in
Eq 5.4-21. Taking the contour C

as [z[ = 2, show that the error made cannot exceed


(cosh 2) /2
10
.
Solution.
1. Now we recall equation that is
1
1 z
= 1 + z + z
2
+ ... + z
n1
+
z
n
1 z
So for substitution z by
z
1
z
then we get
1
1
z
1
z
= 1 +
z
1
z
+
z
2
1
z
2
+ ... +
z
n1
1
z
n1
+
_
z
1
z
_
n
1
z
1
z
Therefore,
152
5 INFINITE SERIES INVOLVING A COMPLEX VARIABLE
f (z
1
) =
1
2i

f (z)
z
_
1
z
1
z
_dz
=
1
2i

f (z)
z
_
_
_
1 +
z
1
z
+
z
2
1
z
2
+ ... +
z
n1
1
z
n1
+
_
z
1
z
_
n
1
z
1
z
_
_
_
dz
=
1
2i

f (z)
_
_
_
_
1
z
+
z
1
z
2
+
z
2
1
z
3
+ ... +
z
n1
1
z
n
+
_
z
n
1
z
n
_
z z
1
_
_
_
_
dz
2. From part (1), we have had
f (z
1
) =
1
2i

f (z)
_
_
_
_
1
z
+
z
1
z
2
+
z
2
1
z
3
+ ... +
z
n1
1
z
n
+
_
z
n
1
z
n
_
z z
1
_
_
_
_
dz
=
1
2i

f (z)
1
z
dz +
1
2i

f (z)
z
1
z
2
dz + ... +
1
2i

f (z)
z
n1
1
z
n
dz +
1
2i

f (z)
_
z
n
1
z
n
_
z z
1
dz
From equation 5.4-17 we construct c
n
=
f
(n)
(0)
n!
=
1
2i

f (z)
z
n+1
dz, we get
f (z
1
) = f
0
(0) +
f
(1)
(0)
1!
z
1
+ ... +
f
(n1)
(0)
(n 1)!
z
n1
1
+
z
n
1
2i

f (z)
z
n
(z z
1
)
dz
3. We set R
n
=
z
n
1
2i

f (z)
z
n
(z z
1
)
dz is the remainder and assume [f (z)[ m everywhere
on [z[ = b (the contour C

).
Let take [R
n
[ =

z
n
1
2i

f (z)
z
n
(z z
1
)
dz

ML (ML inequality)
First we examine L = 2b is the length of C

. And the second is

z
n
1
2i
.
f (z)
z
n
(z z
1
)

1
2

z
n
1
z
n
(z z
1
)

[f (z)[
1
2

z
n
1
b
n

m
b [z
1
[
= M
Hence,
ML =
1
2

z
n
1
b
n

m
b [z
1
[
2b =

z
n
1
b
n

mb
b [z
1
[
Thus we have [R
n
[

z
n
1
b
n

mb
b [z
1
[
.
4. We already have cosh (z) =
e
z
+ e
z
2
so [cosh (z)[ =

e
z
+ e
z
2


e
x
+ e
x
2
= cosh (x).
Then [cosh (z)[ cosh (x).
153
5 INFINITE SERIES INVOLVING A COMPLEX VARIABLE
Using part (3) with [z[ = 2 = b and f (z) = cosh (z). Therefore, we get [f (z)[ cosh (2) =
m
[R
n
[

z
n
1
b
n

mb
b [z
1
[
=

i
10
2
10

2 cosh (2)
2 [i[
=
cosh (2)
2
9
with n = 10
Hence the remainder is
cosh (2)
2
9
and the error can not exceed
cosh (2)
2
10
Exercise 5.16. (E. 33 P. 263)
Let a function f (z) be entire and have property that f
(n)
(z) = f (z) , n = 1, 2, ... Show by using
a Taylor series expansion that this function must have this addition property : f (z
1
+ z
2
) =
f (z
1
) f (z
2
).
Solution.
We get the Maclaurin expansion of f (z
2
) =

n=0
c
n
z
n
2
where c
n
=
f (0)
n!
for all n N (in
equation 5.4-18 and 5.4-19). Now we use Taylor expansion (equation 5.4-10)
f (z) =

n=0
c
n
(z z
0
)
n
, [z z
0
[ a
Let z = z
1
+ z
2
and z
0
= z
1
then
f (z
1
+ z
2
) =

n=0
b
n
(z
1
+ z
2
z
1
)
n
=

n=0
b
n
(z
2
)
n
=

c
n
(z
2
)
n
So the Taylor series becomes the Maclaurin. Hence, we want to get the Taylor so it must be
b
n
=
f (z
1
) f (0)
(n!)
.
We replace with the one into
f (z
1
+ z
2
) =

n=0
b
n
(z
2
)
n
= f (z
1
)

n=0
f (0)
n!
(z
2
)
n
= f (z
1
) f (z
2
)
154
5 INFINITE SERIES INVOLVING A COMPLEX VARIABLE
5.5 TECHNIQUES FOR OBTAINING TAYLOR SERIES
EXPANSIONS
In this section, the exercises will be shown are 1, 2, 3, 4, 5, 7, 8, 11, 12, 15, 16, 19, 23.
Exercise 5.17. (E. 1, E. 2, E. 3, E. 4 P. 274)
The following exercises involve our generating a new Taylor series through a change of variables
in the geometric series Eq.(5.2-8) or some other familiar expansion. Here a is any constant.
Explain how the following are derived
1.
1
1 + az
= 1 az + a
2
z
2
a
3
z
3
+ ...
_
[z[ <

1
a

_
2.
1
1 + z
2
= 1 z
2
+ z
4
z
6
+ ... ([z[ < 1)
3.
1
1 + a + z
= 1 (z + a) + (z + a)
2
(z + a)
3
+ ... ([z a[ < 1)
4.
a) e
z
2
= 1 z
2
+
z
4
2

z
6
6
+ ... all z
b) Use the preceding result to nd the 10th derivative of e
z
2
at z = 0.
Solution.
1. Because of [z[ <

1
a

, we get
1
1 + az
=
1
1 (az)
=

n=0
(az)
n
2. We use the formular
1
1 + w
= 1 w + w
2
w
3
+ ..., [w[ < 1
And set w = z
2
. we can infer
1
1 + z
2
= 1 z
2
+ z
4
z
6
+ ..., [z[ < 1
3. In a similar fashion, we also use the formular
1
1 w
= 1 + w + w
2
+ w
3
+ ..., w < 1
We can deduce that
1
1 + a + z
= 1 (z + a) + (z + a)
2
(z + a)
3
+ ...
155
5 INFINITE SERIES INVOLVING A COMPLEX VARIABLE
for [a z[ < 1 or [z + a[ < 1
4.
a) The formular is e
w
= 1 + w +
w
2
2!
+ ... for all w
Then with w = z
2
, we replace into this formular and get
e
z
2
= 1 +z
2
+
z
4
2!

z
6
3!
+
z
8
4!
...
b) Apply Taylor series expansion, we see the general coecient is c
n
=
f
(n)
(z
0
)
n!
So with z
0
= 0, we have
f
(n)
(0) = c
n
n!
Hence, the coecient of z
10
is
1
5!
in the series of (a)
Thus f
(10)
(0) =
10!
5!
= 30240
Exercise 5.18. (E. 5, E. 7, E. 8 P. 274)
1. Dierentiate the series of equation 5.5-2 to show that
1
z
3
= 1
3.2
2
(z 1) +
4.3
2
(z 1)
2

5.4
2
(z 1)
3
+ ..., [z 1[ < 1
2. Use the series in equation 5.2-8 and successive dierentiation to show that, for N 0
1
(1 z)
N
=

n=0
c
n
z
n
, c
n
=
(N 1 + n)!
n! (N 1)!
, [z[ < 1
3. Integrate the series of Exercise 2 along a contour connecting the origin to an arbitrary
point z, where [z[ < 1 to show that
tan
1
z =

n=0
(1)
n
z
2n+1
2n + 1
, [z[ < 1
Solution.
1. We can divide it as follows
156
5 INFINITE SERIES INVOLVING A COMPLEX VARIABLE
1
z
3
=
1
z
2
.z
, [z 1[ < 1
=
_

n=0
(n + 1) (1 z)
n
__

n=0
(1 z)
n
_

n=0
(n + 2) (n + 1)
2
(1 z)
n
This series express as the right side in the proof.
2. In equation 5.2-8, we have

j=1
z
j1
= 1 + z + z
2
+ ... = S (z) =
1
1 z
, [z[ < 1
Successive dierentiation is that
1
(1 z)
N
= z
0
+
N!
(N 1)!
z +
(N + 1)!
(N 1)!2!
z
2
+
(N + 2)!
(N 1)!3!
z
3
+ ...
=

n=0
(N 1 + n)!
n! (N 1)!
z
n
where [z[ < 1.
3. We observe the formular below
1
1 + w
2
= 1 w
2
+ w
4
w
6
+ ..., [w[ < 1
And with [z[ < 1, we have

0
z
1
1 + w
2
dw =

0
z
_
1 w
2
+ w
4
w
6
+ ...
_
dw
= z
z
2
3
+
z
5
5

z
7
7
+ ...
Then

0
z
1
1 + w
2
dw = tan
1
z
Thus tan
1
z =

n=0
(1)
n
z
2n+1
2n + 1
with [z[ < 1.
157
5 INFINITE SERIES INVOLVING A COMPLEX VARIABLE
Exercise 5.19. (E. 11 P. 276)
By considering the rst and second derivatives of the geometric series in Equation 5.28, show
that

n=1
n
2
z
n
=
_
z + z
2
_
(1 z)
3
for [z[ < 1
Solution.
In equation 5.2-8, we have had

j=1
z
j1
= 1 + z + z
2
+ z
3
+ z
4
+ z
5
+ ... =
1
1 z
, [z[ < 1
Successive dierentiation the rst time, we obtain
1
(1 z)
2
= 1 + 2z + 3z
2
+ 4z
3
+ 5z
4
+ 6z
5
+ ...
Successive dierentiation the second time, we get
2
(1 z)
3
= 2 + 6z + 12z
2
+ 10z
3
+ ...
We compute the following series
z
(1 z)
2
= z + 2z
2
+ 3z
3
+ 4z
4
+ ... =

n=1
nz
n
2z
(1 z)
3
= 2z + 6z
2
+ 12z
3
+ 20z
4
+ ... =

n=1
n(n + 1) z
n
So
2z
(1 z)
3

z
(1 z)
2
=
2z z (1 z)
(1 z)
3
=
z
2
+ z
(1 z)
3
=

n=1
n(n + 1) z
n

n=1
nz
n
=

n=1
n
2
z
n
Obtain the following Taylor expansions. Give a general formula for the nth coecient, and
state the circle within which your expansion is valid.
Exercise 5.20. (E. 12 P. 276)
Use series multiplication to nd a formula for c
n
in these Maclaurin expansion. In what circle
is each series valid?
cosh z
1 z
=

n=0
c
n
z
n
Solution.
We have that for all z then
cosh z = 1 +
z
2
2!
+
z
4
4!
+
z
6
6!
+ ...
158
5 INFINITE SERIES INVOLVING A COMPLEX VARIABLE
And with [z[ < 1
1
1 z
= 1 + z + z
2
+ z
3
+ ...
Then we multiply them and get
cosh z
1 z
=
_
1 +
z
2
2!
+
z
4
4!
+
z
6
6!
+ ...
_
_
1 + z + z
2
+ z
3
+ ...
_
=

n=0
c
n
z
n
= 1 + z +
_
1 +
1
2!
_
z
2
+
_
1 +
1
2!
_
z
3
+
_
1 +
1
2!
+
1
4!
_
z
2
+ ...
Hence, the general coecient is
c
n
=
_

_
1 +
1
2!
+
1
4!
+ ... +
1
n!
, neven
1 +
1
2!
+
1
4!
+ ... +
1
(n 1)!
, nodd
Exercise 5.21. (E. 15, E. 16, E. 19, E. 23 P. 276)
Obtain the following Taylor expansions. Give a general formula for the nth coecient, and state
the circle within which your expansion is valid.
1.
z
(z 1) (z + 2)
expanded about z = 0
2.
z + 1
(z 1)
2
(z + 2)
expanded about z = 2
3.
1
z
3
expanded about z = i
4.
z
3
+ 2z
2
+ z 1
z
2
4
expanded about z = 1
Solution.
1. We have, from Rule I,
f (z) =
z
(z 1) (z + 2)
=
A
(z 1)
+
B
(z + 2)
=
z (A + B) + 2AB
(z 1) (z + 2)
We obtain a system of equations
159
5 INFINITE SERIES INVOLVING A COMPLEX VARIABLE
_
_
_
A + B = 1
2AB = 0
Then we get the root is
_

_
A =
1
3
B =
2
3
Thus f (z) be rewritten
f (z) =
1
3
1
z 1
+
2
3
1
z + 2
Then, we see 2 familiar series below.
1
3
1
z 1
=
1
3
_
1 + z + z
2
+ z
3
+ ...
_
=
1
3

n=0
z
n
2
3
1
z + 2
=
2
3
1
1 +
z
2
=
2
3

n=0
(1)
n
2
n
z
n
We can infer
f (z) =
1
3

n=0
z
n
+
2
3

n=0
(1)
n
2
n
z
n
=

n=0
_

1
3
+
2
3
(1)
n
2
n
_
z
n
2. In a similar way, we get
2
z + 2
=
2
z 1 + 3
=
2
3
1 +
z 1
3
=
2
3
_
1
z 1
3
+
_
z 1
3
_
2
...
_

1
z + 1
=
1
z 1 + 2
=
1
2
1 +
z 1
2
=
1
2
_
1
z 1
2
+
_
z 1
2
_
2
...
_
Add the two previous series
z
(z + 1) (z + 2)
=

n=0
c
n
(z 1)
n
where c
n
=
1
2
_

1
2
_
n
+
2
3
_

1
3
_
n
160
5 INFINITE SERIES INVOLVING A COMPLEX VARIABLE
3. We have
f (z) =
z + 1
(z 1)
2
(z + 2)
=
A
(z 1)
+
B
(z 1)
2
+
C
(z + 2)
=
A(z 1) (z + 2) + B (z + 2) + C (z 1)
2
(z 1)
2
(z + 2)
=
z
2
(A + C) + z (A + B 2C) 2A + 2B + C
(z 1)
2
(z + 2)
Let the numerator be equal to z + 1 and we get A =
1
9
, B =
2
3
, C =
1
9
Hence,
f (z) =
1
9
.
1
(z 1)
+
2
3
.
1
(z 1)
2

1
9
.
1
(z + 2)
Then
1
9
.
1
(z 1)
=
1
9
.
1
(1 + z 2)
=
1
9
.

n=0
(1)
n
(z 2)
n
, [z 2[ < 1
2
3
.
1
(z 1)
2
=
2
3
.
1
(1 + z 2)
2
=
2
3
.

n=0
(1)
n
(n + 1) (z 2)
n
, [z 1[ < 1

1
9
.
1
(z + 2)
=
1
9
.
1
(4 + z 2)
=
1
36
1
1 +
z 2
4
=
1
36
.

n=0
(1)
n
4
n
(z 2)
n
,

z 1
4

<
1
We can infer
f (z) =

n=0
c
n
z
n
where c
n
=
1
9
. (1)
n
+
2
3
. (1)
n
(n + 1)
1
36
. (1)
n
4
n
4. We see that
f (z) =
z
3
+ 2z
2
+ z 1
z
2
4
= (z 1) + 3 +
5z + 7
z
2
4
And analyse that
5z + 7
z
2
4
=
A
(z + 2)
+
B
(z 2)
=
z (A + B) 2A + 2B
z
2
4
161
5 INFINITE SERIES INVOLVING A COMPLEX VARIABLE
Let the numerator equal to 5z + 7 and we get A =
3
4
, B =
17
4
Then
5z + 7
z
2
4
=
3
4
.
1
(z + 2)
+
17
4
.
1
(z 2)
3
4
.
1
(z + 2)
=
1
4
.
1
_
1 +
z 1
3
_ =

n=0
1
4
. (1)
n
.
1
3
n
. (z 1)
n
, [z 1[<3
17
4
.
1
(z 2)
=
17
4
.
1
(1 (z 1))
=

n=0

17
4
. (z 1)
n
, [z 1[<1
We can infer
f (z) =

n=0
c
n
(z 1)
n
where c
n
=
1
4
. (1)
n
.
1
3
n

17
4
162
5 INFINITE SERIES INVOLVING A COMPLEX VARIABLE
5.6 LAURENT SERIES
We consider some excercises 1, 2, 3, 4, 5, 6, 7, 9, 11.
Exercise 5.22. (E. 1, E. 2, E. 3, E. 4, E. 5 P. 293)
Obtain the following Laurent expansions. State the rst four nonzero terms. State explicitly
the nth term in the series, and state the largest possible annular domain in which your series is
a valid representation of the function.
1.
sinh z
z
3
expanded in powers of z
2.
cos (1/z)
z
3
expanded in powers of z
3. sin
_
1 +
1
z 1
_
expanded in powers of z 1
4. Log
_
1 +
1
z 1
_
expanded in powers of z 1
5.
_
z +
1
z
_
7
expanded in powers of z
Solution.
1. We have
sinh z = z +
z
3
3!
+
z
5
5!
+
z
7
7!
+ ...
So that
f(z) = z
2
+
1
3!
+
z
2
5!
+
z
4
7!
+ ...
=

n=1
z
2n
(2n + 3)!
, [z[ > 0
2. We get
cos
_
1
z
_
= 1
z
2
2!
+
z
4
4!

z
6
6!
+ ...
Hence,
163
5 INFINITE SERIES INVOLVING A COMPLEX VARIABLE
f(z) = z
3

1
2!
z
5
+
z
7
4!
+
z
9
6!
+ ...
=
3

n=
z
(2n+1)
(2n 1)!
=
3

n=
z
m
(m3)
, [z[ > 0
3. We use the formula of sin (a + b) for this function.
sin
_
1 +
1
z 1
_
= sin(1) cos
_
1
z 1
_
cos (1) sin
_
1
z 1
_
cos
_
1
z 1
_
=

n=1
_
1
z 1
_
2n

1
(2n)!
(1)
m
sin
_
1
z 1
_
=

m=0
_
1
z 1
_
2m+1

1
(2m + 1)!
(1)
m
Thus
f(z) =
n

i=0
c
i
(z 1)
i
with the general coefficient
_
_
_
c
i
=
(1)
i/2
sin(1)
(i)!
, ieven
c
i
=
(1)
(i+1)/2
cos(1)
(i)!
, iodd
4. We can see that
Log
_
1 +
1
z 1
_
= Log (z) + Log
_
1
z 1
_
Log (z) =

n=0
(1)
2n+1
nz
n
Log
_
1
z 1
_
=

z
0
1
1 w
dw
=

z
0

n=0
w
n
dw
=

n=0

z
0
w
n
dw
=

m=1
z
m
m
164
5 INFINITE SERIES INVOLVING A COMPLEX VARIABLE
So the result is
f(z) =

n=0
(1)
2n+1
nz
n

m=1
z
m
m
=

c
k
z
k
with the general coefficient
_
_
_
c
k
=
(1)
2k+1
k
, k 0
c
k
=
1
m
, k 1;
5. We solve this excercise easily.
f(z) =
_
z +
1
z
_
7
=
7

i=0
C
i
7
z
72i
= z
7
+ 7z
5
+ 21z
3
+ 35z + ...
=
7

i=0
7!
(7 i)!i!
; [z[ > 0
Exercise 5.23. (E. 6, E. 7 P. 293)
Obtain the indicated Laurent expansions of
1
z + i
. State the nth terms of the series.
1. An expansion valid for [z[ > 1
2. An expansion valid for [z i[ > 2
Solution.
f(z) =
1
i + z
; [z[ > 1
=
1
z

n=0
_
i
z
_
n
(1)
n
=

n=0
i
n
(1)
n
z
n1
=
1

m=
_
1
i
_
m+1
(1)
m+1
z
m
165
5 INFINITE SERIES INVOLVING A COMPLEX VARIABLE
f(z) =
1
i + z
; [z i[ > 2
1
z i

n=0
_
2i
z i
_
n
(1)
n
=

n=0
2
n
i
n
(1)
n
(z i)
n1
=
1

m
_
1
2i
_
m+1
(1)
m+1
(z i)
m
Exercise 5.24. (E. 9 P. 293)
Expand the following functions in a Laurent series valid in a domain whose outer radius is
innite. State the center and inner radius of the domain. Give the nth term of the series.
1
z + 2
expanded in powers of z i
Solution.
f(z) =
1
z + 2
=
1
z i + i + 2
=
1
(z i)
_
1 +
i+2
zi
_
=

n=0
_
(i + 2)
n
(1)
n
(z i)
n+1
_
; [z i[ >

5
=
1

m=
(z i)
m
(i + 2)
m1
(1)
m+1
; [z i[ > 1
Exercise 5.25. (E. 11 P. 293)
1. Consider f (z) =
1
z (z 1) (z + 3)
. This function is expanded in three dierent Laurent
series involving powers of z. State the three domains in which Laurent series are available.
2. Find each series and give an explicit formula for the nth term.
Solution.
1. We will expand on 3 domains as follow
M
1
= z : 0 < [z[ < 1
M
2
= z : 1 < [z[ < 3
M
3
= z : [z[ > 3
2. Consider the domain M
1
, we have
166
5 INFINITE SERIES INVOLVING A COMPLEX VARIABLE
f(z) =
1
[z (z 1) (z + 3)]
=
1
4z
_
1
z 1

1
z + 3
_
=
1
4z
_

n=0
z
n

1
3

n=0
_
z
3
_
n
(1)
n
_
=

n=0
1
4
z
n1

n=0
z
n1
_
1
4

1
3
n+1
_
(1)
n
=

n=0
1
4
z
n1
_
1
1
3
n+1
(1)
n
_
=

m=1
1
4
z
n1
_
1
1
3
m+2
(1)
m+1
_
The next domain M
2
, we obtain
f(z) =
1
[z (z 1) (z + 3)]
=
1
4z
_
1
z 1

1
z + 3
_
=
1
4z
_
1
z

n=0
_
1
z
_
n

1
3

m=0
(1)
m
_
z
3
_
m
_
=
2

k=
z
k
_
1
4
_

t=1
(1)
t+1
z
i
_
1
4

1
3
t+2
_
=

c
t
z
t
with the general coefficient is
_
_
_
c
t
=
1
4
t 2
c
t
= (1)
t
_
1
4

1
3
t+2
_
, t 1
And the last is M
3
167
5 INFINITE SERIES INVOLVING A COMPLEX VARIABLE
f(z) =
1
[z (z 1) (z + 3)]
=
1
4z
_
1
z 1

1
z + 3
_
=
1
4z
_
1
z

n=0
_
1
z
_
n

1
z

m=0
(1)
m
_
3
z
_
m
_
=

n=0
1
4
z
n2

m=0
1
4
(1)
m
3
m
z
m2
=

n=0
1
4
z
n2
_
1 + (1)
n+1
3
n
_
=
2

n=
1
4
z
k
_
1 + (1)
k+1
3
k2
_
168
6 RESIDUES AND THEIR USE IN
INTEGRATION
6.1 INTRODUCTION AND DEFINITION OF THE RESIDUES
The rst is shown by the exercises such as 1, 2, 3, 4, 5, 6, 7, 10.
Exercise 6.1. (E. 1, E. 2, E. 3 P. 341)
Using the method of residues, evaluate the integral

C
1
(z 1)
2
+
i
z 1
+ 2 (z 1) +
3
z 4
dz
, where the contour C is given below.
1. [z 1[ = 2
2. [z 5[ = 2
3. The rectangle with corners at (5 i)
Solution.
1. We can see that the function f (z) = 2 (z 1) +
3
z 4
is integrability on and even in the
contour C, so

C
f (z) dz = 0. Then we need to compute

C
1
(z 1)
2
+
i
z 1
+ f (z) dz = 2i
_
Res
_
1
(z 1)
2
, 1
_
+ Res
_
i
z 1
, 1
__
= 2i (0 + i) = 2
2. Using a similar comment, we have

C
1
(z 1)
2
+
i
z 1
+ 2 (z 1) dz = 0. Hence,

C
1
(z 1)
2
+
i
z 1
+ 2 (z 1) +
3
z 4
dz = 2iRes
_
3
z 4
, 4
_
= 6i
3. In this exercise, in the given rectangle, we obtain

C
2 (z 1) dz = 0. Thus

C
1
(z 1)
2
+
i
z 1
+2 (z 1)+
3
z 4
dz = 2i
_
Res
_
1
(z 1)
2
, 1
_
+ Res
_
i
z 1
, 1
_
+ Res
_
3
z 4
, 4
__
169
6 RESIDUES AND THEIR USE IN INTEGRATION
According to exercise 1 and 2, we get the result is

C
1
(z 1)
2
+
i
z 1
+ 2 (z 1) +
3
z 4
dz = 2i (3 + i)
Exercise 6.2. (E. 4, E. 5, E. 6, E. 7 P. 341)
Evaluate the following integrals by using the method of residues. In Problems 6-8 use Laurent
expansions valid in deleted neiborghoods of the singular points to get the residue.
1.

n=
e
n
2
(n 1) (z 1)
n
dz around [z[ = 2
2.

n=5
1
(z + i)
n
(n + 6)!
dz around [z i[ = 3
3.

n=
cosh (1/z) dz around the square with corners at (1 i)
4.

z sin
_
1
z 1
_
dz around [z[ = 2
Solution.
1. We consider the function f (z) =

n=
e
n
2
(n 1) (z 1)
n
on the contour [z[ = 2 and
see that f is integrability for all z ,= 1. So we have a singular points z
0
= 1 where n < 0.
Applying the method of residues, we get

C
f (z) dz = Res [f (z) , 1]
= 2e
1
2. In this exercise, we note that

C
4

n=
1
(z + i)
n
(n + 6)!
= 0
So that the rst integral becomes

n=5
1
(z + i)
n
(n + 6)!
=

n=
1
(z + i)
n
(n + 6)!
In a similar fashion, we compute the result is

n=5
1
(z + i)
n
(n + 6)!
=
2i
7
170
6 RESIDUES AND THEIR USE IN INTEGRATION
3. We have z
0
= 0 is a singular point of the function f (z) = cosh (1/z) in the square with
the corners (1 i). Then we need to extense the function by Laurent series.
cosh (1/z) = 1 +
1
2!z
2
+
1
4!z
4
+ ...
Hence,

n=
cosh (1/z) dz = Res [cosh (1/z) , 0] = 0
4. Like the exercise above, we see that z
0
= 1 is a singular point of the function f (z) =
z sin
_
1
z 1
_
in the contour [z[ = 2. Next, we must expand the function by Laurent
series.
z sin
_
1
z 1
_
= (z 1) sin
_
1
z 1
_
+ sin
_
1
z 1
_
= (z 1)
_
_
1
z 1
_

1
3!
_
1
z 1
_
3
+ . . .
_
+
_
_
1
z 1
_

1
3!
_
1
z 1
_
3
+ . . .
_
= 1 +
_
1
z 1
_

1
3!
_
1
z 1
_
2

1
3!
_
1
z 1
_
3
+ . . .
Thus the result is

C
z sin
_
1
z 1
_
dz = 2iRes
_
z sin
_
1
z 1
_
, 1
_
= 2i
Exercise 6.3. (E. 10 P. 341)
Use residue calculus to show that if n 1 is an integer, then

C
_
z +
1
z
_
n
dz =
_

_
2in!

n 1
2

n + 1
2

!
, nodd
0, neven
where C is any simple closed contour encircling the origin.
Solution.
We consider the function as follows
f (z) =
_
z +
1
z
_
n
=
n

k=0
C
k
n
z
nk
z
k
=
n

k=0
C
k
n
z
n2k
171
6 RESIDUES AND THEIR USE IN INTEGRATION
There are 2 cases below.
n = 2m. We note that

|zz
0
|=r
(z z
0
)
l
dz =
_
_
_
0, l ,= 1
2i, l = 1
Because of n 2k = 2m2k ,= 1 for all m, k, we obtain

c
f (z) dz = 0.
n = 2m + 1. We see that

c
f (z) dz =

c
2m+1

k=0
C
k
2m+1
z
2m+12k
dz
=
2m+1

k=0
C
k
2m+1

c
z
2m+12k
dz
Then, by solving the equation 2m + 1 2k = 1, we have k = m + 1. Hence,

c
f (z) dz = 2iC
m+1
2m+1
=
2i (2m + 1)
(m + 1)! (2m + 1 m1)!
=
2in!
_
n 1
2
_
!
_
n + 1
2
_
!
So we complete the proof.
172
6 RESIDUES AND THEIR USE IN INTEGRATION
6.2 ISOLATED SINGULARITIES
We use the exercises 1, 2, 3, 8, 9, 10, 17, 18, 19, 23 for our examples.
Exercise 6.4. (E. 1, E. 2, E. 5 P. 350)
Show by means of a Laurent series expansion

n=
c
n
(z z
0
)
n
that the following functions
have essential singularities at the points stated. State the residue and give c
2
, c
1
, c
0
, c
1
.
1. sinh (1/z) at z = 0
2. (z 1)
3
cosh (1/ (z 1)) at z = 1
3. e
1/(zi)
e
zi
at z = i
Solution.
By using Laurent series for the given function, we can easily compute the coecients are
needed. Moreover, the rst negative coecient is really the residue. Indeed, the reader can see
as follows
1. We have
sinh
_
1
z
_
=
1
z
+
1
3!z
3
+
1
5!z
5
+ . . . , z ,= 0
Then c
2
= 0, c
1
= 1, c
0
= 0, c
1
= 0.
2. We get
(z 1)
3
cosh
_
1
z 1
_
= (z 1)
3
_
1 +
1
2! (z 1)
2
+
1
4! (z 1)
4
+ . . .
_
= (z 1)
3
+
1
2!
(z 1) +
1
4! (z 1)
+ . . .
Then c
2
= 0, c
1
=
1
4!
, c
0
=
1
2
, c
1
= 0.
3. We obtain
e
1
zi
e
zi
=
_
1 +
1
1! (z i)
+
1
2! (z i)
2
+ . . .
_ _
1 + (z i) +
1
2!
(z i)
2
+ . . .
_
=

n=0

k=0
1
n!
1
k!
(z i)
nk
Then
173
6 RESIDUES AND THEIR USE IN INTEGRATION
c
2
=

n=2
1
n! (n 2)!
c
1
=

n=1
1
n! (n 1)!
c
0
=

n=0
_
1
(n!)
2
_
c
1
=

n=0
1
n! (n + 1)!
Exercise 6.5. (E. 8, E. 9, E. 10 P. 350)
Use series expansions or LHopitals rule to show that the following functions possess removable
singularities at the indicated singular points. You must show that lim
zz
0
f (z) exists and is nite
at the singular point. Also, state how f (z
0
) should be dened at each point in order to remove
the singularity. Use principal branches where there is any ambiguity.
1.
e
z
1
z
at z = 0
2.
e
z
e
Logz
at z = 1
3.
sinh z
z
2
+
2
at the two singular points
Solution.
In three exercises, we use LHopital rule to show the functions possess removable singularities
at the indicated singular points.
1. We have
lim
z0
e
z
1
z
= lim
z0
e
z
= 1
Hence,
f (z) =
_
_
_
e
z
1
z
, z ,= 0
1 , z = 0
2. Next, we get
lim
z1
e
z
e
Logz
= lim
z1
e
z
1
z
= e
Thus
174
6 RESIDUES AND THEIR USE IN INTEGRATION
f (z) =
_

_
e
z
e
log z
, z ,= 1
e , z = 1
3. We are carefully because we obtain 2 roots in the equation z
2
+
2
= 0. Then, we have
lim
zi
sinh z
z
2
+
2
= lim
zi
1
(1 + z
2
)
1/2
2z
=
1
2i (1 + i)
1/2
lim
zi
sinh z
z
2
+
2
= lim
zi
1
(1 + z
2
)
1/2
2z
=
1
2i (1 i)
1/2
So the result is
f (z) =
_

_
sinh z
z
2
+
2
, z ,= i
1
2i (1 + i)
1/2
, z = i
1
2i (1 i)
1/2
, z = i
Exercise 6.6. (E. 17, E. 18, E. 19, E. 23 P. 351)
State the location of all the poles for each of the following functions and give the order of each
pole. Use the principal branch of the given functions if there is any ambiguity.
1.
1
z
2
+ 2z + 1
2.
1
z
2
+ z + 1
3.
1
z
3
1
4.
1
10
z
e
z
Solution.
In four exercises, we need to deal with the equation of denominator for each function.
1. The root in there is z = 1, so we try assuming that z = 1 is a pole of order two.
lim
z1
(z + 1)
2
1
z
2
+ 2z + 1
= 0
Hence, our assumption is true.
2. In a similar fashion, the roots are z =
1
2
+ i

3
2
, z =
1
2
i

3
2
. Then this means that
two roots are really two simple poles.
175
6 RESIDUES AND THEIR USE IN INTEGRATION
3. We see easily that the roots in this function are z = 1, z =
1
2
+ i

3
2
, z =
1
2
i

3
2
.
We deduce similarly that three roots are simple poles.
4. We carefully solve the equation 10
z
= e
z
and obtain the root is z =
i2k
log 10 1
. So that
this is a simple pole.
176
6 RESIDUES AND THEIR USE IN INTEGRATION
6.3 FINDING THE RESIDUE
Exercise 6.7. (E. 3, E. 4, E. 5 P. 357)
For each of the following functions state the location and order of each pole and nd the
corresponding residue. Use the principal branch of any multivalued function given below.
1.
cos z
z
2
+ z + 1
2.
1
z

e
z
z (z + 1)
+
1
(z 1)
4
3.
1
z
1/2
(z
2
9)
2
Solution.
1. We factor the denominator and obtain
f (z) =
cos z
_
z
1 i

3
2
__
z
1 + i

3
2
_
We can easily examine that both z =
1 i

3
2
are single poles of f (z).
So we will compute the residue of each pole
Res
_
f (z) ,
1 i

3
2
_
= lim
z
1i

3
2
_
z
1 i

3
2
_
f (z)
=
cos
_
1 i

3
2
_
i

3
Res
_
f (z) ,
1 + i

3
2
_
= lim
z
1+i

3
2
_
z
1 + i

3
2
_
f (z)
=
cos
_
1 + i

3
2
_
i

3
2. Take f (z) =
1
z

e
z
z (z + 1)
+
1
(z 1)
4
Just like Exercise 3 we can see that f (z) has two poles at z = 1 with order 1 and z = 1
with order 4 as the reader can verify themselves.
Because lim
z0
zf (z) = 0, z = 0 is not a pole of f (z)
So we compute easily
177
6 RESIDUES AND THEIR USE IN INTEGRATION
Res [f (z) , 1] = lim
z1
(z + 1) f (z) = e
1
Res [f (z) , 1] = lim
z1
d
3
2dz
3
_
(z 1)
4
f (z)
_
= 0
3. Because we use principle branch of z
1
2
, f (z) have no pole at z = 0 and z = 3 but it has
a pole z = 3 with order 2.
Res [f (z) , 3] = lim
z3
d
dz
_
(z 3)
2
f (z)
_
= lim
z3

1
2

z
(z + 3) 2

z
z (z + 3)
3
=
1
72

3
Exercise 6.8. (E. 16, E. 18, E. 20 P. 358)
Find the residue of the following functions at the indicated point.
1.
z + 1
z
sin (1/z) at 0
2.
1
(z + i)
5
at i
3.
z
12
(z 1)
10
at 1
Solution.
1. We see that the function f (z) =
z + 1
z
sin (1/z) has a pole of order 2 at z = 0. We need
to prove that
lim
z0
z
2
f (z) = 0
Indeed, we have

z
2
z + 1
z
sin (1/z)

[z (z + 1)[
We know that lim
z0
z (z + 1) = 0, so we obtain the proof.
2. We see that the function has a pole of order 6 at z = i. Indeed, we have
lim
zi
(z + i)
6
1
(z + i)
5
= lim
zi
(z + i) = 0
3. In a similar fashion, the function has a pole of order 11 at z = 1. We see this as follows
178
6 RESIDUES AND THEIR USE IN INTEGRATION
lim
z1
(z 1)
11
z
12
(z 1)
10
= lim
z1
z
12
(z 1) = 0
Exercise 6.9. (E. 27, E. 31 P. 358)
Use residues to evaluate the following integrals. Use the principal branch of multivalued func-
tions.
1.

dz
sin z
around [z 6[ = 4
2.

e
1/z
z
2
1
dz around [z 1[ = 3/2
Solution.
179
6 RESIDUES AND THEIR USE IN INTEGRATION
6.4 EVALUATION OF REAL INTEGRALS WITH RESIDUE
CALCULUS I
The reader can see exercises 3, 4, 7, 8 for examples of residue calculus I.
Exercise 6.10. (E. 3, E. 4 P. 364)
Using residue calculus, establish the following identities.
1.

d
a + b cos
=
2

a
2
b
2
for a > b 0.
2.
3
2

2
cos
a + b cos
d =
2
b
_
1
a

a
2
b
2
_
for a > b > 0.
Solution.
1. We put t = + and see that

d
a + b cos
=

2
0
d
a b cos
The next is the denominator is denited because of a > b 0. Then we apply the method
of evaluation of real integrals with residue calculus 1. We get z = e
i
, so that
_
_
_
d =
dz
iz
cos =
z+z
1
2
Then the integral becomes

2
0
d
a b cos
=

|z|=1
2dz
2iaz ibz
2
ib
We need to face with the equation ibz
2
2iaz + ib = 0 and have 2 roots easily.
z =
a

a
2
b
2
b
But the only point, z =
a

a
2
b
2
b
, is in the unit circle, the reader can verify themselves.
Now we only compute the residue at this point.

2
0
2d
2iaz ibz
2
ib
= 4iRes
_
1
2iaz ibz
2
ib
;
a

a
2
b
2
b
_
=
2

a
2
b
2
Note that, we only get the result in the case a > b > 0, but do not worry, it is very easy
to get the proof for the cases a > b = 0.
180
6 RESIDUES AND THEIR USE IN INTEGRATION
2. Like the exercise above, we put t = +

2
and see that
3
2

2
cos
a + b cos
d =

2
0
sin
a + b sin
d
The next is the denominator is denited because of a > b > 0. Then we apply the method
of evaluation of real integrals with residue calculus 1. We get z = e
i
, so that
_
_
_
d =
dz
iz
sin =
zz
1
2i
Then the integral becomes

2
0
sin
a + b sin
d =

|z|=1
_
z
2
1
_
dz
iz (bz
2
+ 2iaz b)
We need to face with the equation bz
2
+ 2iaz b = 0 and have 2 roots easily.
z =
ia i

a
2
b
2
b
But the only point, z =
ia + i

a
2
b
2
b
, is in the unit circle, the reader can verify them-
selves.
Now we only compute the residue at this point and the point z = 0.

2
0
sin
a + b sin
d = 2i
_
Res
_
z
2
1
bz
2
+ 2iaz b
;
ia + i

a
2
b
2
b
_
+ Res
_
z
2
1
bz
2
+ 2iaz b
; 0
_
_
=
2
b
_
1
a

a
2
b
2
_
Exercise 6.11. (E. 7, E. 8, E. 9 P. 364)
Using residue calculus, establish the following identities.
1.

2
0
d
(a + b sin )
2
=
2a
_

a
2
b
2
_
3
for a > b 0.
2.

2
0
d
a + sin
2

=
2
_
a (a + 1)
for a > 0.
Solution.
1. Like the exercises above, the denominator is denited because of a > b 0. Then we
apply the method of evaluation of real integrals with residue calculus 1. We get z = e
i
,
so that
181
6 RESIDUES AND THEIR USE IN INTEGRATION
_
_
_
d =
dz
iz
sin =
zz
1
2i
Then the integral becomes

2
0
d
(a + b sin )
2
=

|z|=1
4zdz
i (bz
2
+ 2iaz b)
2
Using the root of exercise 4, we still choose z =
ia + i

a
2
b
2
b
is in the unit circle but
it is a second-order pole. Then we have

2
0
d
(a + b sin )
2
= 2iRes
_
4z
i (bz
2
+ 2iaz b)
2
; z
1
=
ia + i

a
2
b
2
b
_
= 2 lim
zz
1
d
dz
_
(z z
1
)
2
4z
(bz
2
+ 2iaz b)
2
_
=
2
_

a
2
b
2
_
3
2. We will reduce the degree of sin
2
by using formula sin
2
=
1 cos 2
2
and the denom-
inator is denited because of a > 0. Then we apply the method of evaluation of real
integrals with residue calculus 1. We get z = e
i
, so that
_
_
_
d =
dz
iz
cos 2 =
z
2
+z
2
2
Then the integral becomes

2
0
2d
2a + 1 cos 2
=

|z|=1
4zdz
i [z
4
z
2
(4a + 2) + 1]
We need to solve the equation z
4
z
2
(4a + 1) + 1 = 0 then we have 4 roots.
z =
_
2a + 1
_
a (4a + 1)
But there are only 2 points in the unit circle, these are z =
_
2a + 1
_
a (4a + 1).
Thus if we call these are z
1
; z
2
respectively, we will have

2
0
2d
2a + 1 cos 2
= 8i
_
Res
_
z
i [z
4
z
2
(4a + 2) + 1]
; z
1
_
+ Res
_
z
i [z
4
z
2
(4a + 2) + 1]
; z
2
__
=
2
_
a (a + 1)
182
6 RESIDUES AND THEIR USE IN INTEGRATION
6.5 EVALUATION OF INTEGRALS II
We think 4 exercises below are the basic things to help the reader understanding in this section.
Exercise 6.12. (E. 19, E. 20, E. 21 P. 371)
Evaluate the following integrals by means of residue calculus.
1.

dx
x
2
+ x + 1
2.

dx
(x
2
+ x + 1) (x
2
+ 1)
3.

x
4
dx
x
6
+ 1
Solution.
In 3 exercises, the denominator is nonzero for all real numbers and the degree of it diers
from the numerator by 2 or more. Thus at all poles in u.h.p, we have the result respectively.
1. This follows that

dx
x
2
+ x + 1
= 2i

Res
1
z
2
+ z + 1
Then we solve the equation z
2
+ z + 1 = 0 and get
z =
1 i

3
2
= e
i
2
3
; e
i
2
3
Hence, this function has a simple pole in the u.h.p at e
i
2
3
. We will evaluate the residue
at this point.
Res
_
1
z
2
+ z + 1
; e
i
2
3
_
=
1
(z
2
+ z + 1)

z=e
i
2
3
=
1
2e
i
2
3
+ 1
=
1
i

3
So that the result is
2

3
.
2. This follows that

dx
(x
2
+ x + 1) (x
2
+ 1)
= 2i

Res
1
(z
2
+ z + 1) (z
2
+ 1)
Then by solving the equations z
2
+ z + 1 = 0 and z
2
+ 1 = 0, we get
z =
1 i

3
2
= e
i
2
3
; e
i
2
3
183
6 RESIDUES AND THEIR USE IN INTEGRATION
and
z = i
Hence, this function has 2 simple poles in the u.h.p at e
i
2
3
and i . We will evaluate the
residues at these point.
Res
_
1
(z
2
+ z + 1) (z
2
+ 1)
; e
i
2
3
_
=
1
(z
2
+ z + 1)

(z
2
+ 1)

z=e
i
2
3
=
1
_
2e
i
2
3
+ 1
__
e
i
4
3
+ 1
_
=
2
i

3
_
1 i

3
_
Res
_
1
(z
2
+ z + 1) (z
2
+ 1)
; i
_
=
1
(z
2
+ z + 1) (z
2
+ 1)

z=i
=
1
2
So that the result is

3
.
3. This follows that

x
4
dx
x
6
+ 1
= 2i

Res
z
4
(z
4
z
2
+ 1) (z
2
+ 1)
Then by solving the equations z
4
z
2
+ 1 = 0 and z
2
+ 1 = 0, we get
z =

3 i
2
= e
i
5
6
; e
i
5
6
and
z =

3 i
2
= e
i

6
; e
i

6
and
z = i
Hence, this function has 2 simple poles in the u.h.p at e
i

6
, e
i
5
6
and i . We will evaluate
the residues at these point.
184
6 RESIDUES AND THEIR USE IN INTEGRATION
Res
_
z
4
(z
4
z
2
+ 1) (z
2
+ 1)
; e
i

6
_
= lim
ze
i

6
z
4
_
z e
i

6
__
z e
i
5
6
__
z e
i
5
6
_
(z
2
+ 1)
=
1 + i

3
3i
_
3 + i
_
2
Res
_
z
4
(z
4
z
2
+ 1) (z
2
+ 1)
; e
i
5
6
_
= lim
ze
i
5
6
z
4
_
z e
i

6
__
z e
i

6
__
z e
i
5
6
_
(z
2
+ 1)
=
1 i

3
3i
_
3 i
_
2
Res
_
z
4
(z
4
z
2
+ 1) (z
2
+ 1)
; i
_
=
z
4
(z
4
z
2
+ 1) (z
2
+ 1)

z=i
=
1
6i
So that the result is
2
3
.
Exercise 6.13. (E. 28 P. 372)
Show that for a, b, c real and b
2
< 4ac, the following hold

dx
ax
2
+ bx + c
=
2

4ac b
2
Solution.
Because of b
2
< 4ac, the denominator on the right hand is denited and the equation ax
2
+
bx + c = 0 has 2 complex roots. This means that
x =
b i

4ac b
2
2a
Assume that a > 0, we choose the point
b + i

4ac b
2
2a
, we will evaluate the residue at this
point.

dx
ax
2
+ bx + c
= 2iRes
_
1
az
2
+ bz + c
;
b + i

4ac b
2
2a
_
= 2i
_
1
2az + b

z=
b+i

4acb
2
2a
_
=
2

4ac b
2
And if we consider the case a < 0, by choosing the point
b i

4ac b
2
2a
and applying
method above, we still have the result is the right hand.
185
6 RESIDUES AND THEIR USE IN INTEGRATION
6.6 EVALUATION OF INTEGRALS III
Exercises 1, 3, 5, 7, 13, 24 are expressed there to solve easily.
Exercise 6.14. (E. 1, E. 3, E. 5, E. 7, E. 13 P. 382)
Evaluate the following integrals by residue calculus. Use Cauchy principal values and take
advantage of even and odd symmetries where appropriate
1.

cos (2x)
x
2
+ 9
dx
2.

xe
ix
(x 1)
2
+ 9
dx
3.

xsin (2x)
x
2
+ x + 1
dx
4.

_
x
3
+ x
2
_
cos
_
2x
_
x
4
+ 1
dx
5.

0
cos (x)
(x
2
+ 4)
2
dx
Solution.
We consider arc C below.
Now we know completely how to evaluate the residue at each point in some sections before.
Then we only need to solve some exercises by using equations which is gotten in this section.
1.
186
6 RESIDUES AND THEIR USE IN INTEGRATION

cos (2x)
x
2
+ 9
dx = 1
_
_

e
i2x
x
2
+ 9
dx
_
_
= 1
_
2i

Res
_
e
i2z
z
2
+ 9
__
= 1
_
2i.Res
_
e
i2z
z
2
+ 9
, z
1
= 3i
__
= 1
_
2i.
e
i2z
2z

z=3i
_
= 1
_
2i.
e
6
6i
_
=

3
e
6
2.

xe
ix
(x 1)
2
+ 9
dx = 2i

Res
_
ze
iz
(z 1)
2
+ 9
_
= 2i.Res
_
ze
iz
(z 1)
2
+ 9
, z
1
= 1 + 3i
_
= 2i.
ze
iz
2 (z 1)

z=1+3i
= 2i.
(1 + 3i) e
3+i
6i
=

3
. (1 + 3i) e
3+i
3.
187
6 RESIDUES AND THEIR USE IN INTEGRATION

xsin (2x)
x
2
+ x + 1
dx =
_
2i

Res
_
ze
2zi
z
2
+ z + 1
__
=
_
2i.Res
_
ze
2zi
z
2
+ z + 1
, z
1
=
1 +

3i
2
__
=
_
2i.
ze
2zi
2z + 1

z=(1+

3i)/2
_
=
_

_
1 +

3i
_
e

3i

3
_
=
_
.e

3
_
1 +

3i
_
(cos 1 i sin 1)
_
=
_
.e

3
_
cos 1 + i sin 1 +

3i cos 1 +

3 sin 1
_
_
=
.e

3
.
_
sin 1 +

3 cos 1
_
=
2.e

3
sin
_
2
3
1
_
4.

_
x
3
+ x
2
_
cos
_
2x
_
x
4
+ 1
dx = 1
_
2i

Res
_
_
z
3
+ z
2
_
e
i

2z
z
4
+ 1
__
= 1
_
2i.Res
_
_
z
3
+ z
2
_
e
i

2z
z
4
+ 1
, e
i
4
_
+ 2i.Res
_
_
z
3
+ z
2
_
e
i

2z
z
4
+ 1
, e
i3
4
__
= 1
_
2i.
_
z
3
+ z
2
_
e
i

2z
4z
3

z=e
i
4
+ 2i.
_
z
3
+ z
2
_
e
i

2z
4z
3

z=e
i3
4
_
= 1
_

_
2i.
_
_
e
i3
4
+ e
i
2
_
_
e
1+i
4e
i3
4
+ 2i.
_
_
e
i9
4
+ e
i3
2
_
_
e
1i
4e
i9
4
_

_
=

2
e
1
(cos 1 sin 1)
5. Since
cos (x)
(x
2
+ 4)
2
=
cos (x)
_
(x)
2
+ 4
_
2
, we can derive that
188
6 RESIDUES AND THEIR USE IN INTEGRATION

0
cos (x)
(x
2
+ 4)
2
dx =
1
2
1
_
2i

Res
_
e
iz
(z
2
+ 4)
2
__
= 1
_
iRes
_
e
iz
(z
2
+ 4)
2
, z
1
= 2i
__
= 1
_

_
i lim
z2i
d
_
(z 2i)
2
e
iz
(z
2
+ 4)
2
_
dz
_

_
= 1
_

_
i lim
z2i
d
_
e
iz
(z + 2i)
2
_
dz
_

_
= 1
_
i lim
z2i
ie
iz
(z + 2i)
2
2 (z + 2i) e
iz
(z + 2i)
4
_
= 1
_
i
ie
2
(2i + 2i)
2
2 (2i + 2i) e
2
(2i + 2i)
4
_
= 1
_
e
2
16 + 8
256
_
=
3e
2
32
189
6 RESIDUES AND THEIR USE IN INTEGRATION
Exercise 6.15. (E. 24 P. 384)
To establish the well-known result

0
sin x
x
dx =

2
we proceed as follows:
1. Show that
f (z) =
e
iz
1
z
has a removable singularity at z = 0. How should f (0)be defined to remove the singularity?
2. Using the contour of Fig 6.5-1, prove that
R

R
e
ix
1
x
dx +

C
1
e
iz
1
z
= 0
and also
R

R
cos x 1
x
dx + i
R

R
sin x
x
dx =

C
1
1
z
dz

C
1
e
iz
z
dz
3. Evaluate the first integral on the above right by using the polar representation of C
1
:
z = Re
i
, 0 . Pass to the limit R and explain why the second integral on
the right goes to zero. Thus prove that

cos x 1
x
dx = 0

sin x
x
dx = 0
and finally that

0
sin x
x
dx =

2
190
6 RESIDUES AND THEIR USE IN INTEGRATION
Solution.
1. We have
f (z) =
e
iz
1
z
=

n=0
i
n+1
(n + 1)!
z
n
Thus, f (z) has a removable singularity at z = 0.
And we define that
f (0) = lim
z0
f (z)
= lim
z0
ie
iz
1
= i
2. By using a contour of integration containing a semicircular arc in the u.h.p, we get
R

R
e
ix
1
x
dx +

C
1
e
iz
1
z
= 2i

Res
_
e
iz
1
z
_
= 2iRes
_
e
iz
1
z
, z
1
= 0
_
= 2i. lim
z0
_
z.
e
iz
1
z
_
= 2i. lim
z0
_
e
iz
1
_
= 0
Thus,
R

R
cos x 1
x
dx +
R

R
sin x
x
dx =

C
1
1
z
dz

C
1
e
iz
z
dz
We symbolize the equation above is (*).
3. We will evaluate I =

C
1
1
z
dz before by using the polar representation of C
1
: z = Re
i
, 0
. Then we obtain dz = iRe
i
d.
Thus
191
6 RESIDUES AND THEIR USE IN INTEGRATION
I =

0
1
Re
i
iRe
i
d
=

0
id = i
We continue to symbolize this equation is (**). The next is evaluation of J =

C
1
e
iz
z
dz.
We get a comment that since lim
z

1
z

= 0, lim
R

C
1
e
iz
z
dz = 0 (***)
From (*),(**) and (***), we have

sin x
x
dx =
And
sin x
x
is even function, so that

0
sin x
x
dx =

2
Hence, we complete the proof.
192

MATHEMATICAL BOOKS














To myself I am only a child playing on the beach,
while vast oceans of truth lie undiscovered before me. I. Newton.

I am sure that no subject loses more than mathematics
by any attempt to dissociate it from its history. J.W.L. Glaisher.

You might also like